| NCERT Exemplar Solutions | ||||||
|---|---|---|---|---|---|---|
| 6th | 7th | 8th | 9th | 10th | 11th | 12th |
Chapter 8 Binomial Theorem
Welcome to this comprehensive resource featuring detailed solutions for the Class 11 NCERT Exemplar problems centered on the powerful Binomial Theorem. While the theorem itself provides a straightforward formula for expanding binomials raised to a positive integral power, the Exemplar questions are designed to push students beyond simple expansions. They demand a deeper understanding of the theorem's structure, properties, and applications, often requiring more sophisticated analysis of specific terms and coefficients within the expansion, as well as skillful manipulation of binomial coefficients. Mastering these advanced applications is crucial for further studies in algebra, calculus, probability, and other fields.
The solutions begin by reinforcing the statement of the Binomial Theorem for any positive integer $n$: $(a+b)^n = \sum_{r=0}^{n} \binom{n}{r} a^{n-r} b^r = \binom{n}{0}a^n + \binom{n}{1}a^{n-1}b + \binom{n}{2}a^{n-2}b^2 + \dots + \binom{n}{n}b^n$ where $\binom{n}{r} = {}^nC_r = \frac{n!}{r!(n-r)!}$ are the binomial coefficients. While basic expansion problems are covered, the Exemplar often introduces expansions where the terms $a$ and $b$ are themselves more complex expressions involving roots, fractions, or multiple variables (e.g., expanding $(x^2 - \frac{2}{x})^{10}$), demanding careful attention to exponents and coefficients during the expansion process.
A major focus of the Exemplar, and thus these solutions, is the detailed analysis of specific terms within a binomial expansion. Central to this is the proficient use of the general term formula, which represents the $(r+1)^{th}$ term in the expansion of $(a+b)^n$: $T_{r+1} = \binom{n}{r} a^{n-r} b^r$ Our solutions demonstrate the systematic application of this formula to address various common and challenging Exemplar tasks:
- Finding a specific term in the expansion, such as the $4^{th}$ term (which corresponds to $r=3$).
- Identifying and calculating the middle term(s). Solutions address both cases: when $n$ is even (one middle term at $r = n/2$) and when $n$ is odd (two middle terms at $r = (n-1)/2$ and $r = (n+1)/2$).
- Finding the term independent of $x$ (or any other variable). This involves setting up the general term, collecting all powers of $x$, and solving for the value of $r$ that makes the resulting exponent of $x$ equal to zero.
- Determining the coefficient of a specific power of $x$ (e.g., finding the coefficient of $x^5$). Similar to finding the term independent of $x$, this requires setting the exponent of $x$ in the general term equal to the desired power (e.g., 5) and solving for $r$. The coefficient is then calculated using this value of $r$.
Exemplar problems often present these tasks within expansions involving complex terms for $a$ and $b$, necessitating careful algebraic simplification and exponent handling, skills meticulously illustrated in the solutions.
Properties of binomial coefficients ($\binom{n}{r}$) themselves are another significant area explored. While direct proofs using the formula $\frac{n!}{r!(n-r)!}$ are standard, the Exemplar might require proving identities involving these coefficients, such as Pascal's identity $\binom{n}{r} + \binom{n}{r+1} = \binom{n+1}{r+1}$, the symmetry property $\binom{n}{r} = \binom{n}{n-r}$, or the sum identity $\sum_{r=0}^{n} \binom{n}{r} = 2^n$. Solutions may demonstrate proofs using combinatorial arguments or by cleverly applying the binomial theorem itself (e.g., proving the sum identity by setting $a=1, b=1$ in $(a+b)^n$). Applications covered might include using the binomial theorem for approximations, such as estimating $(1.01)^6$ by expanding $(1+0.01)^6$ and considering the first few terms, although finding the numerically greatest term is a less common but possible extension. These solutions, addressing all typical Exemplar formats (MCQs, Fill-in-the-Blanks, True/False, Short/Long Answer), provide the clear steps, meticulous calculations, and strategic insights needed to master the versatile applications of the Binomial Theorem.
Solved Examples
Example 1 to 10 (Short Answer Type Questions)
Example 1: Find the rth term in the expansion of $\left( x + \frac{1}{x} \right)^{2r}$
Answer:
Given:
The given binomial expansion is $\left( x + \frac{1}{x} \right)^{2r}$.
To Find:
The rth term in the expansion.
Solution:
The general term, $T_{k+1}$, in the binomial expansion of $(a+b)^n$ is given by the formula:
$T_{k+1} = \binom{n}{k} a^{n-k} b^k$
For the given expansion $\left( x + \frac{1}{x} \right)^{2r}$, we compare it with $(a+b)^n$ to identify the values of $a$, $b$, and $n$:
$a = x$
$b = \frac{1}{x}$
$n = 2r$
We are asked to find the rth term of the expansion. The rth term is denoted by $T_r$.
Using the general term formula $T_{k+1} = \binom{n}{k} a^{n-k} b^k$, we set $k+1$ equal to the term number we want to find, which is $r$.
$k+1 = r$
Solving for $k$, we get:
$k = r-1$
Now, substitute the values of $n$, $k$, $a$, and $b$ into the general term formula:
$T_r = T_{(r-1)+1} = \binom{2r}{r-1} (x)^{2r - (r-1)} \left( \frac{1}{x} \right)^{r-1}$
Simplify the exponents of $x$ and $\frac{1}{x}$:
$T_r = \binom{2r}{r-1} x^{2r - r + 1} (x^{-1})^{r-1}$
$T_r = \binom{2r}{r-1} x^{r + 1} x^{-(r-1)}$
$T_r = \binom{2r}{r-1} x^{r + 1} x^{-r + 1}$
Using the property of exponents $x^m \cdot x^p = x^{m+p}$, combine the terms with $x$:
$T_r = \binom{2r}{r-1} x^{(r+1) + (-r+1)}$
$T_r = \binom{2r}{r-1} x^{r+1-r+1}$
$T_r = \binom{2r}{r-1} x^2$
Therefore, the rth term in the expansion of $\left( x + \frac{1}{x} \right)^{2r}$ is $\binom{2r}{r-1} x^2$.
The final answer is $\boxed{\binom{2r}{r-1} x^2}$.
Example 2: Expand the following (1 – x + x2)4
Answer:
Given:
The expression to be expanded is $(1 - x + x^2)^4$.
To Expand:
We need to find the complete expansion of the given expression.
Solution (using Multinomial Theorem):
The multinomial theorem states that the expansion of $(a_1 + a_2 + \dots + a_m)^n$ is given by:
$\sum_{n_1+n_2+\dots+n_m=n} \frac{n!}{n_1! n_2! \dots n_m!} a_1^{n_1} a_2^{n_2} \dots a_m^{n_m}$
In this case, we have a trinomial $(1 + (-x) + x^2)^4$. So, $m=3$, $n=4$, $a_1=1$, $a_2=-x$, and $a_3=x^2$.
The general term in the expansion is $\frac{4!}{n_1! n_2! n_3!} (1)^{n_1} (-x)^{n_2} (x^2)^{n_3}$, where $n_1, n_2, n_3$ are non-negative integers such that $n_1 + n_2 + n_3 = 4$.
The general term simplifies to:
$T = \frac{4!}{n_1! n_2! n_3!} (1) (-1)^{n_2} x^{n_2} x^{2n_3} = \frac{4!}{n_1! n_2! n_3!} (-1)^{n_2} x^{n_2 + 2n_3}$
We need to find all combinations of non-negative integers $(n_1, n_2, n_3)$ such that $n_1 + n_2 + n_3 = 4$ and calculate the corresponding terms:
| $(n_1, n_2, n_3)$ | $n_1+n_2+n_3=4$ | Coefficient $\frac{4!}{n_1! n_2! n_3!}$ | $(-1)^{n_2}$ | $x^{n_2+2n_3}$ | Term |
| (4, 0, 0) | 4 | $\frac{24}{24 \cdot 1 \cdot 1} = 1$ | $(-1)^0 = 1$ | $x^{0+0} = x^0 = 1$ | $1 \cdot 1 \cdot 1 = 1$ |
| (3, 1, 0) | 4 | $\frac{24}{6 \cdot 1 \cdot 1} = 4$ | $(-1)^1 = -1$ | $x^{1+0} = x^1 = x$ | $4 \cdot (-1) \cdot x = -4x$ |
| (3, 0, 1) | 4 | $\frac{24}{6 \cdot 1 \cdot 1} = 4$ | $(-1)^0 = 1$ | $x^{0+2} = x^2$ | $4 \cdot 1 \cdot x^2 = 4x^2$ |
| (2, 2, 0) | 4 | $\frac{24}{2 \cdot 2 \cdot 1} = 6$ | $(-1)^2 = 1$ | $x^{2+0} = x^2$ | $6 \cdot 1 \cdot x^2 = 6x^2$ |
| (2, 1, 1) | 4 | $\frac{24}{2 \cdot 1 \cdot 1} = 12$ | $(-1)^1 = -1$ | $x^{1+2} = x^3$ | $12 \cdot (-1) \cdot x^3 = -12x^3$ |
| (2, 0, 2) | 4 | $\frac{24}{2 \cdot 1 \cdot 2} = 6$ | $(-1)^0 = 1$ | $x^{0+4} = x^4$ | $6 \cdot 1 \cdot x^4 = 6x^4$ |
| (1, 3, 0) | 4 | $\frac{24}{1 \cdot 6 \cdot 1} = 4$ | $(-1)^3 = -1$ | $x^{3+0} = x^3$ | $4 \cdot (-1) \cdot x^3 = -4x^3$ |
| (1, 2, 1) | 4 | $\frac{24}{1 \cdot 2 \cdot 1} = 12$ | $(-1)^2 = 1$ | $x^{2+2} = x^4$ | $12 \cdot 1 \cdot x^4 = 12x^4$ |
| (1, 1, 2) | 4 | $\frac{24}{1 \cdot 1 \cdot 2} = 12$ | $(-1)^1 = -1$ | $x^{1+4} = x^5$ | $12 \cdot (-1) \cdot x^5 = -12x^5$ |
| (1, 0, 3) | 4 | $\frac{24}{1 \cdot 1 \cdot 6} = 4$ | $(-1)^0 = 1$ | $x^{0+6} = x^6$ | $4 \cdot 1 \cdot x^6 = 4x^6$ |
| (0, 4, 0) | 4 | $\frac{24}{1 \cdot 24 \cdot 1} = 1$ | $(-1)^4 = 1$ | $x^{4+0} = x^4$ | $1 \cdot 1 \cdot x^4 = x^4$ |
| (0, 3, 1) | 4 | $\frac{24}{1 \cdot 6 \cdot 1} = 4$ | $(-1)^3 = -1$ | $x^{3+2} = x^5$ | $4 \cdot (-1) \cdot x^5 = -4x^5$ |
| (0, 2, 2) | 4 | $\frac{24}{1 \cdot 2 \cdot 2} = 6$ | $(-1)^2 = 1$ | $x^{2+4} = x^6$ | $6 \cdot 1 \cdot x^6 = 6x^6$ |
| (0, 1, 3) | 4 | $\frac{24}{1 \cdot 1 \cdot 6} = 4$ | $(-1)^1 = -1$ | $x^{1+6} = x^7$ | $4 \cdot (-1) \cdot x^7 = -4x^7$ |
| (0, 0, 4) | 4 | $\frac{24}{1 \cdot 1 \cdot 24} = 1$ | $(-1)^0 = 1$ | $x^{0+8} = x^8$ | $1 \cdot 1 \cdot x^8 = x^8$ |
Now, we sum the terms with the same power of $x$:
$x^0$: $1$
$x^1$: $-4x$
$x^2$: $4x^2 + 6x^2 = 10x^2$
$x^3$: $-12x^3 - 4x^3 = -16x^3$
$x^4$: $6x^4 + 12x^4 + x^4 = 19x^4$
$x^5$: $-12x^5 - 4x^5 = -16x^5$
$x^6$: $4x^6 + 6x^6 = 10x^6$
$x^7$: $-4x^7$
$x^8$: $x^8$
Combining these terms, the expansion is:
$1 - 4x + 10x^2 - 16x^3 + 19x^4 - 16x^5 + 10x^6 - 4x^7 + x^8$
Alternate Solution (by squaring twice):
We can write $(1 - x + x^2)^4$ as $((1 - x + x^2)^2)^2$.
First, let's expand $(1 - x + x^2)^2$:
$(1 - x + x^2)^2 = (1 - (x - x^2))^2 = 1^2 - 2(1)(x - x^2) + (x - x^2)^2$
$= 1 - 2x + 2x^2 + (x^2 - 2(x)(x^2) + (x^2)^2)$
$= 1 - 2x + 2x^2 + x^2 - 2x^3 + x^4$
$= 1 - 2x + 3x^2 - 2x^3 + x^4$
Now, we need to square this result:
$(1 - 2x + 3x^2 - 2x^3 + x^4)^2$
We can expand this as the square of a polynomial:
$(a+b+c+d+e)^2 = a^2+b^2+c^2+d^2+e^2 + 2ab+2ac+2ad+2ae+2bc+2bd+2be+2cd+2ce+2de$
Let $a=1, b=-2x, c=3x^2, d=-2x^3, e=x^4$.
Squares of terms:
$a^2 = (1)^2 = 1$
$b^2 = (-2x)^2 = 4x^2$
$c^2 = (3x^2)^2 = 9x^4$
$d^2 = (-2x^3)^2 = 4x^6$
$e^2 = (x^4)^2 = x^8$
Twice the products of pairs of terms:
$2ab = 2(1)(-2x) = -4x$
$2ac = 2(1)(3x^2) = 6x^2$
$2ad = 2(1)(-2x^3) = -4x^3$
$2ae = 2(1)(x^4) = 2x^4$
$2bc = 2(-2x)(3x^2) = -12x^3$
$2bd = 2(-2x)(-2x^3) = 8x^4$
$2be = 2(-2x)(x^4) = -4x^5$
$2cd = 2(3x^2)(-2x^3) = -12x^5$
$2ce = 2(3x^2)(x^4) = 6x^6$
$2de = 2(-2x^3)(x^4) = -4x^7$
Summing all these terms and collecting by powers of $x$:
$x^0$: $1$
$x^1$: $-4x$
$x^2$: $4x^2 + 6x^2 = 10x^2$
$x^3$: $-4x^3 - 12x^3 = -16x^3$
$x^4$: $9x^4 + 2x^4 + 8x^4 = 19x^4$
$x^5$: $-4x^5 - 12x^5 = -16x^5$
$x^6$: $4x^6 + 6x^6 = 10x^6$
$x^7$: $-4x^7$
$x^8$: $x^8$
Combining these terms, the expansion is:
$1 - 4x + 10x^2 - 16x^3 + 19x^4 - 16x^5 + 10x^6 - 4x^7 + x^8$
Both methods yield the same result.
The final expanded form of $(1 - x + x^2)^4$ is $1 - 4x + 10x^2 - 16x^3 + 19x^4 - 16x^5 + 10x^6 - 4x^7 + x^8$.
The final answer is $\boxed{1 - 4x + 10x^2 - 16x^3 + 19x^4 - 16x^5 + 10x^6 - 4x^7 + x^8}$.
Example 3: Find the 4th term from the end in the expansion of $\left( \frac{x^3}{2} − \frac{2}{x^2} \right)^9$
Answer:
Given:
The binomial expansion $\left( \frac{x^3}{2} − \frac{2}{x^2} \right)^9$.
To Find:
The 4th term from the end of the expansion.
Solution:
The given expansion is of the form $(a+b)^n$, where $a = \frac{x^3}{2}$, $b = -\frac{2}{x^2}$, and $n = 9$.
The total number of terms in the expansion of $(a+b)^n$ is $n+1$.
In this case, the total number of terms is $9+1 = 10$.
To find the rth term from the end of an expansion with $N$ terms, we find the $(N - r + 1)^{\text{th}}$ term from the beginning.
Here, $N=10$ and $r=4$.
The 4th term from the end is the $(10 - 4 + 1)^{\text{th}}$ term from the beginning.
This is the $(6+1)^{\text{th}}$ term from the beginning, which is the 7th term from the beginning.
The general term $T_{k+1}$ in the expansion of $(a+b)^n$ is given by:
$T_{k+1} = \binom{n}{k} a^{n-k} b^k$
We need to find the 7th term from the beginning, so we set $k+1 = 7$, which means $k=6$.
Substitute the values $n=9$, $k=6$, $a = \frac{x^3}{2}$, and $b = -\frac{2}{x^2}$ into the general term formula:
$T_7 = \binom{9}{6} \left(\frac{x^3}{2}\right)^{9-6} \left(-\frac{2}{x^2}\right)^6$
$T_7 = \binom{9}{6} \left(\frac{x^3}{2}\right)^3 \left(-\frac{2}{x^2}\right)^6$
Calculate the binomial coefficient $\binom{9}{6}$:
$\binom{9}{6} = \binom{9}{9-6} = \binom{9}{3} = \frac{9!}{3!(9-3)!} = \frac{9!}{3!6!} = \frac{9 \times 8 \times 7 \times \cancel{6!}}{(3 \times 2 \times 1) \times \cancel{6!}} = \frac{9 \times 8 \times 7}{6} = 3 \times 4 \times 7 = 84$
Calculate the powers of the terms:
$\left(\frac{x^3}{2}\right)^3 = \frac{(x^3)^3}{2^3} = \frac{x^9}{8}$
$\left(-\frac{2}{x^2}\right)^6 = (-1)^6 \left(\frac{2}{x^2}\right)^6 = 1 \cdot \frac{2^6}{(x^2)^6} = \frac{64}{x^{12}}$
Now substitute these values back into the expression for $T_7$:
$T_7 = 84 \cdot \frac{x^9}{8} \cdot \frac{64}{x^{12}}$
Simplify the numerical and variable parts:
$T_7 = 84 \cdot \frac{\cancel{64}^8}{\cancel{8}_1} \cdot \frac{x^9}{x^{12}}$
$T_7 = 84 \cdot 8 \cdot x^{9-12}$
$T_7 = 672 \cdot x^{-3}$
$T_7 = \frac{672}{x^3}$
Alternate Solution:
The rth term from the end in the expansion of $(a+b)^n$ is the same as the rth term from the beginning in the expansion of $(b+a)^n$.
So, the 4th term from the end in $\left( \frac{x^3}{2} − \frac{2}{x^2} \right)^9$ is the 4th term from the beginning in $\left( -\frac{2}{x^2} + \frac{x^3}{2} \right)^9$.
For the expansion $\left( -\frac{2}{x^2} + \frac{x^3}{2} \right)^9$, we have $a' = -\frac{2}{x^2}$, $b' = \frac{x^3}{2}$, and $n=9$.
We need the 4th term from the beginning, $T_4$. For $T_4$, we set $k+1=4$, so $k=3$.
Using the general term formula $T_{k+1} = \binom{n}{k} (a')^{n-k} (b')^k$:
$T_4 = \binom{9}{3} \left(-\frac{2}{x^2}\right)^{9-3} \left(\frac{x^3}{2}\right)^3$
$T_4 = \binom{9}{3} \left(-\frac{2}{x^2}\right)^6 \left(\frac{x^3}{2}\right)^3$
Calculate the terms:
$\binom{9}{3} = 84$ (as calculated before)
$\left(-\frac{2}{x^2}\right)^6 = (-1)^6 \left(\frac{2}{x^2}\right)^6 = 1 \cdot \frac{2^6}{(x^2)^6} = \frac{64}{x^{12}}$
$\left(\frac{x^3}{2}\right)^3 = \frac{(x^3)^3}{2^3} = \frac{x^9}{8}$
Substitute these values back into the expression for $T_4$:
$T_4 = 84 \cdot \frac{64}{x^{12}} \cdot \frac{x^9}{8}$
$T_4 = 84 \cdot \frac{\cancel{64}^8}{\cancel{8}_1} \cdot \frac{x^9}{x^{12}}$
$T_4 = 84 \cdot 8 \cdot x^{9-12}$
$T_4 = 672 \cdot x^{-3}$
$T_4 = \frac{672}{x^3}$
Both methods confirm that the 4th term from the end is $\frac{672}{x^3}$.
The final answer is $\boxed{\frac{672}{x^3}}$.
Example 4: Evaluate: $\left( x^2 − \sqrt{1 − x^2} \right)^4 + \left( x^2 + \sqrt{1 − x^2} \right)^4$.
Answer:
Given:
The expression to be evaluated is $\left( x^2 − \sqrt{1 − x^2} \right)^4 + \left( x^2 + \sqrt{1 − x^2} \right)^4$.
To Evaluate:
We need to simplify and find the value of the given expression.
Solution:
The given expression is in the form $(a-b)^n + (a+b)^n$, where $a = x^2$, $b = \sqrt{1-x^2}$, and $n=4$.
We know the binomial expansions for $(a+b)^n$ and $(a-b)^n$:
$(a+b)^n = \binom{n}{0} a^n b^0 + \binom{n}{1} a^{n-1} b^1 + \binom{n}{2} a^{n-2} b^2 + \binom{n}{3} a^{n-3} b^3 + \dots + \binom{n}{n} a^0 b^n$
$(a-b)^n = \binom{n}{0} a^n b^0 - \binom{n}{1} a^{n-1} b^1 + \binom{n}{2} a^{n-2} b^2 - \binom{n}{3} a^{n-3} b^3 + \dots + (-1)^n \binom{n}{n} a^0 b^n$
Adding these two expansions, the terms with odd powers of $b$ cancel out:
$(a+b)^n + (a-b)^n = 2 \left[ \binom{n}{0} a^n b^0 + \binom{n}{2} a^{n-2} b^2 + \binom{n}{4} a^{n-4} b^4 + \dots \right]$
In this case, $n=4$, $a=x^2$, and $b=\sqrt{1-x^2}$. The sum will include terms with even powers of $b$ up to $b^4$. The relevant values of $k$ are $0, 2, 4$.
So, the expression becomes:
$2 \left[ \binom{4}{0} (x^2)^{4-0} (\sqrt{1-x^2})^0 + \binom{4}{2} (x^2)^{4-2} (\sqrt{1-x^2})^2 + \binom{4}{4} (x^2)^{4-4} (\sqrt{1-x^2})^4 \right]$
Calculate the binomial coefficients:
$\binom{4}{0} = 1$
$\binom{4}{2} = \frac{4!}{2!2!} = \frac{24}{4} = 6$
$\binom{4}{4} = 1$
Calculate the powers of $a$ and $b$:
$(x^2)^{4} = x^8$
$(x^2)^{2} = x^4$
$(x^2)^{0} = 1$
$(\sqrt{1-x^2})^0 = 1$
$(\sqrt{1-x^2})^2 = 1-x^2$
$(\sqrt{1-x^2})^4 = ((1-x^2)^{1/2})^4 = (1-x^2)^2 = 1 - 2x^2 + x^4$
Substitute these values back into the expression:
$2 \left[ 1 \cdot x^8 \cdot 1 + 6 \cdot x^4 \cdot (1-x^2) + 1 \cdot 1 \cdot (1 - 2x^2 + x^4) \right]$
$= 2 \left[ x^8 + 6x^4 - 6x^6 + 1 - 2x^2 + x^4 \right]$
Combine like terms:
$= 2 \left[ x^8 - 6x^6 + (6x^4 + x^4) - 2x^2 + 1 \right]$
$= 2 \left[ x^8 - 6x^6 + 7x^4 - 2x^2 + 1 \right]$
Distribute the factor of 2:
$= 2x^8 - 12x^6 + 14x^4 - 4x^2 + 2$
The final evaluated expression is $2x^8 - 12x^6 + 14x^4 - 4x^2 + 2$.
The final answer is $\boxed{2x^8 - 12x^6 + 14x^4 - 4x^2 + 2}$.
Example 5: Find the coefficient of x11 in the expansion of $\left( x^3 - \frac{2}{x^2} \right)^{12}$ .
Answer:
Given:
The binomial expansion is $\left( x^3 - \frac{2}{x^2} \right)^{12}$.
To Find:
The coefficient of $x^{11}$ in the given expansion.
Solution:
The given expansion is of the form $(a+b)^n$, where $a = x^3$, $b = -\frac{2}{x^2}$, and $n = 12$.
The general term, $T_{k+1}$, in the expansion of $(a+b)^n$ is given by the formula:
$T_{k+1} = \binom{n}{k} a^{n-k} b^k$
Substitute the values of $n$, $a$, and $b$ for the given expansion:
$T_{k+1} = \binom{12}{k} (x^3)^{12-k} \left(-\frac{2}{x^2}\right)^k$
Now, let's simplify the terms involving $x$ to find the power of $x$ in the general term:
$(x^3)^{12-k} = x^{3(12-k)} = x^{36-3k}$
$\left(-\frac{2}{x^2}\right)^k = (-2)^k \left(\frac{1}{x^2}\right)^k = (-2)^k (x^{-2})^k = (-2)^k x^{-2k}$
The variable part of the general term is the product of the terms involving $x$:
$x^{36-3k} \cdot x^{-2k} = x^{(36-3k) + (-2k)} = x^{36-5k}$
We want to find the coefficient of $x^{11}$. Therefore, we set the power of $x$ in the general term equal to 11:
$36 - 5k = 11$
Subtract 36 from both sides:
$-5k = 11 - 36$
$-5k = -25$
Divide by -5:
$k = \frac{-25}{-5}$
$k = 5$
Since $k=5$ is a non-negative integer and $0 \leq 5 \leq 12$, this value of $k$ is valid for the expansion.
The term containing $x^{11}$ corresponds to $k=5$. The coefficient of this term is the part of $T_{k+1}$ that does not involve $x$, which is $\binom{12}{k} (-2)^k$ for $k=5$.
Coefficient of $x^{11} = \binom{12}{5} (-2)^5$
Calculate $\binom{12}{5}$:
$\binom{12}{5} = \frac{12!}{5!(12-5)!} = \frac{12!}{5!7!} = \frac{12 \times 11 \times 10 \times 9 \times 8 \times \cancel{7!}}{(5 \times 4 \times 3 \times 2 \times 1) \times \cancel{7!}} = \frac{12 \times 11 \times 10 \times 9 \times 8}{120}$
$\binom{12}{5} = \frac{\cancel{120}^{1} \times 11 \times \cancel{10}^{1} \times 9 \times 8}{\cancel{120}_{1}} = 11 \times 9 \times 8 = 11 \times 72 = 792$
Calculate $(-2)^5$:
$(-2)^5 = -32$
Now, multiply the results:
Coefficient of $x^{11} = 792 \times (-32)$
$792 \times (-32) = -25344$
The coefficient of $x^{11}$ in the expansion of $\left( x^3 - \frac{2}{x^2} \right)^{12}$ is $-25344$.
The final answer is $\boxed{-25344}$.
Example 6: Determine whether the expansion of $\left( x^2 - \frac{2}{x} \right)^{18}$ will contain a term containing x10?
Answer:
Given:
The binomial expansion is $\left( x^2 - \frac{2}{x} \right)^{18}$.
To Determine:
Whether the expansion contains a term with $x^{10}$.
Solution:
The given expansion is of the form $(a+b)^n$, where $a = x^2$, $b = -\frac{2}{x}$, and $n = 18$.
The general term, $T_{k+1}$, in the expansion of $(a+b)^n$ is given by the formula:
$T_{k+1} = \binom{n}{k} a^{n-k} b^k$
Substitute the values of $n$, $a$, and $b$ for the given expansion:
$T_{k+1} = \binom{18}{k} (x^2)^{18-k} \left(-\frac{2}{x}\right)^k$
Simplify the terms involving $x$ to find the power of $x$ in the general term:
$(x^2)^{18-k} = x^{2(18-k)} = x^{36-2k}$
$\left(-\frac{2}{x}\right)^k = (-2)^k \left(\frac{1}{x}\right)^k = (-2)^k (x^{-1})^k = (-2)^k x^{-k}$
The variable part of the general term is the product of the terms involving $x$:
$x^{36-2k} \cdot x^{-k} = x^{(36-2k) - k} = x^{36-3k}$
To determine if there is a term containing $x^{10}$, we set the power of $x$ in the general term equal to 10:
$36 - 3k = 10$
Now, we solve this equation for $k$:
$36 - 10 = 3k$
$26 = 3k$
$k = \frac{26}{3}$
The index $k$ in the general term $T_{k+1}$ must be a non-negative integer, where $0 \leq k \leq n$. In this case, $n=18$, so $k$ must be an integer such that $0 \leq k \leq 18$.
Since the calculated value of $k = \frac{26}{3}$ is not an integer, there is no integer value of $k$ that will result in a term with $x^{10}$.
Therefore, the expansion of $\left( x^2 - \frac{2}{x} \right)^{18}$ does not contain a term with $x^{10}$.
The final answer is $\boxed{\text{No}}$.
Example 7: Find the term independent of x in the expansion of $\left( \frac{\sqrt{x}}{\sqrt{3}} + \frac{\sqrt{3}}{2x^3} \right)^{10}$
Answer:
Given:
The binomial expansion is $\left( \frac{\sqrt{x}}{\sqrt{3}} + \frac{\sqrt{3}}{2x^3} \right)^{10}$.
To Find:
The term independent of $x$ in the expansion.
Solution:
The given expansion is of the form $(a+b)^n$, where $a = \frac{\sqrt{x}}{\sqrt{3}}$, $b = \frac{\sqrt{3}}{2x^3}$, and $n = 10$.
We can write $a$ and $b$ using exponents:
$a = \frac{x^{1/2}}{3^{1/2}} = \frac{1}{\sqrt{3}} x^{1/2}$
$b = \frac{3^{1/2}}{2 x^3} = \frac{\sqrt{3}}{2} x^{-3}$
The general term, $T_{k+1}$, in the expansion of $(a+b)^n$ is given by the formula:
$T_{k+1} = \binom{n}{k} a^{n-k} b^k$
Substitute the values of $n$, $a$, and $b$ for the given expansion:
$T_{k+1} = \binom{10}{k} \left(\frac{1}{\sqrt{3}} x^{1/2}\right)^{10-k} \left(\frac{\sqrt{3}}{2} x^{-3}\right)^k$
Now, let's simplify the terms involving $x$ and the constant terms separately.
The terms involving $x$ are:
$(x^{1/2})^{10-k} \cdot (x^{-3})^k = x^{(1/2)(10-k)} \cdot x^{-3k} = x^{\frac{10-k}{2}} \cdot x^{-3k}$
Combine the powers of $x$:
$x^{\frac{10-k}{2} - 3k} = x^{\frac{10-k - 6k}{2}} = x^{\frac{10-7k}{2}}$
The constant terms are:
$\binom{10}{k} \left(\frac{1}{\sqrt{3}}\right)^{10-k} \left(\frac{\sqrt{3}}{2}\right)^k = \binom{10}{k} (3^{-1/2})^{10-k} \frac{(3^{1/2})^k}{2^k}$
$= \binom{10}{k} 3^{-(1/2)(10-k)} \frac{3^{k/2}}{2^k} = \binom{10}{k} 3^{-5 + k/2} \frac{3^{k/2}}{2^k} = \binom{10}{k} \frac{3^{k/2 + k/2}}{3^5 \cdot 2^k} = \binom{10}{k} \frac{3^k}{3^5 \cdot 2^k} = \binom{10}{k} \frac{1}{3^{5-k} 2^k}$
The general term is:
$T_{k+1} = \binom{10}{k} \frac{3^{k-5}}{2^k} x^{\frac{10-7k}{2}}$
For the term independent of $x$, the power of $x$ must be zero. So, we set the exponent of $x$ equal to 0:
$\frac{10-7k}{2} = 0$
Multiply both sides by 2:
$10 - 7k = 0$
Solve for $k$:
$10 = 7k$
$k = \frac{10}{7}$
The index $k$ in the binomial expansion must be a non-negative integer such that $0 \leq k \leq n$. In this case, $n=10$, so $k$ must be an integer from 0 to 10.
Since the calculated value of $k = \frac{10}{7}$ is not an integer, there is no integer value of $k$ for which the power of $x$ is 0.
Therefore, there is no term independent of $x$ in the expansion of $\left( \frac{\sqrt{x}}{\sqrt{3}} + \frac{\sqrt{3}}{2x^3} \right)^{10}$.
The expansion does not contain a term independent of $x$.
The final answer is $\boxed{\text{No term independent of } x}$.
Example 8: Find the middle term in the expansion of $\left( 2ax - \frac{b}{x^2} \right)^{12}$ .
Answer:
Given:
The binomial expansion is $\left( 2ax - \frac{b}{x^2} \right)^{12}$.
To Find:
The middle term in the expansion.
Solution:
The given expansion is of the form $(a+b)^n$, where $a = 2ax$, $b = -\frac{b}{x^2}$, and $n = 12$.
The total number of terms in the expansion of $(a+b)^n$ is $n+1$.
In this case, $n=12$, so the total number of terms is $12+1 = 13$.
Since the number of terms (13) is odd, there is only one middle term.
The position of the middle term is $\left( \frac{n}{2} + 1 \right)^{\text{th}}$ term when $n$ is even.
Here, $n=12$, so the middle term is the $\left( \frac{12}{2} + 1 \right)^{\text{th}} = (6+1)^{\text{th}} = 7^{\text{th}}$ term.
We need to find the 7th term, $T_7$, in the expansion.
The general term, $T_{k+1}$, in the expansion of $(a+b)^n$ is given by the formula:
$T_{k+1} = \binom{n}{k} a^{n-k} b^k$
For the 7th term, we have $k+1 = 7$, so $k=6$.
Substitute the values of $n=12$, $k=6$, $a=2ax$, and $b=-\frac{b}{x^2}$ into the general term formula:
$T_7 = \binom{12}{6} (2ax)^{12-6} \left(-\frac{b}{x^2}\right)^6$
$T_7 = \binom{12}{6} (2ax)^6 \left(-\frac{b}{x^2}\right)^6$
Calculate the binomial coefficient $\binom{12}{6}$:
$\binom{12}{6} = \frac{12!}{6!6!} = \frac{12 \times 11 \times 10 \times 9 \times 8 \times 7 \times \cancel{6!}}{(\cancel{6 \times 5 \times 4 \times 3 \times 2 \times 1}) \times \cancel{6!}} = \frac{12 \times 11 \times 10 \times 9 \times 8 \times 7}{720}$
$\binom{12}{6} = \frac{\cancel{12}^{1} \times 11 \times \cancel{10}^{1} \times \cancel{9}^{3} \times \cancel{8}^{2} \times 7}{\cancel{720}_{60}} = \frac{11 \times 1 \times 3 \times 2 \times 7}{60} = \frac{462}{60}$ (Let's recalculate using simplification before multiplying)
$\binom{12}{6} = \frac{12 \times 11 \times 10 \times 9 \times 8 \times 7}{6 \times 5 \times 4 \times 3 \times 2 \times 1} = \frac{\cancel{12}^{2} \times 11 \times \cancel{10}^{2} \times \cancel{9}^{3} \times \cancel{8}^{1} \times 7}{\cancel{6 \times 5 \times 4 \times 3 \times 2 \times 1}_{1 \times 1 \times 1 \times 1 \times 1 \times 1}}$ (Simplifying carefully)
$\binom{12}{6} = \frac{\cancel{12}^{1} \times 11 \times \cancel{10}^{1} \times \cancel{9}^{1} \times \cancel{8}^{1} \times 7}{\cancel{6}_{1} \times \cancel{5}_{1} \times \cancel{4}_{1} \times \cancel{3}_{1} \times \cancel{2}_{1} \times \cancel{1}_{1}}$ (Trying again with better cancellation visualization)
$\binom{12}{6} = \frac{\cancel{12}^{2} \times 11 \times \cancel{10}^{1} \times \cancel{9}^{1} \times 8 \times 7}{\cancel{6}_{1} \times 5 \times \cancel{4}_{1} \times \cancel{3}_{1} \times \cancel{2}_{1} \times 1} = \frac{2 \times 11 \times 1 \times 1 \times 8 \times 7}{5}$ (Still not right)
Let's do it step by step: $6 \times 5 \times 4 \times 3 \times 2 \times 1 = 720$.
$12 \times 11 \times 10 \times 9 \times 8 \times 7 = 665280$.
$\binom{12}{6} = \frac{665280}{720} = 924$.
Calculate the powers of the terms:
$(2ax)^6 = 2^6 a^6 x^6 = 64 a^6 x^6$
$\left(-\frac{b}{x^2}\right)^6 = (-1)^6 \frac{b^6}{(x^2)^6} = 1 \cdot \frac{b^6}{x^{12}} = \frac{b^6}{x^{12}}$
Substitute these values back into the expression for $T_7$:
$T_7 = 924 \cdot (64 a^6 x^6) \cdot \left(\frac{b^6}{x^{12}}\right)$
$T_7 = 924 \cdot 64 \cdot a^6 \cdot b^6 \cdot \frac{x^6}{x^{12}}$
Simplify the numerical and variable parts:
$924 \times 64 = 59136$
$\frac{x^6}{x^{12}} = x^{6-12} = x^{-6} = \frac{1}{x^6}$
$T_7 = 59136 a^6 b^6 \frac{1}{x^6}$
$T_7 = \frac{59136 a^6 b^6}{x^6}$
The middle term in the expansion is $\frac{59136 a^6 b^6}{x^6}$.
The final answer is $\boxed{\frac{59136 a^6 b^6}{x^6}}$.
Example 9: Find the middle term (terms) in the expansion of $\left( \frac{p}{x} + \frac{x}{p} \right)^9$ .
Answer:
Given:
The binomial expansion is $\left( \frac{p}{x} + \frac{x}{p} \right)^9$.
To Find:
The middle term(s) in the expansion.
Solution:
The given expansion is of the form $(a+b)^n$, where $a = \frac{p}{x}$, $b = \frac{x}{p}$, and $n = 9$.
The total number of terms in the expansion of $(a+b)^n$ is $n+1$.
In this case, $n=9$, so the total number of terms is $9+1 = 10$.
Since the total number of terms (10) is even, there are two middle terms.
The positions of the middle terms are the $\left(\frac{10}{2}\right)^{\text{th}}$ and $\left(\frac{10}{2} + 1\right)^{\text{th}}$ terms.
These are the 5th term and the 6th term.
The general term, $T_{k+1}$, in the expansion of $(a+b)^n$ is given by the formula:
$T_{k+1} = \binom{n}{k} a^{n-k} b^k$
Finding the 5th term:
For the 5th term, we have $k+1 = 5$, so $k=4$.
Substitute $n=9$, $k=4$, $a=\frac{p}{x}$, and $b=\frac{x}{p}$ into the general term formula:
$T_5 = \binom{9}{4} \left(\frac{p}{x}\right)^{9-4} \left(\frac{x}{p}\right)^4$
$T_5 = \binom{9}{4} \left(\frac{p}{x}\right)^5 \left(\frac{x}{p}\right)^4$
Calculate the binomial coefficient $\binom{9}{4}$:
$\binom{9}{4} = \frac{9!}{4!(9-4)!} = \frac{9!}{4!5!} = \frac{9 \times 8 \times 7 \times 6 \times \cancel{5!}}{(4 \times 3 \times 2 \times 1) \times \cancel{5!}} = \frac{9 \times 8 \times 7 \times 6}{24}$
$\binom{9}{4} = \frac{\cancel{9}^{3} \times \cancel{8}^{1} \times 7 \times \cancel{6}^{1}}{\cancel{24}_{1}} = 3 \times 1 \times 7 \times 1 = 126$
Calculate the powers of the terms:
$\left(\frac{p}{x}\right)^5 = \frac{p^5}{x^5}$
$\left(\frac{x}{p}\right)^4 = \frac{x^4}{p^4}$
Substitute these values back into the expression for $T_5$:
$T_5 = 126 \cdot \frac{p^5}{x^5} \cdot \frac{x^4}{p^4}$
$T_5 = 126 \cdot p^{5-4} \cdot x^{4-5}$
$T_5 = 126 p^1 x^{-1} = \frac{126p}{x}$
Finding the 6th term:
For the 6th term, we have $k+1 = 6$, so $k=5$.
Substitute $n=9$, $k=5$, $a=\frac{p}{x}$, and $b=\frac{x}{p}$ into the general term formula:
$T_6 = \binom{9}{5} \left(\frac{p}{x}\right)^{9-5} \left(\frac{x}{p}\right)^5$
$T_6 = \binom{9}{5} \left(\frac{p}{x}\right)^4 \left(\frac{x}{p}\right)^5$
Calculate the binomial coefficient $\binom{9}{5}$:
$\binom{9}{5} = \binom{9}{9-5} = \binom{9}{4} = 126$ (as calculated for $T_5$)
Calculate the powers of the terms:
$\left(\frac{p}{x}\right)^4 = \frac{p^4}{x^4}$
$\left(\frac{x}{p}\right)^5 = \frac{x^5}{p^5}$
Substitute these values back into the expression for $T_6$:
$T_6 = 126 \cdot \frac{p^4}{x^4} \cdot \frac{x^5}{p^5}$
$T_6 = 126 \cdot p^{4-5} \cdot x^{5-4}$
$T_6 = 126 p^{-1} x^1 = \frac{126x}{p}$
The middle terms in the expansion are the 5th term and the 6th term, which are $\frac{126p}{x}$ and $\frac{126x}{p}$.
The final answer is $\boxed{\frac{126p}{x}, \frac{126x}{p}}$.
Example 10: Show that 24n + 4 – 15n – 16, where n ∈ N is divisible by 225.
Answer:
Given:
The expression is $2^{4n + 4} – 15n – 16$, where $n \in \mathbb{N}$.
To Prove:
The expression $2^{4n + 4} – 15n – 16$ is divisible by 225 for all $n \in \mathbb{N}$.
Solution:
Let the given expression be $E$.
$E = 2^{4n + 4} – 15n – 16$
We can rewrite $2^{4n+4}$ as $2^{4(n+1)} = (2^4)^{n+1} = 16^{n+1}$.
So, $E = 16^{n+1} - 15n - 16$.
We know that $16 = 1 + 15$. Let's expand $16^{n+1}$ using the binomial theorem for $(1+x)^m$ with $x=15$ and $m=n+1$.
$(1+x)^m = \binom{m}{0} + \binom{m}{1}x + \binom{m}{2}x^2 + \binom{m}{3}x^3 + \dots + \binom{m}{m}x^m$
Substituting $m=n+1$ and $x=15$, we get:
$(1+15)^{n+1} = \binom{n+1}{0} + \binom{n+1}{1}(15) + \binom{n+1}{2}(15)^2 + \binom{n+1}{3}(15)^3 + \dots + \binom{n+1}{n+1}(15)^{n+1}$
We know that $\binom{n+1}{0} = 1$ and $\binom{n+1}{1} = n+1$.
So, the expansion becomes:
$16^{n+1} = 1 + (n+1)(15) + \binom{n+1}{2}(15)^2 + \binom{n+1}{3}(15)^3 + \dots + (15)^{n+1}$
$16^{n+1} = 1 + 15n + 15 + \binom{n+1}{2}(15^2) + \binom{n+1}{3}(15^3) + \dots + 15^{n+1}$
$16^{n+1} = 16 + 15n + \binom{n+1}{2}(15^2) + \binom{n+1}{3}(15^3) + \dots + 15^{n+1}$
Now, substitute this expansion back into the expression for $E$:
$E = (16 + 15n + \binom{n+1}{2}(15^2) + \binom{n+1}{3}(15^3) + \dots + 15^{n+1}) - 15n - 16$
Group the terms:
$E = (16 - 16) + (15n - 15n) + \binom{n+1}{2}(15^2) + \binom{n+1}{3}(15^3) + \dots + 15^{n+1}$
$E = 0 + 0 + \binom{n+1}{2}(15^2) + \binom{n+1}{3}(15^3) + \dots + 15^{n+1}$
$E = \binom{n+1}{2}(15^2) + \binom{n+1}{3}(15^3) + \dots + \binom{n+1}{n+1}(15)^{n+1}$
We know that $15^2 = 225$. Notice that every term from the first term onwards in this summation has a factor of $15^2$ or a higher power of 15.
We can factor out $15^2$ (which is 225) from each term:
$E = 15^2 \left[ \binom{n+1}{2} + \binom{n+1}{3}(15)^1 + \binom{n+1}{4}(15)^2 + \dots + \binom{n+1}{n+1}(15)^{n-1} \right]$
$E = 225 \left[ \binom{n+1}{2} + 15 \binom{n+1}{3} + 15^2 \binom{n+1}{4} + \dots + 15^{n-1} \binom{n+1}{n+1} \right]$
Let $K = \binom{n+1}{2} + 15 \binom{n+1}{3} + 15^2 \binom{n+1}{4} + \dots + 15^{n-1} \binom{n+1}{n+1}$.
For any natural number $n \in \mathbb{N}$, $n+1$ is an integer greater than or equal to 2.
The binomial coefficients $\binom{n+1}{k}$ are integers for all integers $k$ where $0 \leq k \leq n+1$.
The terms $15, 15^2, \dots, 15^{n-1}$ are also integers for $n \in \mathbb{N}$. (If $n=1$, the sum for $K$ starts from $k=2$ and ends at $k=n+1=2$, so $K = \binom{1+1}{2} = \binom{2}{2} = 1$, which is an integer. If $n \ge 2$, the powers of 15 are positive integers).
Therefore, the sum $K$ is an integer for all $n \in \mathbb{N}$.
So, $E = 225 \times K$, where $K$ is an integer.
This means that the expression $2^{4n+4} - 15n - 16$ is a multiple of 225 for all $n \in \mathbb{N}$.
Hence, $2^{4n + 4} – 15n – 16$ is divisible by 225 for all $n \in \mathbb{N}$.
The final answer is $\boxed{2^{4n + 4} – 15n – 16 \text{ is divisible by } 225}$.
Example 11 to 15 (Long Answer Type Questions)
Example 11: Find numerically the greatest term in the expansion of (2 + 3x)9, where $x = \frac{3}{2}$ .
Answer:
Given:
Expansion: $(2 + 3x)^9$
Value of x: $x = \frac{3}{2}$
To Find:
Numerically greatest term.
Solution:
The expansion is $(2+3x)^9$. With $x=\frac{3}{2}$, the expression inside the parenthesis is $2 + 3(\frac{3}{2}) = 2 + \frac{9}{2}$.
We can rewrite the expansion as $2^9 \left( 1 + \frac{3x}{2} \right)^9$.
Let $y = \frac{3x}{2}$. Substituting $x=\frac{3}{2}$, $y = \frac{3}{2} \cdot \frac{3}{2} = \frac{9}{4}$.
We consider the expansion $(1+y)^n$, where $n=9$ and $y=\frac{9}{4}$.
The numerically greatest term $T_{r+1}$ is found by determining the integer $r$ such that $\left| \frac{T_{r+1}}{T_r} \right| \geq 1$.
$\left| \frac{n-r+1}{r} y \right| \geq 1$
Substituting $n=9$ and $y=\frac{9}{4}$:
$\left| \frac{9-r+1}{r} \cdot \frac{9}{4} \right| \geq 1$
$\left| \frac{10-r}{r} \cdot \frac{9}{4} \right| \geq 1$
Since $1 \leq r \leq 9$, we have $\frac{10-r}{r} \cdot \frac{9}{4} \geq 1$
$9(10-r) \geq 4r$
$90 - 9r \geq 4r$
$90 \geq 13r$
$r \leq \frac{90}{13} \approx 6.923$
The greatest integer value of $r$ satisfying this is $r=6$. This means $T_{r+1} = T_7$ is the numerically greatest term.
The general term in the expansion of $(a+b)^n$ is $T_{k+1} = \binom{n}{k} a^{n-k} b^k$.
For $(2+3x)^9$, $a=2$, $b=3x$, $n=9$. For the 7th term, $k+1=7$, so $k=6$.
$T_7 = \binom{9}{6} (2)^{9-6} (3x)^6$
$T_7 = \binom{9}{6} (2)^3 (3x)^6$
Substitute $x=\frac{3}{2}$:
$T_7 = \binom{9}{6} (2)^3 \left(3 \cdot \frac{3}{2}\right)^6$
$T_7 = \binom{9}{6} (2)^3 \left(\frac{9}{2}\right)^6$
Calculate the terms:
$\binom{9}{6} = \binom{9}{3} = \frac{9 \times 8 \times 7}{3 \times 2 \times 1} = 84$
$(2)^3 = 8$
$\left(\frac{9}{2}\right)^6 = \frac{9^6}{2^6} = \frac{531441}{64}$
$T_7 = 84 \cdot 8 \cdot \frac{531441}{64}$
$T_7 = 84 \cdot \frac{8}{64} \cdot 531441$
$T_7 = 84 \cdot \frac{1}{8} \cdot 531441$
$T_7 = \frac{84}{8} \cdot 531441$
$T_7 = 10.5 \cdot 531441$
$T_7 = 5580130.5$
The numerical value of the greatest term is $5580130.5$.
The final answer is $\boxed{5580130.5}$.
Example 12: If n is a positive integer, find the coefficient of x–1 in the expansion of $(1 + x)^n \left( 1+\frac{1}{x} \right)^n$.
Answer:
Given:
The expression is $(1 + x)^n \left( 1+\frac{1}{x} \right)^n$, where $n$ is a positive integer.
To Find:
The coefficient of $x^{-1}$ in the expansion of the given expression.
Solution:
Let the given expression be $E$.
$E = (1 + x)^n \left( 1+\frac{1}{x} \right)^n$
We can rewrite the second factor:
$1 + \frac{1}{x} = \frac{x+1}{x}$
Substitute this into the expression $E$:
$E = (1 + x)^n \left( \frac{x+1}{x} \right)^n$
Using the property $(ab)^m = a^m b^m$, we can write $\left( \frac{x+1}{x} \right)^n = \frac{(x+1)^n}{x^n}$.
$E = (1 + x)^n \cdot \frac{(1+x)^n}{x^n}$
Using the property $a^m a^p = a^{m+p}$, combine the terms with $(1+x)$:
$E = \frac{(1+x)^{n+n}}{x^n} = \frac{(1+x)^{2n}}{x^n}$
Rewrite $\frac{1}{x^n}$ as $x^{-n}$:
$E = x^{-n} (1+x)^{2n}$
Now, we expand $(1+x)^{2n}$ using the binomial theorem. The general term in the expansion of $(1+y)^m$ is $\binom{m}{k} y^k$. Here $y=x$ and $m=2n$.
$(1+x)^{2n} = \sum_{k=0}^{2n} \binom{2n}{k} x^k$
Substitute this expansion back into the expression for $E$:
$E = x^{-n} \sum_{k=0}^{2n} \binom{2n}{k} x^k$
Multiply $x^{-n}$ with each term in the sum:
$E = \sum_{k=0}^{2n} \binom{2n}{k} x^{-n} x^k$
Using the property $x^p x^q = x^{p+q}$:
$E = \sum_{k=0}^{2n} \binom{2n}{k} x^{k-n}$
We want to find the coefficient of $x^{-1}$ in this expansion. This means the power of $x$ must be equal to $-1$.
So, we set the exponent of $x$ equal to $-1$:
$k - n = -1$
Solve for $k$:
$k = n - 1$
Since $n$ is a positive integer ($n \in \mathbb{N}$), $n \geq 1$. The possible values for the index $k$ in the summation are integers from $0$ to $2n$. For $n \geq 1$, $k=n-1$ is an integer such that $0 \leq n-1 < 2n$, so $0 \leq k \leq 2n$ is satisfied.
The coefficient of $x^{-1}$ is the coefficient of the term where $k = n-1$. This coefficient is given by $\binom{2n}{k}$.
Coefficient of $x^{-1} = \binom{2n}{n-1}$
Using the symmetry property of binomial coefficients, $\binom{N}{K} = \binom{N}{N-K}$, we have:
$\binom{2n}{n-1} = \binom{2n}{2n - (n-1)} = \binom{2n}{2n - n + 1} = \binom{2n}{n+1}$
Both $\binom{2n}{n-1}$ and $\binom{2n}{n+1}$ are valid expressions for the coefficient.
The coefficient of $x^{-1}$ in the expansion of $(1 + x)^n \left( 1+\frac{1}{x} \right)^n$ is $\binom{2n}{n-1}$.
The final answer is $\boxed{\binom{2n}{n-1}}$.
Example 13: Which of the following is larger?
9950 + 10050 or 10150
Answer:
Given:
Quantities to compare: $99^{50} + 100^{50}$ and $101^{50}$.
To Determine:
Which quantity is larger.
Solution:
We can write $101^{50} = (100+1)^{50}$ and $99^{50} = (100-1)^{50}$.
Using the binomial theorem:
$(a+b)^n = \binom{n}{0}a^n + \binom{n}{1}a^{n-1}b + \binom{n}{2}a^{n-2}b^2 + \dots$
$(a-b)^n = \binom{n}{0}a^n - \binom{n}{1}a^{n-1}b + \binom{n}{2}a^{n-2}b^2 - \dots$
Let $a=100$, $b=1$, and $n=50$.
$101^{50} = (100+1)^{50} = \binom{50}{0}100^{50} + \binom{50}{1}100^{49} + \binom{50}{2}100^{48} + \binom{50}{3}100^{47} + \dots$
$99^{50} = (100-1)^{50} = \binom{50}{0}100^{50} - \binom{50}{1}100^{49} + \binom{50}{2}100^{48} - \binom{50}{3}100^{47} + \dots$
Consider the difference between $101^{50}$ and $99^{50}$:
$101^{50} - 99^{50} = [(100+1)^{50}] - [(100-1)^{50}]$
$101^{50} - 99^{50} = \left[ \binom{50}{0}100^{50} + \binom{50}{1}100^{49} + \binom{50}{2}100^{48} + \dots \right] - \left[ \binom{50}{0}100^{50} - \binom{50}{1}100^{49} + \binom{50}{2}100^{48} - \dots \right]$
$101^{50} - 99^{50} = 2 \left[ \binom{50}{1}100^{49} + \binom{50}{3}100^{47} + \binom{50}{5}100^{45} + \dots + \binom{50}{49}100^1 \right]$
The first term inside the bracket is $\binom{50}{1}100^{49} = 50 \cdot 100^{49}$.
$101^{50} - 99^{50} = 2 \cdot (50 \cdot 100^{49}) + 2 \left[ \binom{50}{3}100^{47} + \dots \right]$
$101^{50} - 99^{50} = 100 \cdot 100^{49} + 2 \left[ \binom{50}{3}100^{47} + \dots \right]$
$101^{50} - 99^{50} = 100^{50} + 2 \left[ \binom{50}{3}100^{47} + \binom{50}{5}100^{45} + \dots + \binom{50}{49}100 \right]$
The terms $\binom{50}{3}, \binom{50}{5}, \dots, \binom{50}{49}$ are all positive integers, and the powers of 100 are also positive. Thus, the quantity inside the square brackets is positive.
$2 \left[ \binom{50}{3}100^{47} + \dots \right] > 0$
So, $101^{50} - 99^{50} = 100^{50} + (\text{a positive value})$
This implies that $101^{50} - 99^{50} > 100^{50}$.
Adding $99^{50}$ to both sides of the inequality:
$101^{50} > 100^{50} + 99^{50}$
Therefore, $101^{50}$ is larger.
The final answer is $\boxed{101^{50}}$.
Example 14: Find the coefficient of x50 after simplifying and collecting the like terms in the expansion of (1 + x)1000 + x (1 + x)999 + x2 (1 + x)998 + ... + x1000.
Answer:
Given:
The expression: $(1 + x)^{1000} + x (1 + x)^{999} + x^2 (1 + x)^{998} + \dots + x^{1000}$.
To Find:
The coefficient of $x^{50}$ in the simplified expansion.
Solution:
The given expression is a finite geometric series:
$S = (1 + x)^{1000} + x (1 + x)^{999} + x^2 (1 + x)^{998} + \dots + x^{1000}$
The first term is $A = (1 + x)^{1000}$.
The terms can be written as $x^k (1+x)^{1000-k}$ for $k=0, 1, 2, \dots, 1000$.
Term 0: $x^0 (1+x)^{1000-0} = (1+x)^{1000}$
Term 1: $x^1 (1+x)^{1000-1} = x(1+x)^{999}$
... environments.
Term 1000: $x^{1000} (1+x)^{1000-1000} = x^{1000}$
The common ratio is $R = \frac{x (1+x)^{999}}{(1+x)^{1000}} = \frac{x}{1+x}$.
The number of terms is $N = 1000 - 0 + 1 = 1001$.
The sum of a finite geometric series is $S = A \frac{1 - R^N}{1 - R}$.
$S = (1+x)^{1000} \frac{1 - \left(\frac{x}{1+x}\right)^{1001}}{1 - \frac{x}{1+x}}$
Simplify the denominator:
$1 - \frac{x}{1+x} = \frac{(1+x) - x}{1+x} = \frac{1}{1+x}$
Simplify the numerator of the fraction:
$1 - \left(\frac{x}{1+x}\right)^{1001} = 1 - \frac{x^{1001}}{(1+x)^{1001}} = \frac{(1+x)^{1001} - x^{1001}}{(1+x)^{1001}}$
Substitute back into the sum formula:
$S = (1+x)^{1000} \frac{\frac{(1+x)^{1001} - x^{1001}}{(1+x)^{1001}}}{\frac{1}{1+x}}$
$S = (1+x)^{1000} \cdot \frac{(1+x)^{1001} - x^{1001}}{(1+x)^{1001}} \cdot (1+x)$
$S = (1+x)^{1000} \cdot \frac{(1+x)^{1001} - x^{1001}}{(1+x)^{1000}}$
$S = (1+x)^{1001} - x^{1001}$
We need to find the coefficient of $x^{50}$ in the expansion of $(1+x)^{1001} - x^{1001}$.
Consider the two terms separately:
1. $(1+x)^{1001}$:
The binomial expansion is $(1+x)^{1001} = \sum_{k=0}^{1001} \binom{1001}{k} x^k$. The coefficient of $x^{50}$ in this term is obtained when $k=50$, which is $\binom{1001}{50}$.
2. $-x^{1001}$:
This term is $-x^{1001}$. The power of $x$ here is 1001. We are looking for the coefficient of $x^{50}$. Since $50 \neq 1001$, the coefficient of $x^{50}$ in this term is 0.
The coefficient of $x^{50}$ in the sum is the sum of the coefficients of $x^{50}$ from each term:
Coefficient of $x^{50}$ in $S = (\text{Coefficient of } x^{50} \text{ in } (1+x)^{1001}) + (\text{Coefficient of } x^{50} \text{ in } -x^{1001})$
Coefficient of $x^{50} = \binom{1001}{50} + 0 = \binom{1001}{50}$.
The coefficient of $x^{50}$ in the simplified expansion is $\binom{1001}{50}$.
The final answer is $\boxed{\binom{1001}{50}}$.
Example 15: If a1, a2, a3 and a4 are the coefficient of any four consecutive terms in the expansion of (1 + x)n, prove that
$\frac{a_1}{a_1 + a_2} + \frac{a_3}{a_3 + a_4} = \frac{2a_2}{a_2 + a_3}$
Answer:
Given:
$a_1, a_2, a_3, a_4$ are the coefficients of four consecutive terms in the expansion of $(1+x)^n$, where $n$ is a positive integer.
To Prove:
$\frac{a_1}{a_1 + a_2} + \frac{a_3}{a_3 + a_4} = \frac{2a_2}{a_2 + a_3}$
Proof:
Let the four consecutive terms be the $(k+1)^{\text{th}}$, $(k+2)^{\text{th}}$, $(k+3)^{\text{th}}$, and $(k+4)^{\text{th}}$ terms in the expansion of $(1+x)^n$.
The general term in the expansion of $(1+x)^n$ is $T_{r+1} = \binom{n}{r} x^r$. The coefficient of the $(r+1)^{\text{th}}$ term is $\binom{n}{r}$.
So, the coefficients of the four consecutive terms are:
$a_1 = \binom{n}{k}$
$a_2 = \binom{n}{k+1}$
$a_3 = \binom{n}{k+2}$
$a_4 = \binom{n}{k+3}$
where $k$ is a non-negative integer such that $0 \leq k < k+1 < k+2 < k+3 \leq n$.
We use the property of binomial coefficients: $\frac{\binom{n}{r}}{\binom{n}{r-1}} = \frac{n-r+1}{r}$.
Consider the ratios of consecutive coefficients:
$\frac{a_2}{a_1} = \frac{\binom{n}{k+1}}{\binom{n}{k}} = \frac{n-(k+1)+1}{k+1} = \frac{n-k}{k+1}$
$\frac{a_3}{a_2} = \frac{\binom{n}{k+2}}{\binom{n}{k+1}} = \frac{n-(k+2)+1}{k+2} = \frac{n-k-1}{k+2}$
$\frac{a_4}{a_3} = \frac{\binom{n}{k+3}}{\binom{n}{k+2}} = \frac{n-(k+3)+1}{k+3} = \frac{n-k-2}{k+3}$
Now, let's simplify the terms in the given identity:
Consider the term $\frac{a_1}{a_1 + a_2}$:
$\frac{a_1}{a_1 + a_2} = \frac{a_1}{a_1(1 + a_2/a_1)} = \frac{1}{1 + a_2/a_1}$
Substitute the ratio $\frac{a_2}{a_1}$:
$\frac{a_1}{a_1 + a_2} = \frac{1}{1 + \frac{n-k}{k+1}} = \frac{1}{\frac{k+1 + n-k}{k+1}} = \frac{k+1}{n+1}$
Consider the term $\frac{a_3}{a_3 + a_4}$:
$\frac{a_3}{a_3 + a_4} = \frac{a_3}{a_3(1 + a_4/a_3)} = \frac{1}{1 + a_4/a_3}$
Substitute the ratio $\frac{a_4}{a_3}$:
$\frac{a_3}{a_3 + a_4} = \frac{1}{1 + \frac{n-k-2}{k+3}} = \frac{1}{\frac{k+3 + n-k-2}{k+3}} = \frac{k+3}{n+1}$
Consider the term $\frac{a_2}{a_2 + a_3}$:
$\frac{a_2}{a_2 + a_3} = \frac{a_2}{a_2(1 + a_3/a_2)} = \frac{1}{1 + a_3/a_2}$
Substitute the ratio $\frac{a_3}{a_2}$:
$\frac{a_2}{a_2 + a_3} = \frac{1}{1 + \frac{n-k-1}{k+2}} = \frac{1}{\frac{k+2 + n-k-1}{k+2}} = \frac{k+2}{n+1}$
Now, substitute these simplified terms into the left side (LHS) of the identity:
LHS $= \frac{a_1}{a_1 + a_2} + \frac{a_3}{a_3 + a_4}$
LHS $= \frac{k+1}{n+1} + \frac{k+3}{n+1}$
LHS $= \frac{(k+1) + (k+3)}{n+1}$
LHS $= \frac{2k + 4}{n+1}$
Now, consider the right side (RHS) of the identity:
RHS $= \frac{2a_2}{a_2 + a_3}$
RHS $= 2 \cdot \left( \frac{a_2}{a_2 + a_3} \right)$
RHS $= 2 \cdot \frac{k+2}{n+1}$
RHS $= \frac{2(k+2)}{n+1} = \frac{2k + 4}{n+1}$
Comparing the LHS and RHS, we have:
LHS $= \frac{2k + 4}{n+1}$
RHS $= \frac{2k + 4}{n+1}$
Since LHS = RHS, the identity is proven.
Thus, if $a_1, a_2, a_3$ and $a_4$ are the coefficients of any four consecutive terms in the expansion of $(1+x)^n$, then $\frac{a_1}{a_1 + a_2} + \frac{a_3}{a_3 + a_4} = \frac{2a_2}{a_2 + a_3}$.
Example 16 to 22 (Multiple Choice Questions)
Example 16: The total number of terms in the expansion of (x + a)51 – (x – a)51 after simplification is
(a) 102
(b) 25
(c) 26
(d) None of these
Answer:
Given:
The expression $(x + a)^{51} – (x – a)^{51}$.
To Find:
The total number of terms after simplification.
Solution:
Let the given expression be $E$.
$E = (x + a)^{51} – (x – a)^{51}$
We use the binomial expansions for $(A+B)^n$ and $(A-B)^n$:
$(A+B)^n = \binom{n}{0}A^n B^0 + \binom{n}{1}A^{n-1}B^1 + \binom{n}{2}A^{n-2}B^2 + \dots + \binom{n}{n}A^0 B^n$
$(A-B)^n = \binom{n}{0}A^n B^0 - \binom{n}{1}A^{n-1}B^1 + \binom{n}{2}A^{n-2}B^2 - \dots + (-1)^n \binom{n}{n}A^0 B^n$
Subtracting the second expansion from the first:
$(A+B)^n - (A-B)^n = 2 \left[ \binom{n}{1}A^{n-1}B^1 + \binom{n}{3}A^{n-3}B^3 + \binom{n}{5}A^{n-5}B^5 + \dots \right]$
The terms that remain are those with odd indices in the binomial coefficient, multiplied by 2.
In our case, $A=x$, $B=a$, and $n=51$. So the expression is:
$(x+a)^{51} - (x-a)^{51} = 2 \left[ \binom{51}{1}x^{51-1}a^1 + \binom{51}{3}x^{51-3}a^3 + \binom{51}{5}x^{51-5}a^5 + \dots + \binom{51}{51}x^{51-51}a^{51} \right]$
$(x+a)^{51} - (x-a)^{51} = 2 \left[ \binom{51}{1}x^{50}a + \binom{51}{3}x^{48}a^3 + \binom{51}{5}x^{46}a^5 + \dots + \binom{51}{51}a^{51} \right]$
The powers of $a$ in the remaining terms are $1, 3, 5, \dots, 51$. Since these powers of $a$ are all distinct odd numbers, the terms $x^{51-k}a^k$ for $k \in \{1, 3, 5, \dots, 51\}$ are distinct terms and cannot be combined further by collecting like terms (assuming $x \neq 0$ and $a \neq 0$).
The number of terms in the simplified expansion is equal to the number of odd integers from 1 to 51.
To find the number of odd integers from 1 to 51, we can use the formula for an arithmetic sequence: Number of terms = $\frac{\text{Last term} - \text{First term}}{\text{Common difference}} + 1$.
The sequence of odd numbers is $1, 3, 5, \dots, 51$.
Number of terms = $\frac{51 - 1}{2} + 1 = \frac{50}{2} + 1 = 25 + 1 = 26$.
Thus, there are 26 terms in the simplified expansion.
The total number of terms in the expansion of $(x + a)^{51} – (x – a)^{51}$ after simplification is 26.
The correct option is (c) 26.
Example 17: If the coefficients of x7 and x8 in $\left( 2 + \frac{x}{3} \right)^n$ are equal, then n is
(a) 56
(b) 55
(c) 45
(d) 15
Answer:
Given:
The expansion is $\left( 2 + \frac{x}{3} \right)^n$.
The coefficients of $x^7$ and $x^8$ in the expansion are equal.
To Find:
The value of $n$.
Solution:
The given expansion is of the form $(a+b)^n$, where $a=2$, $b=\frac{x}{3}$.
The general term, $T_{k+1}$, in the expansion of $(a+b)^n$ is given by $T_{k+1} = \binom{n}{k} a^{n-k} b^k$.
Substituting the values $a=2$ and $b=\frac{x}{3}$:
$T_{k+1} = \binom{n}{k} (2)^{n-k} \left(\frac{x}{3}\right)^k$
$T_{k+1} = \binom{n}{k} 2^{n-k} \frac{x^k}{3^k}$
$T_{k+1} = \binom{n}{k} 2^{n-k} 3^{-k} x^k$
The coefficient of $x^k$ in the expansion is $\binom{n}{k} 2^{n-k} 3^{-k}$.
The coefficient of $x^7$ is obtained by setting $k=7$:
Coefficient of $x^7 = \binom{n}{7} 2^{n-7} 3^{-7}$
The coefficient of $x^8$ is obtained by setting $k=8$:
Coefficient of $x^8 = \binom{n}{8} 2^{n-8} 3^{-8}$
According to the problem statement, the coefficients of $x^7$ and $x^8$ are equal:
$\binom{n}{7} 2^{n-7} 3^{-7} = \binom{n}{8} 2^{n-8} 3^{-8}$
Rewrite the binomial coefficients and powers:
$\frac{n!}{7!(n-7)!} \cdot 2^{n-7} \cdot \frac{1}{3^7} = \frac{n!}{8!(n-8)!} \cdot 2^{n-8} \cdot \frac{1}{3^8}$
Expand factorials $8! = 8 \cdot 7!$ and $(n-7)! = (n-7)(n-8)!$ (assuming $n \ge 8$):
$\frac{n!}{7!(n-7)(n-8)!} \cdot 2^{n-7} \cdot \frac{1}{3^7} = \frac{n!}{8 \cdot 7!(n-8)!} \cdot 2^{n-8} \cdot \frac{1}{3^8}$
Cancel common terms $n!$, $7!$, $(n-8)!$ from both sides:
$\frac{1}{n-7} \cdot 2^{n-7} \cdot \frac{1}{3^7} = \frac{1}{8} \cdot 2^{n-8} \cdot \frac{1}{3^8}$
Rearrange terms:
$\frac{2^{n-7}}{3^7 (n-7)} = \frac{2^{n-8}}{3^8 \cdot 8}$
Divide both sides by $2^{n-8}$ and multiply by $3^8$:
$\frac{2^{n-7}}{2^{n-8}} \cdot \frac{3^8}{3^7} = \frac{n-7}{8}$
$2^{n-7 - (n-8)} \cdot 3^{8-7} = \frac{n-7}{8}$
$2^1 \cdot 3^1 = \frac{n-7}{8}$
$6 = \frac{n-7}{8}$
Multiply both sides by 8:
$6 \times 8 = n-7$
$48 = n-7$
$n = 48 + 7$
$n = 55$
The value of $n=55$ satisfies the condition $n \ge 8$ required for $\binom{n}{8}$ to be defined in this context.
The value of $n$ is 55.
The correct option is (b) 55.
Example 18: If (1 – x + x2)n = a0 + a1x + a2x2 + ... + a2n x2n , then a0 + a2 + a4 + ...+ a2n equals.
(A) $\frac{3^n + 1}{2}$
(B) $\frac{3^n − 1}{2}$
(C) $\frac{1 − 3^n}{2}$
(D) $3n + \frac{1}{2}$
Answer:
Given:
The expansion $(1 – x + x^2)^n = a_0 + a_1x + a_2x^2 + \dots + a_{2n} x^{2n}$.
To Find:
The value of $a_0 + a_2 + a_4 + \dots + a_{2n}$.
Solution:
We are given the polynomial identity:
$(1 – x + x^2)^n = a_0 + a_1x + a_2x^2 + a_3x^3 + \dots + a_{2n-1}x^{2n-1} + a_{2n} x^{2n}$
To find the sum of coefficients with even indices ($a_0 + a_2 + a_4 + \dots$), we can use properties of polynomial evaluations.
Substitute $x=1$ into the identity:
$(1 – 1 + 1^2)^n = a_0 + a_1(1) + a_2(1)^2 + a_3(1)^3 + \dots + a_{2n}(1)^{2n}$
$(1 – 1 + 1)^n = a_0 + a_1 + a_2 + a_3 + \dots + a_{2n}$
$1^n = a_0 + a_1 + a_2 + a_3 + \dots + a_{2n}$
$a_0 + a_1 + a_2 + a_3 + \dots + a_{2n} = 1$
…(i)
Substitute $x=-1$ into the identity:
$(1 – (-1) + (-1)^2)^n = a_0 + a_1(-1) + a_2(-1)^2 + a_3(-1)^3 + \dots + a_{2n}(-1)^{2n}$
$(1 + 1 + 1)^n = a_0 - a_1 + a_2 - a_3 + \dots + a_{2n-1}(-1)^{2n-1} + a_{2n}(-1)^{2n}$
Since $2n$ is even, $(-1)^{2n} = 1$. Since $2n-1$ is odd, $(-1)^{2n-1} = -1$. In general, $(-1)^k$ is 1 if $k$ is even and -1 if $k$ is odd.
$3^n = a_0 - a_1 + a_2 - a_3 + \dots - a_{2n-1} + a_{2n}$
$a_0 - a_1 + a_2 - a_3 + \dots + a_{2n} = 3^n$
…(ii)
We want to find $a_0 + a_2 + a_4 + \dots + a_{2n}$, which is the sum of coefficients with even indices.
Add equation (i) and equation (ii):
$(a_0 + a_1 + a_2 + a_3 + \dots + a_{2n}) + (a_0 - a_1 + a_2 - a_3 + \dots + a_{2n}) = 1 + 3^n$
Combine like terms. The coefficients with odd indices ($a_1, a_3, \dots$) cancel out:
$(a_0 + a_0) + (a_1 - a_1) + (a_2 + a_2) + (a_3 - a_3) + \dots + (a_{2n} + a_{2n}) = 1 + 3^n$
$2a_0 + 0 + 2a_2 + 0 + \dots + 2a_{2n} = 1 + 3^n$
$2(a_0 + a_2 + a_4 + \dots + a_{2n}) = 1 + 3^n$
Divide by 2:
$a_0 + a_2 + a_4 + \dots + a_{2n} = \frac{1 + 3^n}{2}$
The value of $a_0 + a_2 + a_4 + \dots + a_{2n}$ is $\frac{3^n + 1}{2}$.
The correct option is (A) $\frac{3^n + 1}{2}$.
Example 19: The coefficient of xp and xq (p and q are positive integers) in the expansion of (1 + x)p + q are
(A) equal
(B) equal with opposite signs
(C) reciprocal of each other
(D) none of these
Answer:
Given:
The expansion is $(1 + x)^{p + q}$, where $p$ and $q$ are positive integers.
To Find:
The relationship between the coefficient of $x^p$ and $x^q$ in the expansion.
Solution:
The binomial expansion of $(1+x)^n$ is given by:
$(1+x)^n = \sum_{k=0}^n \binom{n}{k} x^k$
The coefficient of $x^k$ in the expansion of $(1+x)^n$ is $\binom{n}{k}$.
In the given expansion $(1 + x)^{p + q}$, the value of $n$ is $p+q$.
The coefficient of $x^p$ is obtained by setting $k=p$. The coefficient is $\binom{p+q}{p}$.
The coefficient of $x^q$ is obtained by setting $k=q$. The coefficient is $\binom{p+q}{q}$.
We need to compare $\binom{p+q}{p}$ and $\binom{p+q}{q}$.
Using the symmetry property of binomial coefficients, $\binom{N}{K} = \binom{N}{N-K}$, where $N$ and $K$ are non-negative integers with $0 \leq K \leq N$.
In our case, let $N = p+q$ and $K = p$. Since $p$ and $q$ are positive integers, $p+q$ is a positive integer, and $0 \leq p \leq p+q$ is true.
Applying the symmetry property:
$\binom{p+q}{p} = \binom{p+q}{(p+q) - p}$
$\binom{p+q}{p} = \binom{p+q}{q}$
This shows that the coefficient of $x^p$ is equal to the coefficient of $x^q$.
The coefficient of $x^p$ and $x^q$ in the expansion of $(1 + x)^{p + q}$ are equal.
The correct option is (A) equal.
Example 20: The number of terms in the expansion of (a + b + c)n, where n ∈ N is
(A) $\frac{(n+1) (n+2)}{2}$
(B) n + 1
(C) n + 2
(D) (n + 1) n
Answer:
Given:
The expression $(a + b + c)^n$, where $n \in \mathbb{N}$.
To Find:
The number of terms in the expansion.
Solution:
The expansion of $(a_1 + a_2 + \dots + a_m)^n$ is given by the multinomial theorem:
$\sum_{n_1+n_2+\dots+n_m=n} \frac{n!}{n_1! n_2! \dots n_m!} a_1^{n_1} a_2^{n_2} \dots a_m^{n_m}$
Each term in the expansion corresponds to a unique combination of non-negative integers $(n_1, n_2, \dots, n_m)$ such that $n_1 + n_2 + \dots + n_m = n$.
In this case, we have a trinomial $(a+b+c)^n$, so $m=3$. The terms are of the form $C \cdot a^{n_1} b^{n_2} c^{n_3}$, where $C$ is the coefficient and $n_1, n_2, n_3$ are non-negative integers such that $n_1 + n_2 + n_3 = n$.
The number of distinct terms in the expansion is equal to the number of distinct non-negative integer solutions to the equation $n_1 + n_2 + n_3 = n$.
This is a classic combinatorial problem that can be solved using stars and bars. We have $n$ "stars" to distribute among 3 "bins" (corresponding to the powers $n_1, n_2, n_3$). We need $m-1 = 3-1 = 2$ "bars" to separate the bins.
The number of ways to arrange $n$ stars and 2 bars is the number of ways to choose the positions of the 2 bars (or the $n$ stars) in a sequence of $n+2$ positions.
The number of terms is $\binom{n + m - 1}{m - 1}$ or $\binom{n + m - 1}{n}$.
Using $m=3$, the number of terms is $\binom{n + 3 - 1}{3 - 1} = \binom{n+2}{2}$.
Calculate the binomial coefficient:
$\binom{n+2}{2} = \frac{(n+2)!}{2!(n+2-2)!} = \frac{(n+2)!}{2!n!} = \frac{(n+2)(n+1)\cancel{n!}}{2 \times 1 \times \cancel{n!}} = \frac{(n+2)(n+1)}{2}$
The number of terms in the expansion of $(a + b + c)^n$ is $\frac{(n+1)(n+2)}{2}$.
The correct option is (A) $\frac{(n+1) (n+2)}{2}$.
Example 21: The ratio of the coefficient of x15 to the term independent of x in $\left( x^2 + \frac{2}{x} \right)^{15}$ is
(A) 12:32
(B) 1:32
(C) 32:12
(D) 32:1
Answer:
Given:
The expansion is $\left( x^2 + \frac{2}{x} \right)^{15}$.
To Find:
The ratio of the coefficient of $x^{15}$ to the term independent of $x$.
Solution:
The given expansion is of the form $(a+b)^n$, where $a=x^2$, $b=\frac{2}{x}$, and $n=15$.
The general term, $T_{k+1}$, in the expansion is given by $T_{k+1} = \binom{n}{k} a^{n-k} b^k$.
Substituting the values $a=x^2$, $b=\frac{2}{x}$, and $n=15$:
$T_{k+1} = \binom{15}{k} (x^2)^{15-k} \left(\frac{2}{x}\right)^k$
Simplify the terms involving $x$:
$(x^2)^{15-k} = x^{2(15-k)} = x^{30-2k}$
$\left(\frac{2}{x}\right)^k = \frac{2^k}{x^k} = 2^k x^{-k}$
The variable part of the general term is $x^{30-2k} \cdot x^{-k} = x^{30-2k-k} = x^{30-3k}$.
The general term is $T_{k+1} = \binom{15}{k} 2^k x^{30-3k}$.
Coefficient of $x^{15}$:
For the coefficient of $x^{15}$, we set the power of $x$ equal to 15:
$30 - 3k = 15$
$30 - 15 = 3k$
$15 = 3k$
$k = 5$
The coefficient of $x^{15}$ is the constant part of the general term when $k=5$:
Coefficient of $x^{15} = \binom{15}{5} 2^5$
$\binom{15}{5} = \frac{15!}{5!10!} = \frac{15 \times 14 \times 13 \times 12 \times 11}{5 \times 4 \times 3 \times 2 \times 1} = \frac{\cancel{15}^{1} \times \cancel{14}^{7} \times 13 \times \cancel{12}^{1} \times 11}{\cancel{5}_{1} \times \cancel{4}_{1} \times \cancel{3}_{1} \times \cancel{2}_{1} \times 1} = 1 \times 7 \times 13 \times 1 \times 11 = 3003$
$2^5 = 32$
Coefficient of $x^{15} = 3003 \times 32 = 96096$.
Term independent of $x$:
The term independent of $x$ occurs when the power of $x$ is 0:
$30 - 3k = 0$
$30 = 3k$
$k = 10$
The term independent of $x$ is the value of the general term when $k=10$ (without the $x^0$):
Term independent of $x = \binom{15}{10} 2^{10}$
$\binom{15}{10} = \binom{15}{15-10} = \binom{15}{5} = 3003$ (as calculated before)
$2^{10} = 1024$
Term independent of $x = 3003 \times 1024 = 3075072$.
Ratio:
We need the ratio of the coefficient of $x^{15}$ to the term independent of $x$.
Ratio $= \frac{\text{Coefficient of } x^{15}}{\text{Term independent of } x} = \frac{\binom{15}{5} 2^5}{\binom{15}{10} 2^{10}}$
Since $\binom{15}{5} = \binom{15}{10}$, these terms cancel out:
Ratio $= \frac{2^5}{2^{10}} = 2^{5-10} = 2^{-5} = \frac{1}{2^5} = \frac{1}{32}$
The ratio is $1:32$.
The ratio of the coefficient of $x^{15}$ to the term independent of $x$ is $1:32$.
The correct option is (B) 1:32.
Example 22: If $z = \left( \frac{\sqrt{3}}{2} + \frac{i}{2} \right)^5 + \left( \frac{\sqrt{3}}{2} - \frac{i}{2} \right)^5$, then
(A) Re (z) = 0
(B) Im (z) = 0
(C) Re (z) > 0, Im (z) > 0
(D) Re (z) > 0, Im (z) < 0
Answer:
Given:
$z = \left( \frac{\sqrt{3}}{2} + \frac{i}{2} \right)^5 + \left( \frac{\sqrt{3}}{2} - \frac{i}{2} \right)^5$.
To Find:
The properties of the complex number $z$, specifically its real and imaginary parts.
Solution:
Let $a = \frac{\sqrt{3}}{2}$ and $b = \frac{1}{2}$. The expression for $z$ is in the form $(a+ib)^5 + (a-ib)^5$.
We can use the binomial theorem to expand each term. Let $n=5$.
$(a+ib)^n = \binom{n}{0}a^n(ib)^0 + \binom{n}{1}a^{n-1}(ib)^1 + \binom{n}{2}a^{n-2}(ib)^2 + \binom{n}{3}a^{n-3}(ib)^3 + \dots + \binom{n}{n}a^0(ib)^n$
$(a-ib)^n = \binom{n}{0}a^n(-ib)^0 + \binom{n}{1}a^{n-1}(-ib)^1 + \binom{n}{2}a^{n-2}(-ib)^2 + \binom{n}{3}a^{n-3}(-ib)^3 + \dots + \binom{n}{n}a^0(-ib)^n$
Using the property $(-ib)^k = (-1)^k (ib)^k$.
Summing the two expansions:
$(a+ib)^n + (a-ib)^n = \sum_{k=0}^n \binom{n}{k} a^{n-k} (ib)^k + \sum_{k=0}^n \binom{n}{k} a^{n-k} (-1)^k (ib)^k$
$= \sum_{k=0}^n \binom{n}{k} a^{n-k} (ib)^k (1 + (-1)^k)$
The term $(1 + (-1)^k)$ is 2 if $k$ is even, and 0 if $k$ is odd.
So, only the terms where $k$ is even will contribute to the sum.
For $n=5$, the possible values of $k$ are $0, 1, 2, 3, 4, 5$. The even values of $k$ are $0, 2, 4$.
$z = (a+ib)^5 + (a-ib)^5 = 2 \left[ \binom{5}{0}a^5(ib)^0 + \binom{5}{2}a^3(ib)^2 + \binom{5}{4}a^1(ib)^4 \right]$
Substitute $a=\frac{\sqrt{3}}{2}$ and $b=\frac{1}{2}$, and powers of $i$: $i^0=1, i^2=-1, i^4=1$.
$z = 2 \left[ \binom{5}{0}\left(\frac{\sqrt{3}}{2}\right)^5(1) + \binom{5}{2}\left(\frac{\sqrt{3}}{2}\right)^3(i^2 (\frac{1}{2})^2) + \binom{5}{4}\left(\frac{\sqrt{3}}{2}\right)^1(i^4 (\frac{1}{2})^4) \right]$
$z = 2 \left[ \binom{5}{0}\left(\frac{\sqrt{3}}{2}\right)^5 + \binom{5}{2}\left(\frac{\sqrt{3}}{2}\right)^3(-1 \cdot \frac{1}{4}) + \binom{5}{4}\left(\frac{\sqrt{3}}{2}\right)\left(1 \cdot \frac{1}{16}\right) \right]$
$z = 2 \left[ \binom{5}{0}\left(\frac{\sqrt{3}}{2}\right)^5 - \frac{1}{4}\binom{5}{2}\left(\frac{\sqrt{3}}{2}\right)^3 + \frac{1}{16}\binom{5}{4}\left(\frac{\sqrt{3}}{2}\right) \right]$
Calculate the binomial coefficients:
$\binom{5}{0} = 1$
$\binom{5}{2} = \frac{5 \times 4}{2 \times 1} = 10$
$\binom{5}{4} = \binom{5}{1} = 5$
Calculate the powers of $\frac{\sqrt{3}}{2}$:
$\left(\frac{\sqrt{3}}{2}\right)^1 = \frac{\sqrt{3}}{2}$
$\left(\frac{\sqrt{3}}{2}\right)^3 = \frac{(\sqrt{3})^3}{2^3} = \frac{3\sqrt{3}}{8}$
$\left(\frac{\sqrt{3}}{2}\right)^5 = \frac{(\sqrt{3})^5}{2^5} = \frac{9\sqrt{3}}{32}$
Substitute these values into the expression for $z$:
$z = 2 \left[ 1 \cdot \frac{9\sqrt{3}}{32} - \frac{1}{4} \cdot 10 \cdot \frac{3\sqrt{3}}{8} + \frac{1}{16} \cdot 5 \cdot \frac{\sqrt{3}}{2} \right]$
$z = 2 \left[ \frac{9\sqrt{3}}{32} - \frac{10 \cdot 3\sqrt{3}}{32} + \frac{5\sqrt{3}}{32} \right]$
$z = 2 \left[ \frac{9\sqrt{3} - 30\sqrt{3} + 5\sqrt{3}}{32} \right]$
$z = 2 \left[ \frac{(9 - 30 + 5)\sqrt{3}}{32} \right]$
$z = 2 \left[ \frac{-16\sqrt{3}}{32} \right]$
$z = 2 \left[ -\frac{\cancel{16}\sqrt{3}}{\cancel{32}_{2}} \right]$
$z = 2 \left[ -\frac{\sqrt{3}}{2} \right]$
$z = -\sqrt{3}$
The complex number $z = -\sqrt{3}$.
The real part of $z$ is $\text{Re}(z) = -\sqrt{3}$.
The imaginary part of $z$ is $\text{Im}(z) = 0$.
Since $\sqrt{3} \approx 1.732$, $\text{Re}(z) = -\sqrt{3} < 0$.
$\text{Im}(z) = 0$.
Based on the options:
(A) $\text{Re}(z) = 0$ (False)
(B) $\text{Im}(z) = 0$ (True)
(C) $\text{Re}(z) > 0$, $\text{Im}(z) > 0$ (False)
(D) $\text{Re}(z) > 0$, $\text{Im}(z) < 0$ (False)
The imaginary part of $z$ is 0.
The correct option is (B) Im (z) = 0.
Alternate Solution using Polar Form:
The complex number $\frac{\sqrt{3}}{2} + \frac{i}{2}$ can be written in polar form $r(\cos \theta + i \sin \theta)$.
$r = \left| \frac{\sqrt{3}}{2} + \frac{i}{2} \right| = \sqrt{\left(\frac{\sqrt{3}}{2}\right)^2 + \left(\frac{1}{2}\right)^2} = \sqrt{\frac{3}{4} + \frac{1}{4}} = \sqrt{\frac{4}{4}} = \sqrt{1} = 1$
$\cos \theta = \frac{\sqrt{3}}{2}$ and $\sin \theta = \frac{1}{2}$. This means $\theta = \frac{\pi}{6}$ (or $30^\circ$).
So, $\frac{\sqrt{3}}{2} + \frac{i}{2} = 1 \cdot (\cos(\frac{\pi}{6}) + i \sin(\frac{\pi}{6})) = e^{i \pi/6}$.
The complex conjugate is $\frac{\sqrt{3}}{2} - \frac{i}{2} = \cos(\frac{\pi}{6}) - i \sin(\frac{\pi}{6}) = \cos(-\frac{\pi}{6}) + i \sin(-\frac{\pi}{6}) = e^{-i \pi/6}$.
Using De Moivre's Theorem, $(r(\cos \theta + i \sin \theta))^n = r^n(\cos(n\theta) + i \sin(n\theta))$.
$\left( \frac{\sqrt{3}}{2} + \frac{i}{2} \right)^5 = (e^{i \pi/6})^5 = e^{i 5\pi/6} = \cos(\frac{5\pi}{6}) + i \sin(\frac{5\pi}{6})$
$\left( \frac{\sqrt{3}}{2} - \frac{i}{2} \right)^5 = (e^{-i \pi/6})^5 = e^{-i 5\pi/6} = \cos(-\frac{5\pi}{6}) + i \sin(-\frac{5\pi}{6})$
Using $\cos(-x) = \cos(x)$ and $\sin(-x) = -\sin(x)$:
$\left( \frac{\sqrt{3}}{2} - \frac{i}{2} \right)^5 = \cos(\frac{5\pi}{6}) - i \sin(\frac{5\pi}{6})$
Now, sum the two terms to find $z$:
$z = \left( \cos(\frac{5\pi}{6}) + i \sin(\frac{5\pi}{6}) \right) + \left( \cos(\frac{5\pi}{6}) - i \sin(\frac{5\pi}{6}) \right)$
$z = 2 \cos(\frac{5\pi}{6})$
The value of $\cos(\frac{5\pi}{6}) = \cos(\pi - \frac{\pi}{6}) = -\cos(\frac{\pi}{6}) = -\frac{\sqrt{3}}{2}$.
$z = 2 \left(-\frac{\sqrt{3}}{2}\right) = -\sqrt{3}$
So, $z = -\sqrt{3} + 0i$.
$\text{Re}(z) = -\sqrt{3}$ and $\text{Im}(z) = 0$.
This confirms that the imaginary part of $z$ is 0.
The correct option is (B) Im (z) = 0.
Exercise
Question 1 to 11 (Short Answer Type Questions)
Question 1. Find the term independent of x, x ≠ 0, in the expansion of $\left( \frac{3x^2}{2} − \frac{1}{3x} \right)^{15}$.
Answer:
Given:
The binomial expansion is $\left( \frac{3x^2}{2} − \frac{1}{3x} \right)^{15}$, with $x \neq 0$.
To Find:
The term independent of $x$ in the expansion.
Solution:
The given expansion is of the form $(a+b)^n$, where $a = \frac{3x^2}{2}$, $b = -\frac{1}{3x}$, and $n = 15$.
The general term, $T_{k+1}$, in the expansion of $(a+b)^n$ is given by the formula:
$T_{k+1} = \binom{n}{k} a^{n-k} b^k$
Substitute the values of $n$, $a$, and $b$ for the given expansion:
$T_{k+1} = \binom{15}{k} \left(\frac{3x^2}{2}\right)^{15-k} \left(-\frac{1}{3x}\right)^k$
Now, let's separate the constant terms and the terms involving $x$.
Constant part: $\binom{15}{k} \left(\frac{3}{2}\right)^{15-k} \left(-\frac{1}{3}\right)^k$
Variable part: $(x^2)^{15-k} \cdot \left(\frac{1}{x}\right)^k = x^{2(15-k)} \cdot x^{-k} = x^{30-2k} \cdot x^{-k} = x^{30-3k}$
The general term is $T_{k+1} = \binom{15}{k} \left(\frac{3}{2}\right)^{15-k} \left(-\frac{1}{3}\right)^k x^{30-3k}$.
For the term independent of $x$, the power of $x$ must be zero. So, we set the exponent of $x$ equal to 0:
$30 - 3k = 0$
Solve for $k$:
$30 = 3k$
$k = 10$
Since $k=10$ is a non-negative integer and $0 \leq 10 \leq 15$, this value of $k$ is valid.
The term independent of $x$ is the constant part of $T_{k+1}$ when $k=10$.
Term independent of $x = \binom{15}{10} \left(\frac{3}{2}\right)^{15-10} \left(-\frac{1}{3}\right)^{10}$
Term independent of $x = \binom{15}{10} \left(\frac{3}{2}\right)^5 \left(-\frac{1}{3}\right)^{10}$
Calculate the binomial coefficient $\binom{15}{10}$:
$\binom{15}{10} = \binom{15}{15-10} = \binom{15}{5} = \frac{15!}{5!10!} = \frac{15 \times 14 \times 13 \times 12 \times 11}{5 \times 4 \times 3 \times 2 \times 1} = 3003$
Calculate the powers of the fractions:
$\left(\frac{3}{2}\right)^5 = \frac{3^5}{2^5} = \frac{243}{32}$
$\left(-\frac{1}{3}\right)^{10} = (-1)^{10} \frac{1^{10}}{3^{10}} = 1 \cdot \frac{1}{3^{10}} = \frac{1}{59049}$
Substitute these values back into the expression for the term independent of $x$:
Term independent of $x = 3003 \cdot \frac{243}{32} \cdot \frac{1}{59049}$
Simplify the numerical part. Note that $59049 = 3^{10}$ and $243 = 3^5$.
Term independent of $x = 3003 \cdot \frac{3^5}{32} \cdot \frac{1}{3^{10}}$
Term independent of $x = 3003 \cdot \frac{1}{32} \cdot \frac{3^5}{3^{10}}$
Term independent of $x = 3003 \cdot \frac{1}{32} \cdot 3^{5-10}$
Term independent of $x = 3003 \cdot \frac{1}{32} \cdot 3^{-5}$
Term independent of $x = 3003 \cdot \frac{1}{32} \cdot \frac{1}{3^5}$
Term independent of $x = 3003 \cdot \frac{1}{32} \cdot \frac{1}{243}$
Term independent of $x = \frac{3003}{32 \times 243}$
Term independent of $x = \frac{3003}{7776}$
Let's check if there are common factors. The sum of digits of 3003 is $3+0+0+3=6$, which is divisible by 3. So 3003 is divisible by 3. $3003 / 3 = 1001$.
The sum of digits of 7776 is $7+7+7+6=27$, which is divisible by 9 (and 3). So 7776 is divisible by 3. $7776 / 3 = 2592$.
$\frac{3003}{7776} = \frac{1001}{2592}$
Let's express $\binom{15}{10} \frac{3^5}{2^5} \frac{1}{3^{10}}$ as $\frac{15!}{10!5!} \frac{3^5}{2^5 3^{10}} = \frac{15!}{10!5!} \frac{1}{2^5 3^5}$.
$\frac{15 \times 14 \times 13 \times 12 \times 11}{5 \times 4 \times 3 \times 2 \times 1} \times \frac{1}{32 \times 243} = 3003 \times \frac{1}{32 \times 243} = \frac{3003}{7776}$.
Is 1001 divisible by any factors of 2592 other than 1? $2592 = 2^5 \times 3^4$. 1001 is not divisible by 2 or 3. So the fraction is $\frac{1001}{2592}$.
The term independent of $x$ is $\frac{1001}{2592}$.
The final answer is $\boxed{\frac{1001}{2592}}$.
Question 2. If the term free from x in the expansion of $\left( \sqrt{x} - \frac{k}{x^2} \right)^{10}$ is 405, find the value of k.
Answer:
Given:
The expansion is $\left( \sqrt{x} - \frac{k}{x^2} \right)^{10}$.
The term independent of $x$ is 405.
To Find:
The value of $k$.
Solution:
The given expansion is of the form $(a+b)^n$, where $a = \sqrt{x} = x^{1/2}$, $b = -\frac{k}{x^2} = -k x^{-2}$, and $n = 10$.
The general term, $T_{r+1}$, in the expansion is given by the formula:
$T_{r+1} = \binom{n}{r} a^{n-r} b^r$
Substitute the values of $n$, $a$, and $b$ for the given expansion:
$T_{r+1} = \binom{10}{r} (x^{1/2})^{10-r} (-k x^{-2})^r$
Simplify the terms involving $x$ and the constant terms separately.
Constant part: $\binom{10}{r} (-k)^r = \binom{10}{r} (-1)^r k^r$
Variable part: $(x^{1/2})^{10-r} \cdot (x^{-2})^r = x^{(1/2)(10-r)} \cdot x^{-2r} = x^{\frac{10-r}{2}} \cdot x^{-2r} = x^{\frac{10-r}{2} - 2r} = x^{\frac{10-r-4r}{2}} = x^{\frac{10-5r}{2}}$
The general term is $T_{r+1} = \binom{10}{r} (-1)^r k^r x^{\frac{10-5r}{2}}$.
For the term independent of $x$, the power of $x$ must be zero. So, we set the exponent of $x$ equal to 0:
$\frac{10-5r}{2} = 0$
$10 - 5r = 0$
$10 = 5r$
$r = 2$
Since $r=2$ is a non-negative integer and $0 \leq 2 \leq 10$, this value of $r$ is valid.
The term independent of $x$ is the constant part of $T_{r+1}$ when $r=2$.
Term independent of $x = \binom{10}{2} (-1)^2 k^2$
We are given that this term is equal to 405.
$\binom{10}{2} (-1)^2 k^2 = 405$
Calculate the binomial coefficient $\binom{10}{2}$:
$\binom{10}{2} = \frac{10!}{2!8!} = \frac{10 \times 9}{2 \times 1} = 45$
$(-1)^2 = 1$
Substitute these values into the equation:
$45 \cdot 1 \cdot k^2 = 405$
$45k^2 = 405$
Divide both sides by 45:
$k^2 = \frac{405}{45}$
Divide 405 by 45. $405 = 9 \times 45$.
$k^2 = \frac{9 \times \cancel{45}}{\cancel{45}} = 9$
Take the square root of both sides:
$k = \pm \sqrt{9}$
$k = \pm 3$
The possible values for $k$ are 3 and -3.
The final answer is $\boxed{\pm 3}$.
Question 3. Find the coefficient of x in the expansion of (1 – 3x + 7x2) (1 – x)16.
Answer:
Given:
The expression $(1 – 3x + 7x^2) (1 – x)^{16}$.
To Find:
The coefficient of $x$ in the expansion of the given expression.
Solution:
Let the given expression be $E$.
$E = (1 – 3x + 7x^2) (1 – x)^{16}$
We need to find the terms that produce $x^1$ when the two factors are multiplied.
The second factor is $(1-x)^{16}$. The binomial expansion of $(1+y)^n = \sum_{k=0}^n \binom{n}{k} y^k$.
Here $y=-x$ and $n=16$.
$(1-x)^{16} = \binom{16}{0}(1)^{16}(-x)^0 + \binom{16}{1}(1)^{15}(-x)^1 + \binom{16}{2}(1)^{14}(-x)^2 + \dots$
$(1-x)^{16} = \binom{16}{0} - \binom{16}{1}x + \binom{16}{2}x^2 - \binom{16}{3}x^3 + \dots$
$(1-x)^{16} = 1 - 16x + \binom{16}{2}x^2 - \dots$
The first factor is $(1 – 3x + 7x^2)$.
We multiply the two factors:
$E = (1 – 3x + 7x^2) (1 - 16x + \binom{16}{2}x^2 - \dots)$
To get a term with $x^1$, we multiply terms from the first factor with terms from the second factor such that the powers of $x$ add up to 1.
The possible combinations are:
1. Constant term from the first factor multiplied by the $x^1$ term from the second factor.
$(1) \cdot (-\binom{16}{1}x) = -16x$
2. $x^1$ term from the first factor multiplied by the constant term from the second factor.
$(-3x) \cdot (\binom{16}{0}) = -3x \cdot 1 = -3x$
The terms with $x^2$ or higher powers from the first factor, when multiplied by any term from the second factor up to the $x^0$ term, will result in a power of $x$ greater than or equal to 2. For example, $(7x^2) \cdot (1) = 7x^2$.
Therefore, the terms containing $x^1$ in the expansion are $-16x$ and $-3x$.
The coefficient of $x$ is the sum of the coefficients of these terms.
Coefficient of $x = -16 + (-3) = -16 - 3 = -19$.
The coefficient of $x$ in the expansion of $(1 – 3x + 7x^2) (1 – x)^{16}$ is $-19$.
The final answer is $\boxed{-19}$.
Question 4. Find the term independent of x in the expansion of, $\left( 3x - \frac{2}{x^2} \right)^{15}$ .
Answer:
Given:
The binomial expansion is $\left( 3x - \frac{2}{x^2} \right)^{15}$.
To Find:
The term independent of $x$ in the expansion.
Solution:
The given expansion is of the form $(a+b)^n$, where $a = 3x$, $b = -\frac{2}{x^2}$, and $n = 15$.
The general term, $T_{k+1}$, in the expansion of $(a+b)^n$ is given by the formula:
$T_{k+1} = \binom{n}{k} a^{n-k} b^k$
Substitute the values of $n$, $a$, and $b$ for the given expansion:
$T_{k+1} = \binom{15}{k} (3x)^{15-k} \left(-\frac{2}{x^2}\right)^k$
Now, let's separate the constant terms and the terms involving $x$.
Constant part: $\binom{15}{k} 3^{15-k} (-2)^k$
Variable part: $(x)^{15-k} \cdot \left(\frac{1}{x^2}\right)^k = x^{15-k} \cdot (x^{-2})^k = x^{15-k} \cdot x^{-2k} = x^{15-k-2k} = x^{15-3k}$
The general term is $T_{k+1} = \binom{15}{k} 3^{15-k} (-2)^k x^{15-3k}$.
For the term independent of $x$, the power of $x$ must be zero. So, we set the exponent of $x$ equal to 0:
$15 - 3k = 0$
Solve for $k$:
$15 = 3k$
$k = 5$
Since $k=5$ is a non-negative integer and $0 \leq 5 \leq 15$, this value of $k$ is valid.
The term independent of $x$ is the constant part of $T_{k+1}$ when $k=5$.
Term independent of $x = \binom{15}{5} 3^{15-5} (-2)^5$
Term independent of $x = \binom{15}{5} 3^{10} (-2)^5$
Calculate the binomial coefficient $\binom{15}{5}$:
$\binom{15}{5} = \frac{15!}{5!10!} = \frac{15 \times 14 \times 13 \times 12 \times 11}{5 \times 4 \times 3 \times 2 \times 1} = 3003$
Calculate the powers:
$3^{10} = 59049$
$(-2)^5 = -32$
Substitute these values back into the expression for the term independent of $x$:
Term independent of $x = 3003 \cdot 59049 \cdot (-32)$
Term independent of $x = 3003 \times 59049 \times (-32) = 177347947 \times (-32) = -5675134304$
The term independent of $x$ in the expansion is $-5675134304$.
The final answer is $\boxed{-5675134304}$.
Question 5. Find the middle term (terms) in the expansion of
(i) $\left( \frac{x}{a} − \frac{a}{x} \right)^{10}$
(ii) $\left( 3x - \frac{x^3}{6} \right)^9$
Answer:
Given: Expansions (i) $\left( \frac{x}{a} − \frac{a}{x} \right)^{10}$ and (ii) $\left( 3x - \frac{x^3}{6} \right)^9$.
To Find: Middle term(s).
Solution:
The general term $T_{k+1}$ of $(A+B)^n$ is $\binom{n}{k} A^{n-k} B^k$.
(i) $\left( \frac{x}{a} − \frac{a}{x} \right)^{10}$: Here $n=10$ (even). Total terms = 11. Middle term position = $\frac{10}{2} + 1 = 6^{\text{th}}$. So $k=5$.
$T_6 = \binom{10}{5} \left(\frac{x}{a}\right)^{10-5} \left(-\frac{a}{x}\right)^5 = \binom{10}{5} \left(\frac{x}{a}\right)^5 \left(-\frac{a}{x}\right)^5$
$T_6 = \binom{10}{5} \frac{x^5}{a^5} (-1)^5 \frac{a^5}{x^5} = \binom{10}{5} (-1)$
$\binom{10}{5} = \frac{10 \times 9 \times 8 \times 7 \times 6}{5 \times 4 \times 3 \times 2 \times 1} = 252$.
$T_6 = 252 \times (-1) = -252$.
(ii) $\left( 3x - \frac{x^3}{6} \right)^9$: Here $n=9$ (odd). Total terms = 10. Middle term positions = $\frac{9+1}{2} = 5^{\text{th}}$ and $6^{\text{th}}$.
For $T_5$, $k=4$. $A=3x$, $B=-\frac{x^3}{6}$.
$T_5 = \binom{9}{4} (3x)^{9-4} \left(-\frac{x^3}{6}\right)^4 = \binom{9}{4} (3x)^5 \left(-\frac{x^3}{6}\right)^4$
$T_5 = \binom{9}{4} 3^5 x^5 (-1)^4 \frac{(x^3)^4}{6^4} = \binom{9}{4} 3^5 x^5 \frac{x^{12}}{6^4} = \binom{9}{4} \frac{3^5}{6^4} x^{17}$
$\binom{9}{4} = \frac{9 \times 8 \times 7 \times 6}{4 \times 3 \times 2 \times 1} = 126$.
$\frac{3^5}{6^4} = \frac{3^5}{(2 \times 3)^4} = \frac{3^5}{2^4 \times 3^4} = \frac{3}{2^4} = \frac{3}{16}$.
$T_5 = 126 \cdot \frac{3}{16} x^{17} = \frac{\cancel{126}^{63} \times 3}{\cancel{16}_{8}} x^{17} = \frac{189}{8} x^{17}$.
For $T_6$, $k=5$.
$T_6 = \binom{9}{5} (3x)^{9-5} \left(-\frac{x^3}{6}\right)^5 = \binom{9}{5} (3x)^4 \left(-\frac{x^3}{6}\right)^5$
$T_6 = \binom{9}{5} 3^4 x^4 (-1)^5 \frac{(x^3)^5}{6^5} = \binom{9}{5} 3^4 x^4 (-1) \frac{x^{15}}{6^5} = -\binom{9}{5} \frac{3^4}{6^5} x^{19}$
$\binom{9}{5} = \binom{9}{4} = 126$.
$\frac{3^4}{6^5} = \frac{3^4}{(2 \times 3)^5} = \frac{3^4}{2^5 \times 3^5} = \frac{1}{2^5 \times 3} = \frac{1}{32 \times 3} = \frac{1}{96}$.
$T_6 = -126 \cdot \frac{1}{96} x^{19} = -\frac{\cancel{126}^{21}}{\cancel{96}_{16}} x^{19} = -\frac{21}{16} x^{19}$.
The middle term for (i) is -252.
The middle terms for (ii) are $\frac{189}{8} x^{17}$ and $-\frac{21}{16} x^{19}$.
The final answer is $\boxed{\text{(i) -252, (ii) } \frac{189}{8} x^{17}, -\frac{21}{16} x^{19}}$.
Question 6. Find the coefficient of x15 in the expansion of (x – x2)10.
Answer:
Given:
The expansion is $(x – x^2)^{10}$.
To Find:
The coefficient of $x^{15}$ in the expansion.
Solution:
We can factor out $x$ from the base of the binomial:
$(x – x^2)^{10} = (x(1 – x))^{10} = x^{10} (1 – x)^{10}$
Now, expand $(1-x)^{10}$ using the binomial theorem. The general term in the expansion of $(1+y)^n$ is $\binom{n}{k} y^k$. Here $y=-x$ and $n=10$.
$(1-x)^{10} = \sum_{k=0}^{10} \binom{10}{k} (-x)^k = \sum_{k=0}^{10} \binom{10}{k} (-1)^k x^k$
Substitute this back into the expression:
$(x – x^2)^{10} = x^{10} \sum_{k=0}^{10} \binom{10}{k} (-1)^k x^k$
Multiply $x^{10}$ with each term in the sum:
$(x – x^2)^{10} = \sum_{k=0}^{10} \binom{10}{k} (-1)^k x^{10} x^k$
Using the property $x^p x^q = x^{p+q}$:
$(x – x^2)^{10} = \sum_{k=0}^{10} \binom{10}{k} (-1)^k x^{10+k}$
We want to find the coefficient of $x^{15}$. This means the power of $x$ must be equal to 15.
So, we set the exponent of $x$ equal to 15:
$10 + k = 15$
Solve for $k$:
$k = 15 - 10$
$k = 5$
The index $k$ in the summation ranges from 0 to 10. Since $k=5$ is within this range ($0 \leq 5 \leq 10$), there is a term with $x^{15}$.
The coefficient of $x^{15}$ is the coefficient of the term where $k = 5$, which is $\binom{10}{k} (-1)^k$ for $k=5$.
Coefficient of $x^{15} = \binom{10}{5} (-1)^5$
Calculate the binomial coefficient $\binom{10}{5}$:
$\binom{10}{5} = \frac{10!}{5!5!} = \frac{10 \times 9 \times 8 \times 7 \times 6}{5 \times 4 \times 3 \times 2 \times 1} = 252$
Calculate $(-1)^5$:
$(-1)^5 = -1$
Multiply the results:
Coefficient of $x^{15} = 252 \times (-1) = -252$
The coefficient of $x^{15}$ in the expansion of $(x – x^2)^{10}$ is $-252$.
The final answer is $\boxed{-252}$.
Question 7. Find the coefficient of $\frac{1}{x^{17}}$ in the expansion of $\left( x^4 - \frac{1}{x^3} \right)^{15}$ .
Answer:
Given:
The expansion is $\left( x^4 - \frac{1}{x^3} \right)^{15}$.
To Find:
The coefficient of $\frac{1}{x^{17}}$ (which is $x^{-17}$) in the expansion.
Solution:
The given expansion is of the form $(a+b)^n$, where $a = x^4$, $b = -\frac{1}{x^3} = -x^{-3}$, and $n = 15$.
The general term, $T_{k+1}$, in the expansion of $(a+b)^n$ is given by the formula:
$T_{k+1} = \binom{n}{k} a^{n-k} b^k$
Substitute the values of $n$, $a$, and $b$ for the given expansion:
$T_{k+1} = \binom{15}{k} (x^4)^{15-k} (-x^{-3})^k$
Simplify the terms involving $x$ and the constant terms separately.
Constant part: $\binom{15}{k} (-1)^k$
Variable part: $(x^4)^{15-k} \cdot (x^{-3})^k = x^{4(15-k)} \cdot x^{-3k} = x^{60-4k} \cdot x^{-3k} = x^{60-4k-3k} = x^{60-7k}$
The general term is $T_{k+1} = \binom{15}{k} (-1)^k x^{60-7k}$.
For the coefficient of $x^{-17}$, we set the power of $x$ equal to $-17$:
$60 - 7k = -17$
Solve for $k$:
$60 + 17 = 7k$
$77 = 7k$
$k = \frac{77}{7}$
$k = 11$
The index $k$ in the general term $T_{k+1}$ must be a non-negative integer, where $0 \leq k \leq n$. In this case, $n=15$, so $k$ must be an integer such that $0 \leq k \leq 15$.
Since $k=11$ is within this range ($0 \leq 11 \leq 15$), there is a term with $x^{-17}$.
The coefficient of $x^{-17}$ is the constant part of the general term when $k=11$.
Coefficient of $x^{-17} = \binom{15}{11} (-1)^{11}$
Calculate the binomial coefficient $\binom{15}{11}$:
$\binom{15}{11} = \binom{15}{15-11} = \binom{15}{4}$
$\binom{15}{4} = \frac{15!}{4!11!} = \frac{15 \times 14 \times 13 \times 12}{4 \times 3 \times 2 \times 1} = \frac{\cancel{15}^{5} \times \cancel{14}^{7} \times 13 \times \cancel{12}^{1}}{\cancel{24}_{1}} = 5 \times 7 \times 13 \times 1 = 455$
Calculate $(-1)^{11}$:
$(-1)^{11} = -1$
Multiply the results:
Coefficient of $x^{-17} = 455 \times (-1) = -455$
The coefficient of $\frac{1}{x^{17}}$ in the expansion is $-455$.
The final answer is $\boxed{-455}$.
Question 8. Find the sixth term of the expansion $\left( y^{\frac{1}{2}} + x^{\frac{1}{3}} \right)^n$, if the binomial coefficient of the third term from the end is 45.
[Hint: Binomial coefficient of third term from the end = Binomial coefficient of third term from beginning = nC2.]
Answer:
Given:
The expansion is $\left( y^{\frac{1}{2}} + x^{\frac{1}{3}} \right)^n$.
The binomial coefficient of the third term from the end is 45.
To Find:
The sixth term of the expansion.
Solution:
The given expansion is of the form $(a+b)^n$, where $a = y^{1/2}$ and $b = x^{1/3}$.
The total number of terms in the expansion is $n+1$.
The third term from the end is the $( (n+1) - 3 + 1 )^{\text{th}}$ term from the beginning.
Third term from the end is the $(n - 1)^{\text{th}}$ term from the beginning.
The coefficient of the $(r+1)^{\text{th}}$ term from the beginning in $(a+b)^n$ is $\binom{n}{r}$.
The coefficient of the $(n-1)^{\text{th}}$ term from the beginning corresponds to $r+1 = n-1$, so $r = n-2$.
The binomial coefficient of the third term from the end is $\binom{n}{n-2}$.
Using the property $\binom{n}{k} = \binom{n}{n-k}$, we have $\binom{n}{n-2} = \binom{n}{n-(n-2)} = \binom{n}{2}$.
We are given that this coefficient is 45.
$\binom{n}{2} = 45$
$\frac{n(n-1)}{2!} = 45$
$\frac{n(n-1)}{2} = 45$
$n(n-1) = 45 \times 2$
$n(n-1) = 90$
We need to find two consecutive integers whose product is 90. By inspection, $10 \times 9 = 90$.
So, $n=10$ (and $n-1=9$). Since $n$ is the power of the binomial, it must be a positive integer. $n=10$ is a valid value.
Now that we know $n=10$, the expansion is $\left( y^{\frac{1}{2}} + x^{\frac{1}{3}} \right)^{10}$.
We need to find the sixth term of this expansion.
The sixth term is $T_6$. For $T_6$, we set $k+1=6$ in the general term formula $T_{k+1} = \binom{n}{k} a^{n-k} b^k$. So $k=5$.
Here $n=10$, $a = y^{1/2}$, $b = x^{1/3}$.
$T_6 = \binom{10}{5} (y^{1/2})^{10-5} (x^{1/3})^5$
$T_6 = \binom{10}{5} (y^{1/2})^5 (x^{1/3})^5$
$T_6 = \binom{10}{5} y^{(1/2) \times 5} x^{(1/3) \times 5}$
$T_6 = \binom{10}{5} y^{5/2} x^{5/3}$
Calculate the binomial coefficient $\binom{10}{5}$:
$\binom{10}{5} = \frac{10!}{5!5!} = \frac{10 \times 9 \times 8 \times 7 \times 6}{5 \times 4 \times 3 \times 2 \times 1} = 252$
Substitute the value back into the expression for $T_6$:
$T_6 = 252 y^{5/2} x^{5/3}$
The sixth term of the expansion is $252 y^{5/2} x^{5/3}$.
The final answer is $\boxed{252 y^{5/2} x^{5/3}}$.
Question 9. Find the value of r, if the coefficients of (2r + 4)th and (r – 2)th terms in the expansion of (1 + x)18 are equal.
Answer:
Given:
The expansion is $(1 + x)^{18}$.
The coefficients of the $(2r + 4)^{\text{th}}$ and $(r – 2)^{\text{th}}$ terms are equal.
To Find:
The value of $r$.
Solution:
The expansion is $(1+x)^{18}$, so $n=18$.
The coefficient of the $m^{\text{th}}$ term in the expansion of $(1+x)^n$ is $\binom{n}{m-1}$.
The coefficient of the $(2r+4)^{\text{th}}$ term is $\binom{18}{(2r+4)-1} = \binom{18}{2r+3}$.
The coefficient of the $(r-2)^{\text{th}}$ term is $\binom{18}{(r-2)-1} = \binom{18}{r-3}$.
We are given that these coefficients are equal:
$\binom{18}{2r+3} = \binom{18}{r-3}$
For the binomial coefficients $\binom{n}{a}$ and $\binom{n}{b}$ to be equal, there are two possibilities:
1. $a = b$
2. $a + b = n$
Case 1: $2r + 3 = r - 3$
$2r - r = -3 - 3$
$r = -6$
The term number must be a positive integer. The $(r-2)^{\text{th}}$ term means $r-2 \geq 1$, so $r \geq 3$. Also, the index in the binomial coefficient must be non-negative, $r-3 \geq 0$, so $r \geq 3$. The $(2r+4)^{\text{th}}$ term means $2r+4 \geq 1$, and $2r+3 \geq 0$, so $2r \geq -3$, $r \geq -3/2$. The value $r=-6$ does not satisfy $r \geq 3$. So this case is not possible.
Case 2: $(2r + 3) + (r - 3) = 18$
$2r + 3 + r - 3 = 18$
$3r = 18$
$r = \frac{18}{3}$
$r = 6$
Check if this value of $r$ is valid for the term numbers and binomial coefficient indices:
$(2r+4)^{\text{th}}$ term: $2(6)+4 = 12+4 = 16^{\text{th}}$ term. The index is $2r+3 = 2(6)+3 = 12+3 = 15$. $\binom{18}{15}$ is valid ($0 \leq 15 \leq 18$).
$(r-2)^{\text{th}}$ term: $6-2 = 4^{\text{th}}$ term. The index is $r-3 = 6-3 = 3$. $\binom{18}{3}$ is valid ($0 \leq 3 \leq 18$).
The coefficients are $\binom{18}{15}$ and $\binom{18}{3}$. We know $\binom{18}{15} = \binom{18}{18-15} = \binom{18}{3}$, so they are indeed equal.
The value $r=6$ is a valid solution.
The value of $r$ is 6.
The final answer is $\boxed{6}$.
Question 10. If the coefficient of second, third and fourth terms in the expansion of (1 + x)2n are in A.P. Show that 2n2 – 9n + 7 = 0.
Answer:
Given:
The expansion is $(1 + x)^{2n}$.
The coefficients of the second, third, and fourth terms are in Arithmetic Progression (A.P.).
To Show:
$2n^2 – 9n + 7 = 0$.
Proof:
The expansion is $(1+x)^{2n}$. The power of the binomial is $N=2n$.
The coefficient of the $m^{\text{th}}$ term in the expansion of $(1+x)^N$ is $\binom{N}{m-1}$.
Coefficient of the second term ($m=2$) is $\binom{2n}{2-1} = \binom{2n}{1}$.
Coefficient of the third term ($m=3$) is $\binom{2n}{3-1} = \binom{2n}{2}$.
Coefficient of the fourth term ($m=4$) is $\binom{2n}{4-1} = \binom{2n}{3}$.
We are given that these coefficients are in A.P.
If three numbers $a, b, c$ are in A.P., then $2b = a+c$.
So, $2 \cdot \binom{2n}{2} = \binom{2n}{1} + \binom{2n}{3}$.
Write out the binomial coefficients:
$\binom{2n}{1} = \frac{(2n)!}{1!(2n-1)!} = 2n$
$\binom{2n}{2} = \frac{(2n)!}{2!(2n-2)!} = \frac{2n(2n-1)}{2 \times 1} = n(2n-1)$
$\binom{2n}{3} = \frac{(2n)!}{3!(2n-3)!} = \frac{2n(2n-1)(2n-2)}{3 \times 2 \times 1} = \frac{2n(2n-1)2(n-1)}{6} = \frac{2n(2n-1)(n-1)}{3}$
Substitute these values into the A.P. condition:
$2 \cdot n(2n-1) = 2n + \frac{2n(2n-1)(n-1)}{3}$
... (i)
We can assume $n \ge 2$ for the third and fourth terms to exist and for the denominators of the binomial coefficients to be non-zero. If $n=1$, the expansion is $(1+x)^2 = 1+2x+x^2$. The 2nd, 3rd, 4th terms are $2x, x^2, 0$. Coefficients are 2, 1, 0. Are 2, 1, 0 in A.P.? $2(1) = 2+0$, $2=2$. Yes, they are in A.P. If $n=1$, the equation $2n^2 - 9n + 7 = 0$ becomes $2(1)^2 - 9(1) + 7 = 2 - 9 + 7 = 0$. So $n=1$ satisfies the equation. Let's proceed assuming $n \ge 2$ and see if we get the same equation.
Divide equation (i) by $2n$ (since for terms to exist, $n \ge 2$, so $2n \neq 0$):
$2(2n-1) = 1 + \frac{(2n-1)(n-1)}{3}$
$4n - 2 = 1 + \frac{(2n-1)(n-1)}{3}$
Multiply by 3:
$3(4n - 2) = 3(1) + (2n-1)(n-1)$
$12n - 6 = 3 + (2n^2 - 2n - n + 1)$
$12n - 6 = 3 + 2n^2 - 3n + 1$
$12n - 6 = 2n^2 - 3n + 4$
Move all terms to one side to form a quadratic equation:
$0 = 2n^2 - 3n - 12n + 4 + 6$
$0 = 2n^2 - 15n + 10$
Wait, the required equation is $2n^2 - 9n + 7 = 0$. Let's recheck the A.P. condition and the calculation.
Coefficients are $c_1 = \binom{2n}{1}$, $c_2 = \binom{2n}{2}$, $c_3 = \binom{2n}{3}$.
$c_1 = 2n$
$c_2 = \frac{2n(2n-1)}{2} = n(2n-1)$
$c_3 = \frac{2n(2n-1)(2n-2)}{6} = \frac{2n(2n-1)2(n-1)}{6} = \frac{2n(2n-1)(n-1)}{3}$
A.P. condition: $2c_2 = c_1 + c_3$
$2 \cdot n(2n-1) = 2n + \frac{2n(2n-1)(n-1)}{3}$
Divide by $2n$ (assuming $n \ge 2$, so $2n \neq 0$):
$2n-1 = 1 + \frac{(2n-1)(n-1)}{3}$
Multiply by 3:
$3(2n-1) = 3 + (2n-1)(n-1)$
$6n - 3 = 3 + (2n^2 - 2n - n + 1)$
$6n - 3 = 3 + 2n^2 - 3n + 1$
$6n - 3 = 2n^2 - 3n + 4$
Move all terms to the right side:
$0 = 2n^2 - 3n - 6n + 4 + 3$
$0 = 2n^2 - 9n + 7$
This is the required equation.
We assumed $n \ge 2$. If $n=1$, the coefficients are $\binom{2}{1}=2$, $\binom{2}{2}=1$, $\binom{2}{3}=0$. These are 2, 1, 0, which are in A.P. The equation $2n^2 - 9n + 7 = 0$ for $n=1$ is $2(1)^2 - 9(1) + 7 = 2 - 9 + 7 = 0$, which is true. So $n=1$ is a possible value of $n$ that satisfies the condition.
The derivation $2n^2 - 9n + 7 = 0$ holds for $n \ge 2$. Since $n=1$ also satisfies the equation, the equation holds for all positive integers $n$ where the second, third, and fourth terms exist (i.e., $2n \ge 3$, which means $n \ge 3/2$). So, the derivation is valid for $n \ge 2$, and we checked $n=1$ separately.
We have shown that if the coefficients of the second, third, and fourth terms in the expansion of $(1+x)^{2n}$ are in A.P., then $2n^2 - 9n + 7 = 0$.
Question 11. Find the coefficient of x4 in the expansion of (1 + x + x2 + x3)11.
Answer:
Given:
The expansion is $(1 + x + x^2 + x^3)^{11}$.
To Find:
The coefficient of $x^4$ in the expansion.
Solution:
Let the given expression be $E$.
$E = (1 + x + x^2 + x^3)^{11}$
We can factor the base expression $1 + x + x^2 + x^3$. This is a geometric series with first term 1, ratio $x$, and 4 terms.
$1 + x + x^2 + x^3 = \frac{1(1-x^4)}{1-x} = \frac{1-x^4}{1-x}$, provided $x \neq 1$.
Substitute this factorization back into the expression for $E$:
$E = \left( \frac{1-x^4}{1-x} \right)^{11} = \frac{(1-x^4)^{11}}{(1-x)^{11}}$
$E = (1-x^4)^{11} (1-x)^{-11}$
We need to find the coefficient of $x^4$ in the product of these two expansions.
Expand each factor using the binomial theorem:
$(1-x^4)^{11} = \binom{11}{0}(1)^{11}(-x^4)^0 + \binom{11}{1}(1)^{10}(-x^4)^1 + \binom{11}{2}(1)^{9}(-x^4)^2 + \binom{11}{3}(1)^{8}(-x^4)^3 + \dots$
$(1-x^4)^{11} = \binom{11}{0} - \binom{11}{1}x^4 + \binom{11}{2}x^8 - \binom{11}{3}x^{12} + \dots$
$= 1 - 11x^4 + \binom{11}{2}x^8 - \dots$
We are interested in terms up to $x^4$. So we only need the first two terms of this expansion:
$(1-x^4)^{11} = 1 - 11x^4 + (\text{terms with } x^8, x^{12}, \dots)$
Now consider the expansion of $(1-x)^{-11}$. We use the generalized binomial theorem for negative integer exponents: $(1-y)^{-n} = \sum_{k=0}^\infty \binom{n+k-1}{k} y^k$.
Here $y=x$ and $n=11$.
$(1-x)^{-11} = \sum_{k=0}^\infty \binom{11+k-1}{k} x^k = \sum_{k=0}^\infty \binom{10+k}{k} x^k$
We need terms up to $x^4$. So we need the terms for $k=0, 1, 2, 3, 4$:
$(1-x)^{-11} = \binom{10+0}{0}x^0 + \binom{10+1}{1}x^1 + \binom{10+2}{2}x^2 + \binom{10+3}{3}x^3 + \binom{10+4}{4}x^4 + \dots$
$(1-x)^{-11} = \binom{10}{0} + \binom{11}{1}x + \binom{12}{2}x^2 + \binom{13}{3}x^3 + \binom{14}{4}x^4 + \dots$
$= 1 + 11x + \frac{12 \times 11}{2}x^2 + \frac{13 \times 12 \times 11}{3 \times 2 \times 1}x^3 + \frac{14 \times 13 \times 12 \times 11}{4 \times 3 \times 2 \times 1}x^4 + \dots$
$= 1 + 11x + 66x^2 + 286x^3 + 1001x^4 + \dots$
Now, multiply the two expansions and collect the terms with $x^4$:
$E = (1 - 11x^4 + \dots) \cdot (1 + 11x + 66x^2 + 286x^3 + 1001x^4 + \dots)$
The terms that produce $x^4$ are formed by multiplying:
1. Constant term from $(1-x^4)^{11}$ with $x^4$ term from $(1-x)^{-11}$:
$1 \cdot (\binom{14}{4}x^4) = 1001x^4$
2. $x^4$ term from $(1-x^4)^{11}$ with constant term from $(1-x)^{-11}$:
$(-11x^4) \cdot (\binom{10}{0}) = -11x^4 \cdot 1 = -11x^4$
Any other combination of terms from the two limited expansions shown will result in a power of $x$ higher than 4. For example, the $x^8$ term from the first factor multiplied by any term from the second factor will give $x^8$ or higher.
The coefficient of $x^4$ is the sum of the coefficients of these terms:
Coefficient of $x^4 = 1001 + (-11) = 1001 - 11 = 990$.
The coefficient of $x^4$ in the expansion of $(1 + x + x^2 + x^3)^{11}$ is 990.
The final answer is $\boxed{990}$.
Question 12 to 17 (Long Answer Type Questions)
Question 12. If p is a real number and if the middle term in the expansion of $\left( \frac{p}{2} + 2 \right)^8$ is 1120, find p.
Answer:
Given:
The expansion is $\left( \frac{p}{2} + 2 \right)^8$, where $p$ is a real number.
The middle term in the expansion is 1120.
To Find:
The value of $p$.
Solution:
The given expansion is of the form $(a+b)^n$, where $a = \frac{p}{2}$, $b = 2$, and $n = 8$.
The total number of terms in the expansion is $n+1 = 8+1 = 9$.
Since the number of terms (9) is odd, there is one middle term.
The position of the middle term is $\frac{n}{2} + 1$ when $n$ is even.
Middle term position = $\frac{8}{2} + 1 = 4 + 1 = 5^{\text{th}}$ term.
We need to find the 5th term ($T_5$) of the expansion.
The general term, $T_{k+1}$, in the expansion of $(a+b)^n$ is given by $T_{k+1} = \binom{n}{k} a^{n-k} b^k$.
For the 5th term, $k+1=5$, so $k=4$.
Substitute the values of $n=8$, $k=4$, $a=\frac{p}{2}$, and $b=2$ into the general term formula:
$T_5 = \binom{8}{4} \left(\frac{p}{2}\right)^{8-4} (2)^4$
$T_5 = \binom{8}{4} \left(\frac{p}{2}\right)^4 (2)^4$
Calculate the binomial coefficient $\binom{8}{4}$:
$\binom{8}{4} = \frac{8!}{4!4!} = \frac{8 \times 7 \times 6 \times 5}{4 \times 3 \times 2 \times 1} = \frac{\cancel{8}^{1} \times 7 \times \cancel{6}^{1} \times 5}{\cancel{24}_{1}} = 7 \times 5 \times 1 = 70$
Calculate the powers of the terms:
$\left(\frac{p}{2}\right)^4 = \frac{p^4}{2^4} = \frac{p^4}{16}$
$(2)^4 = 16$
Substitute these values back into the expression for $T_5$:
$T_5 = 70 \cdot \frac{p^4}{16} \cdot 16$
Cancel out the 16:
$T_5 = 70 p^4$
We are given that the middle term ($T_5$) is 1120.
$70 p^4 = 1120$
…(i)
Divide both sides by 70:
$p^4 = \frac{1120}{70}$
$p^4 = \frac{112}{7}$
Divide 112 by 7:
$\begin{array}{r} 16 \\ 7{\overline{\smash{\big)}\,112}} \\ \underline{-~\phantom{(}(7)}\phantom{0} \\ 42\phantom{)} \\ \underline{-~\phantom{()}(42)} \\ 0\phantom{)} \end{array}$
So, $p^4 = 16$.
To find $p$, take the fourth root of 16:
$p = \pm \sqrt[4]{16}$
Since $2^4 = 16$ and $(-2)^4 = 16$, the possible real values for $p$ are 2 and -2.
We are given that $p$ is a real number, so both 2 and -2 are valid solutions.
The possible values of $p$ are 2 and -2.
The final answer is $\boxed{\pm 2}$.
Question 13. Show that the middle term in the expansion of $\left( x − \frac{1}{x} \right)^{2n}$ is $\frac{1 × 3 × 5 × … (2n − 1)}{n!} × (-2)^n$ .
Answer:
Given:
The expansion is $\left( x − \frac{1}{x} \right)^{2n}$.
To Show:
The middle term is $\frac{1 \times 3 \times 5 \times \dots (2n − 1)}{n!} \times (-2)^n$.
Proof:
The given expansion is of the form $(a+b)^N$, where $a = x$, $b = -\frac{1}{x}$, and $N = 2n$.
The total number of terms in the expansion is $N+1 = 2n+1$.
Since the number of terms ($2n+1$) is odd, there is one middle term.
The position of the middle term is $\frac{N}{2} + 1$ when $N$ is even.
Middle term position = $\frac{2n}{2} + 1 = n + 1^{\text{th}}$ term.
We need to find the $(n+1)^{\text{th}}$ term ($T_{n+1}$) of the expansion.
The general term, $T_{k+1}$, in the expansion of $(a+b)^N$ is given by $T_{k+1} = \binom{N}{k} a^{N-k} b^k$.
For the $(n+1)^{\text{th}}$ term, $k+1 = n+1$, so $k=n$.
Substitute the values of $N=2n$, $k=n$, $a=x$, and $b=-\frac{1}{x}$ into the general term formula:
$T_{n+1} = \binom{2n}{n} (x)^{2n-n} \left(-\frac{1}{x}\right)^n$
$T_{n+1} = \binom{2n}{n} (x)^n \left(-\frac{1}{x}\right)^n$
$T_{n+1} = \binom{2n}{n} x^n (-1)^n \left(\frac{1}{x}\right)^n$
$T_{n+1} = \binom{2n}{n} x^n (-1)^n \frac{1}{x^n}$
Cancel $x^n$ (assuming $x \neq 0$):
$T_{n+1} = \binom{2n}{n} (-1)^n$
Now, we need to express $\binom{2n}{n}$ in the required form.
$\binom{2n}{n} = \frac{(2n)!}{n!n!}$
Expand $(2n)!$:
$(2n)! = (2n) \times (2n-1) \times (2n-2) \times (2n-3) \times \dots \times 3 \times 2 \times 1$
Group the even and odd terms:
$(2n)! = [(2n) \times (2n-2) \times \dots \times 2] \times [(2n-1) \times (2n-3) \times \dots \times 1]$
Factor out 2 from each term in the first bracket (there are $n$ terms):
$(2n) \times (2n-2) \times \dots \times 2 = (2 \times n) \times (2 \times (n-1)) \times \dots \times (2 \times 1)$
$= 2^n \times [n \times (n-1) \times \dots \times 1] = 2^n \cdot n!$
So, $(2n)! = (2^n \cdot n!) \times [1 \times 3 \times 5 \times \dots \times (2n-1)]$
Substitute this back into the expression for $\binom{2n}{n}$:
$\binom{2n}{n} = \frac{2^n \cdot n! \cdot [1 \times 3 \times 5 \times \dots \times (2n-1)]}{n!n!}$
Cancel one $n!$ from the numerator and denominator:
$\binom{2n}{n} = \frac{2^n \cdot [1 \times 3 \times 5 \times \dots \times (2n-1)]}{n!}$
Now, substitute this expression for $\binom{2n}{n}$ back into the middle term formula $T_{n+1} = \binom{2n}{n} (-1)^n$:
$T_{n+1} = \frac{2^n \cdot [1 \times 3 \times 5 \times \dots \times (2n-1)]}{n!} \cdot (-1)^n$
$T_{n+1} = \frac{[1 \times 3 \times 5 \times \dots \times (2n-1)]}{n!} \cdot 2^n \cdot (-1)^n$
$T_{n+1} = \frac{1 \times 3 \times 5 \times \dots (2n − 1)}{n!} \times (-2)^n$
This matches the required expression for the middle term.
The middle term in the expansion of $\left( x − \frac{1}{x} \right)^{2n}$ is $\frac{1 \times 3 \times 5 \times … (2n − 1)}{n!} \times (-2)^n$.
Question 14. Find n in the binomial $\left( \sqrt[3]{2} + \frac{1}{\sqrt[3]{3}} \right)^n$ if the ratio of 7th term from the beginning to the 7th term from the end is $\frac{1}{6}$.
Answer:
Given:
The binomial expansion is $\left( \sqrt[3]{2} + \frac{1}{\sqrt[3]{3}} \right)^n$.
The ratio of the 7th term from the beginning to the 7th term from the end is $\frac{1}{6}$.
To Find:
The value of $n$.
Solution:
The given expansion is of the form $(a+b)^n$, where $a = \sqrt[3]{2} = 2^{1/3}$ and $b = \frac{1}{\sqrt[3]{3}} = 3^{-1/3}$.
The general term, $T_{k+1}$, in the expansion of $(a+b)^n$ is given by $T_{k+1} = \binom{n}{k} a^{n-k} b^k$.
7th term from the beginning ($T_7$):
For the 7th term from the beginning, we set $k+1=7$, so $k=6$.
$T_7 = \binom{n}{6} (2^{1/3})^{n-6} (3^{-1/3})^6$
$T_7 = \binom{n}{6} 2^{(1/3)(n-6)} 3^{(-1/3) \times 6}$
$T_7 = \binom{n}{6} 2^{\frac{n-6}{3}} 3^{-2}$
$T_7 = \binom{n}{6} 2^{\frac{n-6}{3}} \frac{1}{9}$
... (i)
7th term from the end:
The total number of terms in the expansion is $n+1$. The $r^{\text{th}}$ term from the end is the $( (n+1) - r + 1 )^{\text{th}}$ term from the beginning.
For the 7th term from the end, $r=7$. The position from the beginning is $(n+1 - 7 + 1) = (n-5)^{\text{th}}$.
The $(n-5)^{\text{th}}$ term from the beginning is $T_{n-5}$. For this term, $k+1 = n-5$, so $k=n-6$.
$T_{n-5} = \binom{n}{n-6} (2^{1/3})^{n-(n-6)} (3^{-1/3})^{n-6}$
$T_{n-5} = \binom{n}{n-6} (2^{1/3})^6 (3^{-1/3})^{n-6}$
$T_{n-5} = \binom{n}{n-6} 2^{6/3} 3^{(-1/3)(n-6)}$
$T_{n-5} = \binom{n}{n-6} 2^2 3^{-\frac{n-6}{3}}$
Using the property $\binom{n}{n-k} = \binom{n}{k}$, we have $\binom{n}{n-6} = \binom{n}{6}$.
$T_{n-5} = \binom{n}{6} 2^2 3^{-\frac{n-6}{3}}$
$T_{n-5} = \binom{n}{6} 4 \cdot 3^{-\frac{n-6}{3}}$
... (ii)
We are given that the ratio of the 7th term from the beginning to the 7th term from the end is $\frac{1}{6}$.
$\frac{T_7}{T_{n-5}} = \frac{1}{6}$
(Given)
Substitute the expressions for $T_7$ and $T_{n-5}$ from (i) and (ii):
$\frac{\binom{n}{6} 2^{\frac{n-6}{3}} 3^{-2}}{\binom{n}{6} 2^2 3^{-\frac{n-6}{3}}} = \frac{1}{6}$
Cancel out $\binom{n}{6}$ (assuming $n \ge 6$ for the 7th term to exist) and simplify the powers using exponent rules:
$2^{\frac{n-6}{3} - 2} \cdot 3^{-2 - (-\frac{n-6}{3})} = \frac{1}{6}$
$2^{\frac{n-6-6}{3}} \cdot 3^{-2 + \frac{n-6}{3}} = 6^{-1}$
$2^{\frac{n-12}{3}} \cdot 3^{\frac{-6+n-6}{3}} = 6^{-1}$
$2^{\frac{n-12}{3}} \cdot 3^{\frac{n-12}{3}} = 6^{-1}$
$(2 \cdot 3)^{\frac{n-12}{3}} = 6^{-1}$
$6^{\frac{n-12}{3}} = 6^{-1}$
Since the bases are equal, the exponents must be equal:
$\frac{n-12}{3} = -1$
Multiply both sides by 3:
$n - 12 = -3$
Add 12 to both sides:
$n = -3 + 12$
$n = 9$
Check if $n=9$ is valid. For the 7th term to exist, $n$ must be at least 6. $9 \ge 6$, so this is valid.
The value of $n$ is 9.
The final answer is $\boxed{9}$.
Question 15. In the expansion of (x + a)n if the sum of odd terms is denoted by O and the sum of even term by E.
Then prove that
(i) O2 – E2 = (x2 – a2)n
(ii) 4OE = (x + a)2n – (x – a)2n
Answer:
Given:
The expansion is $(x+a)^n$.
$O$ = sum of odd terms in the expansion.
$E$ = sum of even terms in the expansion.
To Prove:
(i) $O^2 – E^2 = (x^2 – a^2)^n$
(ii) $4OE = (x + a)^{2n} – (x – a)^{2n}$
Proof:
The binomial expansion of $(x+a)^n$ is:
$(x+a)^n = \binom{n}{0}x^n a^0 + \binom{n}{1}x^{n-1} a^1 + \binom{n}{2}x^{n-2} a^2 + \binom{n}{3}x^{n-3} a^3 + \dots$
The terms in the expansion are $T_1, T_2, T_3, T_4, \dots$.
$T_1 = \binom{n}{0}x^n a^0$ (Odd term, as the index of the term is 1)
$T_2 = \binom{n}{1}x^{n-1} a^1$ (Even term, as the index of the term is 2)
$T_3 = \binom{n}{2}x^{n-2} a^2$ (Odd term)
$T_4 = \binom{n}{3}x^{n-3} a^3$ (Even term)
And so on.
The sum of odd terms, $O$, is the sum of $T_1, T_3, T_5, \dots$:
$O = \binom{n}{0}x^n a^0 + \binom{n}{2}x^{n-2} a^2 + \binom{n}{4}x^{n-4} a^4 + \dots$
The sum of even terms, $E$, is the sum of $T_2, T_4, T_6, \dots$:
$E = \binom{n}{1}x^{n-1} a^1 + \binom{n}{3}x^{n-3} a^3 + \binom{n}{5}x^{n-5} a^5 + \dots$
The sum of the entire expansion is $O+E$:
$O+E = (x+a)^n$
... (i)
Now consider the expansion of $(x-a)^n$. Substitute $a$ with $-a$ in the expansion of $(x+a)^n$.
$(x-a)^n = \binom{n}{0}x^n (-a)^0 + \binom{n}{1}x^{n-1} (-a)^1 + \binom{n}{2}x^{n-2} (-a)^2 + \binom{n}{3}x^{n-3} (-a)^3 + \dots$
$(x-a)^n = \binom{n}{0}x^n a^0 - \binom{n}{1}x^{n-1} a^1 + \binom{n}{2}x^{n-2} a^2 - \binom{n}{3}x^{n-3} a^3 + \dots$
The terms with an even index for the power of $a$ (which correspond to odd terms in the $(x+a)^n$ expansion) remain positive. The terms with an odd index for the power of $a$ (which correspond to even terms in the $(x+a)^n$ expansion) become negative.
So, the expansion of $(x-a)^n$ is the sum of the odd terms of $(x+a)^n$ minus the sum of the even terms of $(x+a)^n$.
$(x-a)^n = O - E$
... (ii)
Part (i) Proof:
Consider the expression $O^2 - E^2$. This is a difference of squares, which can be factored as $(O-E)(O+E)$.
$O^2 - E^2 = (O-E)(O+E)$
Substitute the values of $(O+E)$ from (i) and $(O-E)$ from (ii):
$O^2 - E^2 = (x-a)^n \cdot (x+a)^n$
Using the property $(bc)^m = b^m c^m$, we have $(x-a)^n (x+a)^n = ((x-a)(x+a))^n$.
Using the difference of squares formula, $(x-a)(x+a) = x^2 - a^2$.
$O^2 - E^2 = (x^2 - a^2)^n$
Thus, the first part of the identity is proven.
Part (ii) Proof:
Consider the expression $4OE$. We can write this as $4 \cdot (O \cdot E)$.
We know $(O+E) = (x+a)^n$ and $(O-E) = (x-a)^n$.
Multiply these two equations:
$(O+E)(O-E) = (x+a)^n (x-a)^n$
$O^2 - E^2 = (x^2 - a^2)^n$ (This brings us back to part (i))
Let's consider $(O+E)^2$ and $(O-E)^2$.
$(O+E)^2 = ((x+a)^n)^2 = (x+a)^{2n}$
$(O-E)^2 = ((x-a)^n)^2 = (x-a)^{2n}$
Expand the left sides:
$(O+E)^2 = O^2 + 2OE + E^2$
$(O-E)^2 = O^2 - 2OE + E^2$
Subtract the second expanded equation from the first:
$(O^2 + 2OE + E^2) - (O^2 - 2OE + E^2) = (O+E)^2 - (O-E)^2$
$O^2 + 2OE + E^2 - O^2 + 2OE - E^2 = (O+E)^2 - (O-E)^2$
$4OE = (O+E)^2 - (O-E)^2$
Substitute the expressions for $(O+E)^2$ and $(O-E)^2$:
$4OE = (x+a)^{2n} - (x-a)^{2n}$
Thus, the second part of the identity is proven.
We have shown that $O^2 – E^2 = (x^2 – a^2)^n$ and $4OE = (x + a)^{2n} – (x – a)^{2n}$.
Question 16. If xp occurs in the expansion of $\left( x^2 + \frac{1}{x} \right)^{2n}$, prove that its coefficient is
$\frac{2n \ !}{(\frac{4n - p}{3}) \ ! \ (\frac{2n + p}{3}) \ !}$
Answer:
Given:
The expansion is $\left( x^2 + \frac{1}{x} \right)^{2n}$.
The term containing $x^p$ occurs in this expansion, where $p$ is an integer.
To Prove:
The coefficient of $x^p$ is $\frac{(2n)!}{\left(\frac{4n - p}{3}\right)! \left(\frac{2n + p}{3}\right)!}$.
Proof:
The given expansion is of the form $(a+b)^N$, where $a = x^2$, $b = \frac{1}{x} = x^{-1}$, and $N = 2n$.
The general term, $T_{k+1}$, in the expansion of $(a+b)^N$ is given by the formula:
$T_{k+1} = \binom{N}{k} a^{N-k} b^k$
Substitute the values of $N=2n$, $a=x^2$, and $b=x^{-1}$ for the given expansion:
$T_{k+1} = \binom{2n}{k} (x^2)^{2n-k} (x^{-1})^k$
Simplify the terms involving $x$:
$(x^2)^{2n-k} = x^{2(2n-k)} = x^{4n-2k}$
$(x^{-1})^k = x^{-k}$
The variable part of the general term is the product of the terms involving $x$:
$x^{4n-2k} \cdot x^{-k} = x^{(4n-2k) - k} = x^{4n-3k}$
The general term is $T_{k+1} = \binom{2n}{k} x^{4n-3k}$.
We are told that $x^p$ occurs in the expansion. This means there exists a value of $k$ (where $k$ is a non-negative integer and $0 \leq k \leq 2n$) such that the power of $x$ in the general term is equal to $p$.
Set the exponent of $x$ equal to $p$:
$4n - 3k = p$
Solve for $k$:
$4n - p = 3k$
$k = \frac{4n - p}{3}$
For a term containing $x^p$ to exist, $k$ must be a non-negative integer such that $0 \leq k \leq 2n$. This implies that $(4n-p)$ must be divisible by 3, and the value of $k$ must fall within the valid range for the binomial coefficient.
If such an integer $k$ exists, the coefficient of $x^p$ is given by $\binom{2n}{k}$ with $k = \frac{4n-p}{3}$.
Coefficient of $x^p = \binom{2n}{\frac{4n-p}{3}}$
Using the definition of the binomial coefficient $\binom{N}{K} = \frac{N!}{K!(N-K)!}$, we have:
Coefficient of $x^p = \frac{(2n)!}{\left(\frac{4n-p}{3}\right)! \left(2n - \frac{4n-p}{3}\right)!}$
Simplify the term in the second factorial in the denominator:
$2n - \frac{4n-p}{3} = \frac{3(2n) - (4n-p)}{3} = \frac{6n - 4n + p}{3} = \frac{2n + p}{3}$
Substitute this back into the denominator:
Coefficient of $x^p = \frac{(2n)!}{\left(\frac{4n-p}{3}\right)! \left(\frac{2n+p}{3}\right)!}$
This is the required expression for the coefficient of $x^p$. The existence of the term $x^p$ guarantees that $\frac{4n-p}{3}$ is a non-negative integer $\le 2n$.
Thus, if $x^p$ occurs in the expansion of $\left( x^2 + \frac{1}{x} \right)^{2n}$, its coefficient is $\frac{(2n)!}{\left(\frac{4n - p}{3}\right)! \left(\frac{2n + p}{3}\right)!}$.
Question 17. Find the term independent of x in the expansion of $(1 + x + 2x^3) \left( \frac{3}{2} x^2 - \frac{1}{3x} \right)^9$.
Answer:
Given:
The expression is $(1 + x + 2x^3) \left( \frac{3}{2} x^2 - \frac{1}{3x} \right)^9$.
To Find:
The term independent of $x$ in the expansion of the given expression.
Solution:
Let the given expression be $E$.
$E = (1 + x + 2x^3) \cdot \left( \frac{3}{2} x^2 - \frac{1}{3x} \right)^9$
Let the expansion of $\left( \frac{3}{2} x^2 - \frac{1}{3x} \right)^9$ be denoted by $S$. We need to find the terms in $S$ that, when multiplied by the terms in $(1 + x + 2x^3)$, result in a term independent of $x$ (i.e., a term with $x^0$).
The terms in the first factor are $1, x, 2x^3$.
To get a term independent of $x$ in the product, we need to multiply:
1. The constant term (term with $x^0$) from $(1 + x + 2x^3)$ by the term with $x^0$ from $S$.
2. The term with $x^1$ from $(1 + x + 2x^3)$ by the term with $x^{-1}$ from $S$.
3. The term with $x^3$ from $(1 + x + 2x^3)$ by the term with $x^{-3}$ from $S$.
Let's find the general term of the expansion $S = \left( \frac{3}{2} x^2 - \frac{1}{3x} \right)^9$. This is of the form $(a+b)^n$, where $a = \frac{3}{2} x^2$, $b = -\frac{1}{3x}$, and $n = 9$.
The general term, $T_{k+1}$, is $T_{k+1} = \binom{n}{k} a^{n-k} b^k$.
$T_{k+1} = \binom{9}{k} \left(\frac{3}{2} x^2\right)^{9-k} \left(-\frac{1}{3x}\right)^k$
Simplify the terms involving $x$ and the constant terms:
Constant part: $\binom{9}{k} \left(\frac{3}{2}\right)^{9-k} \left(-\frac{1}{3}\right)^k$
Variable part: $(x^2)^{9-k} \cdot \left(\frac{1}{x}\right)^k = x^{2(9-k)} \cdot x^{-k} = x^{18-2k} \cdot x^{-k} = x^{18-3k}$
The general term of $S$ is $T_{k+1} = \binom{9}{k} \left(\frac{3}{2}\right)^{9-k} \left(-\frac{1}{3}\right)^k x^{18-3k}$.
Now, let's find the required terms from the expansion $S$:
1. Term independent of $x$ (coefficient of $x^0$) in $S$: Set the exponent of $x$ to 0.
$18 - 3k = 0 \implies 3k = 18 \implies k = 6$.
This is the 7th term ($T_7$) of $S$. The constant part is the term independent of $x$ in $S$.
Term with $x^0$ in $S = \binom{9}{6} \left(\frac{3}{2}\right)^{9-6} \left(-\frac{1}{3}\right)^6 = \binom{9}{6} \left(\frac{3}{2}\right)^3 \left(-\frac{1}{3}\right)^6$
$\binom{9}{6} = \binom{9}{3} = \frac{9 \times 8 \times 7}{3 \times 2 \times 1} = 84$.
$\left(\frac{3}{2}\right)^3 = \frac{27}{8}$.
$\left(-\frac{1}{3}\right)^6 = \frac{1}{3^6} = \frac{1}{729}$.
Term with $x^0$ in $S = 84 \cdot \frac{27}{8} \cdot \frac{1}{729} = \frac{84 \times 27}{8 \times 729} = \frac{\cancel{84}^{21} \times \cancel{27}^{1}}{\cancel{8}_{2} \times \cancel{729}_{27}} = \frac{21 \times 1}{2 \times 27} = \frac{\cancel{21}^{7}}{\cancel{54}_{18}} = \frac{7}{18}$.
This term will be multiplied by the constant term (1) from $(1 + x + 2x^3)$. Contribution to the term independent of $x$ is $1 \times \frac{7}{18} = \frac{7}{18}$.
2. Term with $x^{-1}$ in $S$: Set the exponent of $x$ to -1.
$18 - 3k = -1 \implies 3k = 18 + 1 \implies 3k = 19 \implies k = \frac{19}{3}$.
Since $k = \frac{19}{3}$ is not an integer, there is no term with $x^{-1}$ in the expansion of $S$. The coefficient of $x^{-1}$ in $S$ is 0.
This term would be multiplied by the $x^1$ term from $(1 + x + 2x^3)$. Contribution to the term independent of $x$ is $x \times 0 = 0$.
3. Term with $x^{-3}$ in $S$: Set the exponent of $x$ to -3.
$18 - 3k = -3 \implies 3k = 18 + 3 \implies 3k = 21 \implies k = 7$.
Since $k=7$ is an integer and $0 \leq 7 \leq 9$, this is a valid term (the 8th term, $T_8$) of $S$. The constant part is:
Coefficient of $x^{-3}$ in $S = \binom{9}{7} \left(\frac{3}{2}\right)^{9-7} \left(-\frac{1}{3}\right)^7 = \binom{9}{7} \left(\frac{3}{2}\right)^2 \left(-\frac{1}{3}\right)^7$
$\binom{9}{7} = \binom{9}{9-7} = \binom{9}{2} = \frac{9 \times 8}{2 \times 1} = 36$.
$\left(\frac{3}{2}\right)^2 = \frac{9}{4}$.
$\left(-\frac{1}{3}\right)^7 = (-1)^7 \frac{1}{3^7} = -\frac{1}{2187}$.
Coefficient of $x^{-3}$ in $S = 36 \cdot \frac{9}{4} \cdot \left(-\frac{1}{2187}\right) = \frac{\cancel{36}^{9} \times 9}{\cancel{4}_{1} \times 2187} \times (-1) = -\frac{9 \times 9}{2187} = -\frac{81}{2187}$.
Simplify the fraction: $2187 = 3^7$, $81 = 3^4$. $\frac{81}{2187} = \frac{3^4}{3^7} = 3^{4-7} = 3^{-3} = \frac{1}{27}$.
Coefficient of $x^{-3}$ in $S = -\frac{1}{27}$.
This term will be multiplied by the $2x^3$ term from $(1 + x + 2x^3)$. Contribution to the term independent of $x$ is $2x^3 \times (-\frac{1}{27} x^{-3}) = 2 \times (-\frac{1}{27}) x^{3-3} = -\frac{2}{27}$.
The term independent of $x$ in the overall expansion is the sum of the contributions from these three cases:
Term independent of $x = (\text{Contribution from } 1 \cdot x^0 \text{ in } S) + (\text{Contribution from } x \cdot x^{-1} \text{ in } S) + (\text{Contribution from } 2x^3 \cdot x^{-3} \text{ in } S)$
Term independent of $x = \frac{7}{18} + 0 + \left(-\frac{2}{27}\right)$
Term independent of $x = \frac{7}{18} - \frac{2}{27}$
Find a common denominator, which is LCM(18, 27). $18 = 2 \times 3^2$, $27 = 3^3$. LCM(18, 27) = $2 \times 3^3 = 2 \times 27 = 54$.
$\frac{7}{18} = \frac{7 \times 3}{18 \times 3} = \frac{21}{54}$
$\frac{2}{27} = \frac{2 \times 2}{27 \times 2} = \frac{4}{54}$
Term independent of $x = \frac{21}{54} - \frac{4}{54} = \frac{21 - 4}{54} = \frac{17}{54}$.
The term independent of $x$ in the expansion of $(1 + x + 2x^3) \left( \frac{3}{2} x^2 - \frac{1}{3x} \right)^9$ is $\frac{17}{54}$.
The final answer is $\boxed{\frac{17}{54}}$.
Question 18 to 24 (Multiple Choice Questions)
Choose the correct answer from the given options in each of the Exercises 18 to 24 (M.C.Q.).
Question 18. The total number of terms in the expansion of (x + a)100 + (x – a)100 after simplification is
(A) 50
(B) 202
(C) 51
(D) none of these
Answer:
Given:
The expression $(x + a)^{100} + (x – a)^{100}$.
To Find:
The total number of terms after simplification.
Solution:
Let the given expression be $E$.
$E = (x + a)^{100} + (x – a)^{100}$
We use the binomial expansions for $(A+B)^n$ and $(A-B)^n$:
$(A+B)^n = \binom{n}{0}A^n B^0 + \binom{n}{1}A^{n-1}B^1 + \binom{n}{2}A^{n-2}B^2 + \dots + \binom{n}{n}A^0 B^n$
$(A-B)^n = \binom{n}{0}A^n B^0 - \binom{n}{1}A^{n-1}B^1 + \binom{n}{2}A^{n-2}B^2 - \dots + (-1)^n \binom{n}{n}A^0 B^n$
Adding the two expansions:
$(A+B)^n + (A-B)^n = 2 \left[ \binom{n}{0}A^n B^0 + \binom{n}{2}A^{n-2}B^2 + \binom{n}{4}A^{n-4}B^4 + \dots \right]$
The terms that remain are those with even indices in the binomial coefficient (starting from index 0), multiplied by 2.
In our case, $A=x$, $B=a$, and $n=100$. So the expression is:
$(x+a)^{100} + (x-a)^{100} = 2 \left[ \binom{100}{0}x^{100}a^0 + \binom{100}{2}x^{98}a^2 + \binom{100}{4}x^{96}a^4 + \dots + \binom{100}{100}x^0a^{100} \right]$
$(x+a)^{100} + (x-a)^{100} = 2 \left[ \binom{100}{0}x^{100} + \binom{100}{2}x^{98}a^2 + \binom{100}{4}x^{96}a^4 + \dots + \binom{100}{100}a^{100} \right]$
The powers of $a$ in the remaining terms are $0, 2, 4, \dots, 100$. Since these powers of $a$ are all distinct even numbers, the terms $x^{100-k}a^k$ for $k \in \{0, 2, 4, \dots, 100\}$ are distinct terms and cannot be combined further by collecting like terms (assuming $x \neq 0$ and $a \neq 0$).
The number of terms in the simplified expansion is equal to the number of even integers from 0 to 100, inclusive.
To find the number of even integers from 0 to 100, we can list them or use a formula. The numbers are $0, 2, 4, \dots, 100$. Let the position be $m$, such that the $m^{\text{th}}$ term is $2(m-1)$. $100 = 2(m-1) \implies 50 = m-1 \implies m=51$. Alternatively, the terms are of the form $2k$, where $k=0, 1, 2, \dots, 50$. The number of values of $k$ is $50 - 0 + 1 = 51$.
Thus, there are 51 terms in the simplified expansion.
The total number of terms in the expansion of $(x + a)^{100} + (x – a)^{100}$ after simplification is 51.
The correct option is (C) 51.
Question 19. Given the integers r > 1, n > 2, and coefficients of (3r)th and (r + 2)nd terms in the binomial expansion of (1 + x)2n are equal, then
(A) n = 2r
(B) n = 3r
(C) n = 2r + 1
(D) none of these
Answer:
Given:
Integers $r > 1$ and $n > 2$.
The expansion is $(1 + x)^{2n}$.
The coefficients of the $(3r)^{\text{th}}$ term and the $(r + 2)^{\text{nd}}$ term are equal.
To Find:
The relationship between $n$ and $r$ from the given options.
Solution:
The expansion is $(1+x)^{2n}$, so the power of the binomial is $N=2n$.
The coefficient of the $m^{\text{th}}$ term in the expansion of $(1+x)^N$ is $\binom{N}{m-1}$.
The coefficient of the $(3r)^{\text{th}}$ term is $\binom{2n}{(3r)-1} = \binom{2n}{3r-1}$.
The coefficient of the $(r+2)^{\text{nd}}$ term is $\binom{2n}{(r+2)-1} = \binom{2n}{r+1}$.
We are given that these coefficients are equal:
$\binom{2n}{3r-1} = \binom{2n}{r+1}$
For the binomial coefficients $\binom{N}{a}$ and $\binom{N}{b}$ to be equal, there are two possibilities:
1. $a = b$
2. $a + b = N$
In our case, $N=2n$, $a=3r-1$, and $b=r+1$.
Case 1: $3r - 1 = r + 1$
$3r - r = 1 + 1$
$2r = 2$
$r = 1$
However, the problem states that $r > 1$. Thus, this case is not possible under the given conditions.
Case 2: $(3r - 1) + (r + 1) = 2n$
$3r - 1 + r + 1 = 2n$
$4r = 2n$
Divide both sides by 2:
$2r = n$
$n = 2r$
We must check if this relationship is consistent with the given conditions $r > 1$ and $n > 2$.
If $r > 1$, then $2r > 2$. Since $n=2r$, we have $n > 2$. This is consistent with the condition $n > 2$.
Also, for the binomial coefficients to be well-defined, the indices must be non-negative integers less than or equal to $N=2n$.
$3r-1 \ge 0 \implies 3r \ge 1 \implies r \ge 1/3$. Given $r>1$, this is satisfied.
$r+1 \ge 0 \implies r \ge -1$. Given $r>1$, this is satisfied.
$3r-1 \le 2n$. Substitute $n=2r$: $3r-1 \le 2(2r) \implies 3r-1 \le 4r \implies -1 \le r$. Given $r>1$, this is satisfied.
$r+1 \le 2n$. Substitute $n=2r$: $r+1 \le 2(2r) \implies r+1 \le 4r \implies 1 \le 3r \implies r \ge 1/3$. Given $r>1$, this is satisfied.
The derived relationship $n=2r$ is consistent with all given conditions.
The relationship between $n$ and $r$ is $n=2r$.
The correct option is (A) n = 2r.
Question 20. The two successive terms in the expansion of (1 + x)24 whose coefficients are in the ratio 1:4 are
(A) 3rd and 4th
(B) 4th and 5th
(C) 5th and 6th
(D) 6th and 7th
[Hint : $\frac{^{24}C_r}{^{24}C_{r+1}} = \frac{1}{4}$
$⇒ \frac{r + 1}{24 − r} = \frac{1}{4}$
⇒ 4r + 4 = 24 − 4
⇒ r = 4 ]
Answer:
Given:
The expansion is $(1 + x)^{24}$.
The ratio of the coefficients of two successive terms is 1:4.
To Find:
The positions of these two successive terms.
Solution:
The expansion is $(1+x)^{24}$, so $n=24$.
The coefficient of the $(k+1)^{\text{th}}$ term in the expansion of $(1+x)^n$ is $\binom{n}{k}$.
Let the two successive terms be the $(r+1)^{\text{th}}$ term and the $(r+2)^{\text{th}}$ term.
The coefficient of the $(r+1)^{\text{th}}$ term is $\binom{24}{r}$.
The coefficient of the $(r+2)^{\text{th}}$ term is $\binom{24}{r+1}$.
We are given that the ratio of their coefficients is 1:4.
There are two possibilities for the order of the ratio:
Possibility 1: $\frac{\text{Coefficient of }(r+1)^{\text{th}} \text{ term}}{\text{Coefficient of }(r+2)^{\text{th}} \text{ term}} = \frac{1}{4}$
$\frac{\binom{24}{r}}{\binom{24}{r+1}} = \frac{1}{4}$
... (i)
Possibility 2: $\frac{\text{Coefficient of }(r+2)^{\text{th}} \text{ term}}{\text{Coefficient of }(r+1)^{\text{th}} \text{ term}} = \frac{1}{4}$
$\frac{\binom{24}{r+1}}{\binom{24}{r}} = \frac{1}{4}$
... (ii)
We know the property of binomial coefficients: $\frac{\binom{n}{k}}{\binom{n}{k+1}} = \frac{k+1}{n-k}$.
Using this property for Possibility 1, with $n=24$ and $k=r$:
$\frac{\binom{24}{r}}{\binom{24}{r+1}} = \frac{r+1}{24-r}$
So, $\frac{r+1}{24-r} = \frac{1}{4}$
Cross-multiply:
$4(r+1) = 1(24-r)$
$4r + 4 = 24 - r$
$4r + r = 24 - 4$
$5r = 20$
$r = \frac{20}{5}$
$r = 4$
If $r=4$, the successive terms are the $(r+1)^{\text{th}} = 5^{\text{th}}$ term and the $(r+2)^{\text{th}} = 6^{\text{th}}$ term.
Their coefficients are $\binom{24}{4}$ and $\binom{24}{5}$. The ratio $\frac{\binom{24}{4}}{\binom{24}{5}} = \frac{4+1}{24-4} = \frac{5}{20} = \frac{1}{4}$. This matches the given ratio 1:4.
Let's check Possibility 2, where the ratio is reversed:
$\frac{\binom{24}{r+1}}{\binom{24}{r}} = \frac{24-(r+1)+1}{r+1} = \frac{24-r-1+1}{r+1} = \frac{24-r}{r+1}$
So, $\frac{24-r}{r+1} = \frac{1}{4}$
Cross-multiply:
$4(24-r) = 1(r+1)$
$96 - 4r = r + 1$
$96 - 1 = r + 4r$
$95 = 5r$
$r = \frac{95}{5}$
$r = 19$
If $r=19$, the successive terms are the $(r+1)^{\text{th}} = 20^{\text{th}}$ term and the $(r+2)^{\text{th}} = 21^{\text{st}}$ term.
Their coefficients are $\binom{24}{19}$ and $\binom{24}{20}$. The ratio $\frac{\binom{24}{19}}{\binom{24}{20}} = \frac{19+1}{24-19} = \frac{20}{5} = 4$. The ratio $\frac{\binom{24}{20}}{\binom{24}{19}} = \frac{5}{20} = \frac{1}{4}$.
So, if the ratio of the 20th term's coefficient to the 19th term's coefficient is 1:4, then $r=19$. If the ratio of the 19th term's coefficient to the 20th term's coefficient is 1:4, then $r=4$.
The phrasing "two successive terms ... whose coefficients are in the ratio 1:4" usually implies the first term mentioned in the ratio is the earlier term in the expansion and the second term mentioned is the later term. So the $(r+1)^{\text{th}}$ term's coefficient has the ratio 1 to the $(r+2)^{\text{th}}$ term's coefficient which has the ratio 4.
Coefficient of $(r+1)^{\text{th}}$ : Coefficient of $(r+2)^{\text{th}} = 1 : 4$.
This corresponds to the case where $r=4$. The terms are the 5th and 6th terms.
The two successive terms are the 5th and 6th terms.
The correct option is (C) 5th and 6th.
Question 21. The coefficient of xn in the expansion of (1 + x)2n and (1 + x)2n – 1 are in the ratio.
(A) 1 : 2
(B) 1 : 3
(C) 3 : 1
(D) 2 : 1
[Hint : 2nCn : 2n – 1Cn]
Answer:
Given:
Two expansions: $(1 + x)^{2n}$ and $(1 + x)^{2n – 1}$.
To Find:
The ratio of the coefficient of $x^n$ in the first expansion to the coefficient of $x^n$ in the second expansion.
Solution:
The coefficient of $x^k$ in the expansion of $(1+x)^N$ is $\binom{N}{k}$.
For the expansion $(1+x)^{2n}$, the power is $N=2n$. The coefficient of $x^n$ is obtained by setting $k=n$.
Coefficient of $x^n$ in $(1+x)^{2n}$ is $\binom{2n}{n}$.
For the expansion $(1+x)^{2n-1}$, the power is $N=2n-1$. The coefficient of $x^n$ is obtained by setting $k=n$.
Coefficient of $x^n$ in $(1+x)^{2n-1}$ is $\binom{2n-1}{n}$.
We need to find the ratio of these two coefficients: $\frac{\binom{2n}{n}}{\binom{2n-1}{n}}$.
Write out the binomial coefficients using factorials:
$\binom{2n}{n} = \frac{(2n)!}{n!(2n-n)!} = \frac{(2n)!}{n!n!}$
$\binom{2n-1}{n} = \frac{(2n-1)!}{n!((2n-1)-n)!} = \frac{(2n-1)!}{n!(n-1)!}$
Now, form the ratio:
Ratio $= \frac{\frac{(2n)!}{n!n!}}{\frac{(2n-1)!}{n!(n-1)!}}$
Ratio $= \frac{(2n)!}{n!n!} \times \frac{n!(n-1)!}{(2n-1)!}$
Ratio $= \frac{(2n)!}{(2n-1)!} \times \frac{n!(n-1)!}{n!n!}$
Simplify the factorial terms:
$\frac{(2n)!}{(2n-1)!} = \frac{2n \times (2n-1)!}{(2n-1)!} = 2n$
$\frac{n!(n-1)!}{n!n!} = \frac{n!}{n!} \times \frac{(n-1)!}{n!} = 1 \times \frac{(n-1)!}{n \times (n-1)!} = \frac{1}{n}$
Substitute these simplified terms back into the ratio:
Ratio $= 2n \times \frac{1}{n}$
Ratio $= \frac{2n}{n} = 2$
The ratio is 2:1.
The ratio of the coefficient of $x^n$ in the expansion of $(1 + x)^{2n}$ and $(1 + x)^{2n – 1}$ is 2:1.
The correct option is (D) 2 : 1.
Question 22. If the coefficients of 2nd, 3rd and the 4th terms in the expansion of (1 + x)n are in A.P., then value of n is
(A) 2
(B) 7
(c) 11
(D) 14
[Hint: 2 nC2 = nC1 + nC3
⇒ n2 – 9n + 14 = 0
⇒ n = 2 or 7]
Answer:
Given: Coefficients of 2nd, 3rd, 4th terms of $(1+x)^n$ are in A.P.
To Find: Value of $n$.
Solution:
The coefficients of the 2nd, 3rd, and 4th terms are $\binom{n}{1}$, $\binom{n}{2}$, and $\binom{n}{3}$ respectively.
Since they are in A.P., we have:
$2 \binom{n}{2} = \binom{n}{1} + \binom{n}{3}$
Substitute the binomial coefficient formulas:
$2 \cdot \frac{n(n-1)}{2} = n + \frac{n(n-1)(n-2)}{6}$
$n(n-1) = n + \frac{n(n-1)(n-2)}{6}$
Assuming $n \ge 3$ (for the 4th term to exist), we can divide by $n$:
$n-1 = 1 + \frac{(n-1)(n-2)}{6}$
$6(n-1) = 6 + (n-1)(n-2)$
$6n - 6 = 6 + (n^2 - 3n + 2)$
$6n - 6 = n^2 - 3n + 8$
$n^2 - 9n + 14 = 0$
Factor the quadratic equation:
$(n-2)(n-7) = 0$
The possible values for $n$ are $n=2$ or $n=7$.
If $n=2$, the coefficients are $\binom{2}{1}=2$, $\binom{2}{2}=1$, $\binom{2}{3}=0$. These are 2, 1, 0, which are in A.P.
If $n=7$, the coefficients are $\binom{7}{1}=7$, $\binom{7}{2}=21$, $\binom{7}{3}=35$. These are 7, 21, 35, which are in A.P. ($2 \times 21 = 7 + 35$).
Both $n=2$ and $n=7$ are valid solutions. Among the given options, 7 is listed.
The value of $n$ is 7.
The correct option is (B) 7.
Question 23. If A and B are coefficient of xn in the expansions of (1 + x)2n and (1 + x)2n–1 respectively, then $\frac{A}{B}$ equals
(A) 1
(B) 2
(C) $\frac{1}{2}$
(D) $\frac{1}{n}$
[Hint : $\frac{A}{B} = \frac{^{2n}C_n}{^{2n−1}C_n} = 2$]
Answer:
Given:
$A$ = coefficient of $x^n$ in $(1 + x)^{2n}$.
$B$ = coefficient of $x^n$ in $(1 + x)^{2n-1}$.
To Find:
The value of $\frac{A}{B}$.
Solution:
The coefficient of $x^k$ in the expansion of $(1+x)^N$ is $\binom{N}{k}$.
For the expansion $(1+x)^{2n}$, $N=2n$. The coefficient of $x^n$ is $A = \binom{2n}{n}$.
For the expansion $(1+x)^{2n-1}$, $N=2n-1$. The coefficient of $x^n$ is $B = \binom{2n-1}{n}$.
We need to find the ratio $\frac{A}{B} = \frac{\binom{2n}{n}}{\binom{2n-1}{n}}$.
Write out the binomial coefficients using factorials:
$A = \binom{2n}{n} = \frac{(2n)!}{n!(2n-n)!} = \frac{(2n)!}{n!n!}$
$B = \binom{2n-1}{n} = \frac{(2n-1)!}{n!((2n-1)-n)!} = \frac{(2n-1)!}{n!(n-1)!}$
Now, form the ratio $\frac{A}{B}$:
$\frac{A}{B} = \frac{\frac{(2n)!}{n!n!}}{\frac{(2n-1)!}{n!(n-1)!}}$
$\frac{A}{B} = \frac{(2n)!}{n!n!} \times \frac{n!(n-1)!}{(2n-1)!}$
$\frac{A}{B} = \frac{(2n)!}{(2n-1)!} \times \frac{n!(n-1)!}{n!n!}$
Simplify the factorial terms:
$\frac{(2n)!}{(2n-1)!} = \frac{2n \times (2n-1)!}{(2n-1)!} = 2n$
$\frac{n!(n-1)!}{n!n!} = \frac{n!}{n!} \times \frac{(n-1)!}{n!} = 1 \times \frac{(n-1)!}{n \times (n-1)!} = \frac{1}{n}$
Substitute these simplified terms back into the ratio $\frac{A}{B}$:
$\frac{A}{B} = 2n \times \frac{1}{n}$
$\frac{A}{B} = \frac{2n}{n} = 2$
The value of $\frac{A}{B}$ is 2.
The correct option is (B) 2.
Question 24. If the middle term of $\left( \frac{1}{x} + x \sin x \right)^{10}$ is equal to $7\frac{7}{8}$ , , then value of x is
(A) $2nπ + \frac{π}{6}$
(B) $nπ + \frac{π}{6}$
(C) $nπ + (-1)^n \frac{π}{6}$
(D) $nπ + (-1)^n \frac{π}{3}$
[Hint: $T_6 =\; ^{10}C_5 \frac{1}{x^5} . x^5 \sin^5 x = \frac{63}{8}$
$⇒ \sin^5 x = \frac{1}{2^5}$
$⇒ \sin x = \frac{1}{2}$
$⇒ x = nπ + (−1)^n \frac{π}{6}$]
Answer:
Given:
The expansion is $\left( \frac{1}{x} + x \sin x \right)^{10}$.
The value of the middle term is $7\frac{7}{8}$.
To Find:
The value of $x$.
Solution:
The given expansion is of the form $(a+b)^n$, where $a = \frac{1}{x}$, $b = x \sin x$, and $n = 10$.
The total number of terms in the expansion is $n+1 = 10+1 = 11$.
Since the number of terms (11) is odd, there is one middle term.
The position of the middle term is $\frac{n}{2} + 1$ when $n$ is even.
Middle term position = $\frac{10}{2} + 1 = 5 + 1 = 6^{\text{th}}$ term.
We need to find the 6th term ($T_6$) of the expansion.
The general term, $T_{k+1}$, in the expansion of $(a+b)^n$ is given by $T_{k+1} = \binom{n}{k} a^{n-k} b^k$.
For the 6th term, $k+1=6$, so $k=5$.
Substitute the values of $n=10$, $k=5$, $a=\frac{1}{x}$, and $b=x \sin x$ into the general term formula:
$T_6 = \binom{10}{5} \left(\frac{1}{x}\right)^{10-5} (x \sin x)^5$
$T_6 = \binom{10}{5} \left(\frac{1}{x}\right)^5 x^5 (\sin x)^5$
$T_6 = \binom{10}{5} \frac{1}{x^5} x^5 \sin^5 x$
Assuming $x \neq 0$, we can cancel $x^5$:
$T_6 = \binom{10}{5} \sin^5 x$
Calculate the binomial coefficient $\binom{10}{5}$:
$\binom{10}{5} = \frac{10!}{5!5!} = \frac{10 \times 9 \times 8 \times 7 \times 6}{5 \times 4 \times 3 \times 2 \times 1} = 252$
So, $T_6 = 252 \sin^5 x$.
We are given that the middle term ($T_6$) is equal to $7\frac{7}{8}$. Convert this mixed number to an improper fraction:
$7\frac{7}{8} = \frac{7 \times 8 + 7}{8} = \frac{56 + 7}{8} = \frac{63}{8}$
Set the expression for $T_6$ equal to the given value:
$252 \sin^5 x = \frac{63}{8}$
... (i)
Solve for $\sin^5 x$:
$\sin^5 x = \frac{63}{8 \times 252}$
Simplify the fraction. Notice that $252 = 4 \times 63$.
$\sin^5 x = \frac{\cancel{63}}{8 \times 4 \times \cancel{63}} = \frac{1}{8 \times 4} = \frac{1}{32}$
We have $\sin^5 x = \frac{1}{32}$. Take the 5th root of both sides:
$\sqrt[5]{\sin^5 x} = \sqrt[5]{\frac{1}{32}}$
$\sin x = \frac{\sqrt[5]{1}}{\sqrt[5]{32}} = \frac{1}{2}$
Now we need to find the general solution for the trigonometric equation $\sin x = \frac{1}{2}$.
The principal value of $x$ for which $\sin x = \frac{1}{2}$ in the interval $[-\frac{\pi}{2}, \frac{\pi}{2}]$ is $\frac{\pi}{6}$.
The general solution for $\sin x = \sin \alpha$ is given by $x = n\pi + (-1)^n \alpha$, where $n$ is an integer ($n \in \mathbb{Z}$).
Using $\alpha = \frac{\pi}{6}$, the general solution for $\sin x = \frac{1}{2}$ is:
$x = n\pi + (-1)^n \frac{\pi}{6}$, where $n \in \mathbb{Z}$.
Comparing this general solution with the given options, we see that Option (C) matches this form.
The value of $x$ is $n\pi + (-1)^n \frac{\pi}{6}$, where $n$ is an integer.
The correct option is (C) $nπ + (-1)^n \frac{π}{6}$.
Question 25 to 33 (Fill in the Blanks)
Fill in the blanks in Exercises 25 to 33.
Question 25. The largest coefficient in the expansion of (1 + x)30 is _______________ .
Answer:
Given:
The expansion is $(1 + x)^{30}$.
To Find:
The largest coefficient in the expansion.
Solution:
The expansion is of the form $(1+x)^n$, where $n=30$.
The coefficients in the expansion of $(1+x)^n$ are $\binom{n}{k}$ for $k=0, 1, \dots, n$.
The binomial coefficients $\binom{n}{k}$ are symmetric, i.e., $\binom{n}{k} = \binom{n}{n-k}$.
When $n$ is even, the largest coefficient occurs at the middle term(s). In the case of $(1+x)^n$ with $n$ even, there is one middle term. The middle index is $\frac{n}{2}$.
Here, $n=30$, which is even. The middle index is $\frac{30}{2} = 15$.
The largest coefficient is $\binom{30}{15}$.
The largest coefficient in the expansion of $(1 + x)^{30}$ is $\binom{30}{15}$.
The final answer is $\boxed{\binom{30}{15}}$.
Question 26. The number of terms in the expansion of (x + y + z)n ________________ .
[Hint: (x + y + z)n = [x + (y + z)]n ]
Answer:
Given:
The expansion is $(x + y + z)^n$.
To Find:
The number of terms in the expansion.
Solution:
The expansion of a multinomial $(a_1 + a_2 + \dots + a_m)^n$ has terms of the form $C \cdot a_1^{n_1} a_2^{n_2} \dots a_m^{n_m}$, where $n_1, n_2, \dots, n_m$ are non-negative integers such that $n_1 + n_2 + \dots + n_m = n$, and $C$ is the multinomial coefficient.
Each distinct combination of exponents $(n_1, n_2, \dots, n_m)$ corresponds to a distinct term in the expansion.
In this case, we have a trinomial $(x+y+z)^n$, so $m=3$. The terms are of the form $C \cdot x^{n_1} y^{n_2} z^{n_3}$, where $n_1, n_2, n_3$ are non-negative integers such that $n_1 + n_2 + n_3 = n$.
The number of distinct terms is equal to the number of distinct non-negative integer solutions to the equation $n_1 + n_2 + n_3 = n$.
This can be solved using stars and bars. We have $n$ "stars" to distribute among 3 "bins" (for $n_1, n_2, n_3$), requiring $m-1 = 3-1 = 2$ "bars". The number of arrangements of $n$ stars and 2 bars is the number of ways to choose the positions of the 2 bars out of $n+2$ total positions, which is $\binom{n+2}{2}$.
Number of terms $= \binom{n+2}{2} = \frac{(n+2)!}{2!(n+2-2)!} = \frac{(n+2)!}{2!n!} = \frac{(n+2)(n+1)\cancel{n!}}{2 \times 1 \times \cancel{n!}} = \frac{(n+1)(n+2)}{2}$.
The number of terms in the expansion of $(x + y + z)^n$ is $\frac{(n+1)(n+2)}{2}$.
The final answer is $\boxed{\frac{(n+1)(n+2)}{2}}$.
Question 27. In the expansion of $\left( x^2 - \frac{1}{x^2} \right)^{16}$ , the value of constant term is __________ .
Answer:
Given: Expansion $\left( x^2 - \frac{1}{x^2} \right)^{16}$.
To Find: Constant term (independent of $x$).
Solution:
The general term $T_{k+1}$ in the expansion of $(a+b)^n$ is $\binom{n}{k} a^{n-k} b^k$.
Here $a=x^2$, $b=-\frac{1}{x^2}= -x^{-2}$, $n=16$.
$T_{k+1} = \binom{16}{k} (x^2)^{16-k} (-x^{-2})^k = \binom{16}{k} x^{32-2k} (-1)^k x^{-2k} = \binom{16}{k} (-1)^k x^{32-4k}$.
For the term independent of $x$, the power of $x$ must be 0.
$32 - 4k = 0 \implies 4k = 32 \implies k = 8$.
The constant term is the value of $T_{k+1}$ when $k=8$ (excluding $x^0$).
Constant term $= \binom{16}{8} (-1)^8 = \binom{16}{8} \times 1 = \binom{16}{8}$.
$\binom{16}{8} = \frac{16!}{8!8!} = \frac{16 \times 15 \times 14 \times 13 \times 12 \times 11 \times 10 \times 9}{8 \times 7 \times 6 \times 5 \times 4 \times 3 \times 2 \times 1}$
$\binom{16}{8} = \frac{\cancel{16}^2 \times \cancel{15}^1 \times \cancel{14}^1 \times 13 \times \cancel{12}^1 \times 11 \times \cancel{10}^1 \times 9}{\cancel{8}_1 \times \cancel{7}_1 \times \cancel{6}_1 \times \cancel{5}_1 \times \cancel{4}_1 \times \cancel{3}_1 \times \cancel{2}_1 \times 1}$ (Careful cancellation: $8 \times 2 = 16$, $5 \times 3 = 15$, $7 \times 1 = 7$, $6$, $4$, $1$ remain)
$\frac{16 \times 15 \times 14 \times 13 \times 12 \times 11 \times 10 \times 9}{8 \times 7 \times 6 \times 5 \times 4 \times 3 \times 2 \times 1} = \frac{(8 \times 2) \times (5 \times 3) \times (7 \times 2) \times 13 \times (6 \times 2) \times 11 \times (5 \times 2) \times (3 \times 3)}{8 \times 7 \times 6 \times 5 \times 4 \times 3 \times 2 \times 1}$
Cancel common factors: $\frac{\cancel{8} \times \cancel{2} \times \cancel{5} \times \cancel{3} \times \cancel{7} \times \cancel{2} \times 13 \times \cancel{6} \times \cancel{2} \times 11 \times \cancel{5} \times \cancel{2} \times \cancel{3} \times 3}{\cancel{8} \times \cancel{7} \times \cancel{6} \times \cancel{5} \times \cancel{4} \times \cancel{3} \times \cancel{2} \times 1}$
= $\frac{2 \times 13 \times 2 \times 11 \times 2 \times 5 \times 3}{4 \times 1}$ (Using $16/(8 \times 2) \rightarrow 1$, $15/(5 \times 3) \rightarrow 1$, $14/7 \rightarrow 2$, $12/6 \rightarrow 2$, $10/(?)$, $9/(?)$ )
Let's do the calculation $13 \times 11 \times \frac{16 \times 15 \times 14 \times 12 \times 10 \times 9}{8 \times 7 \times 6 \times 5 \times 4 \times 3 \times 2 \times 1}$.
$\binom{16}{8} = \frac{16 \times 15 \times 14 \times 13 \times 12 \times 11 \times 10 \times 9}{40320} = 12870$.
The value of the constant term is 12870.
The final answer is $\boxed{12870}$.
Question 28. If the seventh terms from the beginning and the end in the expansion of $\left( \sqrt[3]{2} + \frac{1}{\sqrt[3]{3}} \right)^n$ , are equal, then n equals _________________ .
[Hint : T7 = Tn − 7 + 2
$⇒\; ^nC_6 \left( 2^{\frac{1}{3}} \right)^{n - 6} \left( \frac{1}{3^{\frac{1}{3}}} \right)^6 =\; ^nC_{n − 6} \left( 2^{\frac{1}{3}} \right)^{6} \left( \frac{1}{3^{\frac{1}{3}}} \right)^{n-6}$
$⇒ \left( 2^{\frac{1}{3}} \right)^{n-12} = \left( \frac{1}{3^{\frac{1}{3}}} \right)^{n-12}$
⇒ only problem when n − 12 = 0
⇒ n = 12.]
Answer:
Given:
The expansion is $\left( \sqrt[3]{2} + \frac{1}{\sqrt[3]{3}} \right)^n$.
The seventh term from the beginning and the seventh term from the end are equal.
To Find:
The value of $n$.
Solution:
The given expansion is of the form $(a+b)^n$, where $a = \sqrt[3]{2} = 2^{1/3}$ and $b = \frac{1}{\sqrt[3]{3}} = 3^{-1/3}$.
The general term, $T_{k+1}$, in the expansion of $(a+b)^n$ is given by $T_{k+1} = \binom{n}{k} a^{n-k} b^k$.
Seventh term from the beginning ($T_7$):
For the 7th term from the beginning, set $k+1=7$, so $k=6$.
$T_7 = \binom{n}{6} (2^{1/3})^{n-6} (3^{-1/3})^6 = \binom{n}{6} 2^{(n-6)/3} 3^{-6/3} = \binom{n}{6} 2^{(n-6)/3} 3^{-2}$.
Seventh term from the end:
The total number of terms is $n+1$. The $r^{\text{th}}$ term from the end is the $( (n+1) - r + 1 )^{\text{th}}$ term from the beginning. For $r=7$, the term is the $(n+1 - 7 + 1) = (n-5)^{\text{th}}$ term from the beginning.
The $(n-5)^{\text{th}}$ term from the beginning is $T_{n-5}$. For this term, $k+1 = n-5$, so $k=n-6$.
$T_{n-5} = \binom{n}{n-6} (2^{1/3})^{n-(n-6)} (3^{-1/3})^{n-6} = \binom{n}{n-6} (2^{1/3})^6 (3^{-1/3})^{n-6} = \binom{n}{n-6} 2^{6/3} 3^{-(n-6)/3} = \binom{n}{n-6} 2^2 3^{-(n-6)/3}$.
Using $\binom{n}{n-6} = \binom{n}{6}$, we have $T_{n-5} = \binom{n}{6} 2^2 3^{-(n-6)/3}$.
We are given that the seventh term from the beginning and the seventh term from the end are equal:
$T_7 = T_{n-5}$
(Given)
Substitute the expressions for $T_7$ and $T_{n-5}$ (assuming $n \ge 6$ for the 7th term to exist):
$\binom{n}{6} 2^{(n-6)/3} 3^{-2} = \binom{n}{6} 2^2 3^{-(n-6)/3}$
Since $n \ge 6$, $\binom{n}{6} \neq 0$, so we can divide both sides by $\binom{n}{6}$:
$2^{(n-6)/3} 3^{-2} = 2^2 3^{-(n-6)/3}$
Rearrange the terms to group powers of the same base:
$\frac{2^{(n-6)/3}}{2^2} = \frac{3^{-(n-6)/3}}{3^{-2}}$
$2^{(n-6)/3 - 2} = 3^{-(n-6)/3 - (-2)}$
$2^{\frac{n-6-6}{3}} = 3^{\frac{-(n-6)+6}{3}}$
$2^{\frac{n-12}{3}} = 3^{\frac{-n+6+6}{3}}$
$2^{\frac{n-12}{3}} = 3^{\frac{12-n}{3}}$
$2^{\frac{n-12}{3}} = 3^{-\frac{n-12}{3}}$
$2^{\frac{n-12}{3}} = \frac{1}{3^{\frac{n-12}{3}}}$
$2^{\frac{n-12}{3}} \cdot 3^{\frac{n-12}{3}} = 1$
$(2 \cdot 3)^{\frac{n-12}{3}} = 1$
$6^{\frac{n-12}{3}} = 1$
For an exponential term with a base other than 1 or -1 to be equal to 1, the exponent must be 0.
So, $\frac{n-12}{3} = 0$
$n - 12 = 0$
$n = 12$
Check if $n=12$ is valid. For the 7th term to exist, $n \ge 6$. $12 \ge 6$, so this is valid.
The value of $n$ is 12.
The final answer is $\boxed{12}$.
Question 29. The coefficient of a–6 b4 in the expansion of $\left( \frac{1}{a} − \frac{2b}{3} \right)^{10}$ is _________.
[Hint : $T_5 =\; ^{10}C_4 \left( \frac{1}{a} \right)^b \left( \frac{−2b}{3} \right)^4 = \frac{1120}{27} a^{−6} b^4$]
Answer:
Given:
The expansion is $\left( \frac{1}{a} − \frac{2b}{3} \right)^{10}$.
To Find:
The coefficient of $a^{-6} b^4$ in the expansion.
Solution:
The given expansion is of the form $(x+y)^n$, where $x = \frac{1}{a} = a^{-1}$, $y = -\frac{2b}{3} = -\frac{2}{3} b$, and $n = 10$.
The general term, $T_{k+1}$, in the expansion of $(x+y)^n$ is given by the formula:
$T_{k+1} = \binom{n}{k} x^{n-k} y^k$
Substitute the values of $n=10$, $x=a^{-1}$, and $y=-\frac{2}{3}b$ for the given expansion:
$T_{k+1} = \binom{10}{k} (a^{-1})^{10-k} \left(-\frac{2}{3} b\right)^k$
Simplify the terms involving $a$ and $b$, and the constant terms separately.
Constant part: $\binom{10}{k} \left(-\frac{2}{3}\right)^k$
Variable part: $(a^{-1})^{10-k} \cdot (b)^k = a^{-(10-k)} \cdot b^k = a^{-10+k} b^k$
The general term is $T_{k+1} = \binom{10}{k} \left(-\frac{2}{3}\right)^k a^{-10+k} b^k$.
We want to find the coefficient of $a^{-6} b^4$. By comparing the exponents of $a$ and $b$ in the general term with the target term $a^{-6} b^4$, we can find the value of $k$.
Exponent of $b$: $k = 4$
Let's check if this value of $k$ gives the correct exponent for $a$:
Exponent of $a$: $-10 + k = -10 + 4 = -6$.
This matches the required exponent of $a$. Since $k=4$ is a non-negative integer and $0 \leq 4 \leq 10$, this value of $k$ is valid.
The coefficient of $a^{-6} b^4$ is the constant part of $T_{k+1}$ when $k=4$.
Coefficient of $a^{-6} b^4 = \binom{10}{4} \left(-\frac{2}{3}\right)^4$
Calculate the binomial coefficient $\binom{10}{4}$:
$\binom{10}{4} = \frac{10!}{4!6!} = \frac{10 \times 9 \times 8 \times 7}{4 \times 3 \times 2 \times 1} = \frac{\cancel{10}^{5} \times \cancel{9}^{3} \times \cancel{8}^{1} \times 7}{\cancel{24}_{1}} = 5 \times 3 \times 1 \times 7 = 210$
Calculate $\left(-\frac{2}{3}\right)^4$:
$\left(-\frac{2}{3}\right)^4 = (-1)^4 \frac{2^4}{3^4} = 1 \cdot \frac{16}{81} = \frac{16}{81}$
Multiply the results:
Coefficient of $a^{-6} b^4 = 210 \times \frac{16}{81}$
Simplify the fraction. 210 is divisible by 3 (sum of digits = 3), 81 is divisible by 9 (sum of digits = 9). $210/3 = 70$, $81/3 = 27$.
Coefficient $= \frac{210}{81} \times 16 = \frac{\cancel{210}^{70}}{\cancel{81}_{27}} \times 16 = \frac{70 \times 16}{27}$
$70 \times 16 = 1120$.
Coefficient $= \frac{1120}{27}$.
The term $T_5$ in the hint is the general term $T_{k+1}$ with $k=4$. The hint provides the term as $\frac{1120}{27} a^{-6} b^4$, confirming our calculation of the coefficient and the variables.
The coefficient of $a^{-6} b^4$ in the expansion is $\frac{1120}{27}$.
The final answer is $\boxed{\frac{1120}{27}}$.
Question 30. Middle term in the expansion of (a3 + ba)28 is _________ .
Answer:
Given:
The expansion is $(a^3 + ba)^{28}$.
To Find:
The middle term in the expansion.
Solution:
The given expansion is of the form $(x+y)^n$, where $x = a^3$, $y = ba$, and $n = 28$.
The total number of terms in the expansion is $n+1 = 28+1 = 29$.
Since the number of terms (29) is odd, there is one middle term.
The position of the middle term is $\frac{n}{2} + 1$ when $n$ is even.
Middle term position = $\frac{28}{2} + 1 = 14 + 1 = 15^{\text{th}}$ term.
We need to find the 15th term ($T_{15}$) of the expansion.
The general term, $T_{k+1}$, in the expansion of $(x+y)^n$ is given by $T_{k+1} = \binom{n}{k} x^{n-k} y^k$.
For the 15th term, $k+1=15$, so $k=14$.
Substitute the values of $n=28$, $k=14$, $x=a^3$, and $y=ba$ into the general term formula:
$T_{15} = \binom{28}{14} (a^3)^{28-14} (ba)^{14}$
$T_{15} = \binom{28}{14} (a^3)^{14} (ba)^{14}$
$T_{15} = \binom{28}{14} a^{3 \times 14} b^{14} a^{14}$
$T_{15} = \binom{28}{14} a^{42} b^{14} a^{14}$
Combine the terms involving $a$:
$T_{15} = \binom{28}{14} a^{42+14} b^{14}$
$T_{15} = \binom{28}{14} a^{56} b^{14}$
The middle term is $\binom{28}{14} a^{56} b^{14}$. We do not need to calculate the numerical value of $\binom{28}{14}$ unless specified, as it is part of the term's expression.
The middle term in the expansion of $(a^3 + ba)^{28}$ is $\binom{28}{14} a^{56} b^{14}$.
The final answer is $\boxed{\binom{28}{14} a^{56} b^{14}}$.
Question 31. The ratio of the coefficients of xp and xq in the expansion of (1 + x)p + q is_________
[Hint: p + qCp = p + qCq]
Answer:
Given:
The expansion is $(1 + x)^{p + q}$.
$p$ and $q$ are positive integers (implied by the context of coefficients of $x^p$ and $x^q$ in this type of expansion). Let's assume they are non-negative integers as $x^0$ is possible.
To Find:
The ratio of the coefficient of $x^p$ to the coefficient of $x^q$ in the expansion.
Solution:
The expansion is of the form $(1+x)^n$, where $n = p+q$.
The coefficient of $x^k$ in the expansion of $(1+x)^n$ is $\binom{n}{k}$.
The coefficient of $x^p$ in the expansion of $(1 + x)^{p + q}$ is $\binom{p+q}{p}$.
The coefficient of $x^q$ in the expansion of $(1 + x)^{p + q}$ is $\binom{p+q}{q}$.
We need to find the ratio of these coefficients: $\frac{\text{Coefficient of } x^p}{\text{Coefficient of } x^q} = \frac{\binom{p+q}{p}}{\binom{p+q}{q}}$.
We know the symmetry property of binomial coefficients: $\binom{N}{K} = \binom{N}{N-K}$.
Here, $N=p+q$. Let's look at $\binom{p+q}{q}$ using this property:
$\binom{p+q}{q} = \binom{p+q}{(p+q) - q} = \binom{p+q}{p}$
So, the coefficient of $x^q$ is equal to the coefficient of $x^p$.
The ratio is $\frac{\binom{p+q}{p}}{\binom{p+q}{p}} = 1$, assuming $\binom{p+q}{p} \neq 0$. This is true for non-negative integers $p, q$ such that $p \le p+q$. If $p+q = 0$, the expansion is $(1+x)^0=1$. If $p=0, q=0$, the coefficient of $x^0$ is $\binom{0}{0}=1$. The ratio is $1/1=1$. If $p+q>0$, the coefficients are non-zero for $0 \le p, q \le p+q$.
The ratio of the coefficients is $1:1$, which is equal to 1.
The ratio of the coefficients of $x^p$ and $x^q$ in the expansion of $(1 + x)^{p + q}$ is 1.
The final answer is $\boxed{1}$.
Question 32. The position of the term independent of x in the expansion of $\left( \sqrt{\frac{x}{3}} + \frac{3}{2x^2} \right)^{10}$ is______.
Answer:
Given:
The expansion is $\left( \sqrt{\frac{x}{3}} + \frac{3}{2x^2} \right)^{10}$.
To Find:
The position (term number) of the term independent of $x$.
Solution:
The given expansion is of the form $(a+b)^n$, where $a = \sqrt{\frac{x}{3}} = \frac{x^{1/2}}{\sqrt{3}} = \frac{1}{\sqrt{3}} x^{1/2}$, $b = \frac{3}{2x^2} = \frac{3}{2} x^{-2}$, and $n = 10$.
The general term, $T_{k+1}$, in the expansion of $(a+b)^n$ is given by the formula:
$T_{k+1} = \binom{n}{k} a^{n-k} b^k$
Substitute the values of $n=10$, $a=\frac{1}{\sqrt{3}} x^{1/2}$, and $b=\frac{3}{2} x^{-2}$ for the given expansion:
$T_{k+1} = \binom{10}{k} \left(\frac{1}{\sqrt{3}} x^{1/2}\right)^{10-k} \left(\frac{3}{2} x^{-2}\right)^k$
Simplify the terms involving $x$ and the constant terms separately.
Constant part: $\binom{10}{k} \left(\frac{1}{\sqrt{3}}\right)^{10-k} \left(\frac{3}{2}\right)^k$
Variable part: $(x^{1/2})^{10-k} \cdot (x^{-2})^k = x^{(1/2)(10-k)} \cdot x^{-2k} = x^{\frac{10-k}{2}} \cdot x^{-2k} = x^{\frac{10-k}{2} - 2k} = x^{\frac{10-k-4k}{2}} = x^{\frac{10-5k}{2}}$
The general term is $T_{k+1} = \binom{10}{k} \left(\frac{1}{\sqrt{3}}\right)^{10-k} \left(\frac{3}{2}\right)^k x^{\frac{10-5k}{2}}$.
For the term independent of $x$, the power of $x$ must be zero. So, we set the exponent of $x$ equal to 0:
$\frac{10-5k}{2} = 0$
$10 - 5k = 0$
$10 = 5k$
$k = 2$
The index $k$ determines the position of the term. The term number is $k+1$.
Term number = $k+1 = 2+1 = 3^{\text{rd}}$ term.
Since $k=2$ is a non-negative integer and $0 \leq 2 \leq 10$, this value of $k$ is valid, and a term independent of $x$ exists at this position.
The position of the term independent of $x$ is the 3rd term.
The final answer is $\boxed{\text{3}^{\text{rd}} \text{ term}}$.
Question 33. If 2515 is divided by 13, the reminder is _________ .
Answer:
Given:
We need to find the remainder when $25^{15}$ is divided by 13.
To Find:
The remainder.
Solution:
We can use the concept of modular arithmetic or binomial expansion.
We want to find $25^{15} \pmod{13}$.
Notice that $25 = 2 \times 13 - 1 = 26 - 1$.
So, $25 \equiv -1 \pmod{13}$.
Now, raise both sides to the power of 15:
$25^{15} \equiv (-1)^{15} \pmod{13}$
Since 15 is an odd integer, $(-1)^{15} = -1$.
$25^{15} \equiv -1 \pmod{13}$
A remainder cannot be negative. To find the positive remainder, we add the modulus (13).
$-1 \equiv -1 + 13 \pmod{13}$
$-1 \equiv 12 \pmod{13}$
So, $25^{15} \equiv 12 \pmod{13}$.
The remainder when $25^{15}$ is divided by 13 is 12.
Alternate Solution (using Binomial Expansion):
We write 25 as a number related to 13. $25 = 26 - 1 = 2 \times 13 - 1$.
So, $25^{15} = (2 \times 13 - 1)^{15}$.
Let $x = 2 \times 13 = 26$ and $y = -1$. We expand $(x+y)^{15}$ using the binomial theorem:
$(26 - 1)^{15} = \binom{15}{0}(26)^{15}(-1)^0 + \binom{15}{1}(26)^{14}(-1)^1 + \binom{15}{2}(26)^{13}(-1)^2 + \dots + \binom{15}{14}(26)^1(-1)^{14} + \binom{15}{15}(26)^0(-1)^{15}$
$(26 - 1)^{15} = \binom{15}{0}26^{15} - \binom{15}{1}26^{14} + \binom{15}{2}26^{13} - \dots + \binom{15}{14}26 - \binom{15}{15}$
$(26 - 1)^{15} = \binom{15}{0}26^{15} - \binom{15}{1}26^{14} + \binom{15}{2}26^{13} - \dots + 15 \times 26 - 1$
Notice that every term in the expansion, except the last one $(-\binom{15}{15})$, has a factor of 26. Since $26$ is a multiple of 13, all these terms are divisible by 13.
$26^{15} - \binom{15}{1}26^{14} + \dots + \binom{15}{14}26 = \text{a multiple of } 26 = \text{a multiple of } 13$.
So, $(26 - 1)^{15} = (\text{a multiple of } 13) - \binom{15}{15}$
$(26 - 1)^{15} = (\text{a multiple of } 13) - 1$
Let $25^{15} = 13q + r$, where $q$ is the quotient and $r$ is the remainder, $0 \leq r < 13$.
$25^{15} = (\text{a multiple of } 13) - 1$
This means $25^{15} = 13k - 1$ for some integer $k$.
To find the remainder when divided by 13, we can write $13k - 1 = 13k - 13 + 12 = 13(k-1) + 12$.
So, $25^{15} = 13 \times (\text{an integer}) + 12$.
The remainder is 12.
When $25^{15}$ is divided by 13, the remainder is 12.
The final answer is $\boxed{12}$.
Question 34 to 40 (True or False)
State which of the statement in Exercises 34 to 40 is True or False.
Question 34. The sum of the series $\sum\limits^{10}_{r=0}\; ^{20}C_r$ is $2^{19} + \frac{^{20}C_{10}}{2}$.
Answer:
Given:
The statement: The sum of the series $\sum\limits^{10}_{r=0}\; \binom{20}{r}$ is $2^{19} + \frac{\binom{20}{10}}{2}$.
To Determine:
Whether the given statement is True or False.
Solution:
The given series is $S = \sum\limits^{10}_{r=0}\; \binom{20}{r} = \binom{20}{0} + \binom{20}{1} + \binom{20}{2} + \dots + \binom{20}{10}$.
We know the binomial theorem identity: $\sum\limits^{n}_{r=0}\; \binom{n}{r} = \binom{n}{0} + \binom{n}{1} + \dots + \binom{n}{n} = 2^n$.
For $n=20$, the total sum of coefficients is $\sum\limits^{20}_{r=0}\; \binom{20}{r} = 2^{20}$.
This sum can be split into two parts:
$\sum\limits^{20}_{r=0}\; \binom{20}{r} = \left( \binom{20}{0} + \binom{20}{1} + \dots + \binom{20}{9} \right) + \binom{20}{10} + \left( \binom{20}{11} + \binom{20}{12} + \dots + \binom{20}{20} \right)$
Using the symmetry property $\binom{n}{r} = \binom{n}{n-r}$:
$\binom{20}{0} = \binom{20}{20}$
$\binom{20}{1} = \binom{20}{19}$
... environments.
$\binom{20}{9} = \binom{20}{11}$
So, the terms $\binom{20}{0}, \dots, \binom{20}{9}$ are equal to $\binom{20}{20}, \dots, \binom{20}{11}$ respectively.
Let $S' = \binom{20}{0} + \binom{20}{1} + \dots + \binom{20}{9}$.
The sum $\sum\limits^{20}_{r=0}\; \binom{20}{r}$ can be written as:
$2^{20} = (\binom{20}{0} + \dots + \binom{20}{9}) + \binom{20}{10} + (\binom{20}{11} + \dots + \binom{20}{20})$
$2^{20} = S' + \binom{20}{10} + S'$ (since $\binom{20}{r} = \binom{20}{20-r}$)
$2^{20} = 2S' + \binom{20}{10}$
$2S' = 2^{20} - \binom{20}{10}$
$S' = \frac{1}{2} \left( 2^{20} - \binom{20}{10} \right) = 2^{19} - \frac{\binom{20}{10}}{2}$
The given series is $S = \sum\limits^{10}_{r=0}\; \binom{20}{r} = (\binom{20}{0} + \dots + \binom{20}{9}) + \binom{20}{10} = S' + \binom{20}{10}$.
Substitute the expression for $S'$:
$S = \left( 2^{19} - \frac{\binom{20}{10}}{2} \right) + \binom{20}{10}$
$S = 2^{19} + \binom{20}{10} - \frac{\binom{20}{10}}{2}$
$S = 2^{19} + \frac{\binom{20}{10}}{2}$
This matches the expression given in the statement.
The statement is True.
The final answer is $\boxed{True}$.
Question 35. The expression 79 + 97 is divisible by 64.
[Hint: 79 + 97 = (1 + 8)7 – (1 – 8)9]
Answer:
Given:
The statement: The expression $7^9 + 9^7$ is divisible by 64.
To Determine:
Whether the given statement is True or False.
Solution:
Let the expression be $E = 7^9 + 9^7$.
We can write $7 = 8 - 1$ and $9 = 8 + 1$.
$E = (8 - 1)^9 + (8 + 1)^7$
Let's expand $(x-y)^n$ and $(x+y)^n$ where $x=8$ and $y=1$.
$(x-y)^n = \binom{n}{0}x^n - \binom{n}{1}x^{n-1}y + \binom{n}{2}x^{n-2}y^2 - \dots + (-1)^n \binom{n}{n}y^n$
$(x+y)^n = \binom{n}{0}x^n + \binom{n}{1}x^{n-1}y + \binom{n}{2}x^{n-2}y^2 + \dots + \binom{n}{n}y^n$
Expand $7^9 = (8-1)^9$. Here $n=9, x=8, y=1$.
$7^9 = \binom{9}{0}8^9 - \binom{9}{1}8^8 + \binom{9}{2}8^7 - \binom{9}{3}8^6 + \binom{9}{4}8^5 - \binom{9}{5}8^4 + \binom{9}{6}8^3 - \binom{9}{7}8^2 + \binom{9}{8}8^1 - \binom{9}{9}8^0$
Expand $9^7 = (8+1)^7$. Here $n=7, x=8, y=1$.
$9^7 = \binom{7}{0}8^7 + \binom{7}{1}8^6 + \binom{7}{2}8^5 + \binom{7}{3}8^4 + \binom{7}{4}8^3 + \binom{7}{5}8^2 + \binom{7}{6}8^1 + \binom{7}{7}8^0$
Now, consider the sum $E = 7^9 + 9^7$. We need to check if it is divisible by 64. $64 = 8^2$.
Look at the terms in the expansions. Any term with $8^k$ where $k \ge 2$ is divisible by $8^2 = 64$.
In the expansion of $7^9$: All terms from $\binom{9}{0}8^9$ down to $\binom{9}{7}8^2$ are divisible by 64. The remaining terms are $\binom{9}{8}8^1 - \binom{9}{9}8^0$.
$7^9 = (\text{a multiple of } 64) + \binom{9}{8}8 - \binom{9}{9}$
$\binom{9}{8} = 9$, $\binom{9}{9} = 1$.
$7^9 = (\text{a multiple of } 64) + 9 \times 8 - 1 = (\text{a multiple of } 64) + 72 - 1 = (\text{a multiple of } 64) + 71$.
We can write $71 = 64 + 7$.
$7^9 = (\text{a multiple of } 64) + 64 + 7 = (\text{a multiple of } 64) + 7$.
So, $7^9 \equiv 7 \pmod{64}$.
In the expansion of $9^7$: All terms from $\binom{7}{0}8^7$ down to $\binom{7}{2}8^5$ are divisible by 64. (No, terms up to $\binom{7}{2}8^5$ are divisible by $8^5$. We need $8^2$. Terms with $8^k$ where $k \ge 2$ are divisible by 64). All terms from $\binom{7}{0}8^7$ down to $\binom{7}{2}8^5$ (incorrect indices). Terms are $\binom{7}{0}8^7, \binom{7}{1}8^6, \binom{7}{2}8^5, \dots$.
All terms with $8^k$ where $k \ge 2$ are divisible by 64. These are terms from $k=2$ up to $k=7$. The terms from $\binom{7}{0}8^7$ down to $\binom{7}{2}8^5$ are divisible by 64. The remaining terms are $\binom{7}{5}8^2 + \binom{7}{6}8^1 + \binom{7}{7}8^0$.
No, the powers of 8 are $7, 6, 5, 4, 3, 2, 1, 0$. Terms with $8^k$ for $k \ge 2$ are divisible by 64. These are terms with $k=0, 1, 2, 3, 4, 5$ in the binomial expansion of $(8+1)^7 = \sum \binom{7}{k} 8^{7-k} 1^k$. The power of 8 is $7-k$. We need $7-k \ge 2 \implies k \le 5$. So terms for $k=0, 1, 2, 3, 4, 5$ are divisible by 64.
The remaining terms are when $k=6$ and $k=7$.
$9^7 = (\text{a multiple of } 64) + \binom{7}{6}8^1 + \binom{7}{7}8^0$
$\binom{7}{6} = 7$, $\binom{7}{7} = 1$.
$9^7 = (\text{a multiple of } 64) + 7 \times 8 + 1 = (\text{a multiple of } 64) + 56 + 1 = (\text{a multiple of } 64) + 57$.
So, $9^7 \equiv 57 \pmod{64}$.
Now, sum the remainders:
$7^9 + 9^7 \equiv 7 + 57 \pmod{64}$
$7^9 + 9^7 \equiv 64 \pmod{64}$
$7^9 + 9^7 \equiv 0 \pmod{64}$
Since the remainder is 0, the expression $7^9 + 9^7$ is divisible by 64.
The statement is True.
The final answer is $\boxed{True}$.
Question 36. The number of terms in the expansion of [(2x + y3)4]7 is 8
Answer:
Given:
The statement: The number of terms in the expansion of $[(2x + y^3)^4]^7$ is 8.
To Determine:
Whether the given statement is True or False.
Solution:
The given expression is $[(2x + y^3)^4]^7$.
Using the exponent rule $(a^m)^p = a^{mp}$, we can simplify the expression:
$[(2x + y^3)^4]^7 = (2x + y^3)^{4 \times 7} = (2x + y^3)^{28}$
This is a binomial expansion of the form $(A+B)^n$, where $A = 2x$, $B = y^3$, and $n = 28$.
The total number of terms in the expansion of $(A+B)^n$ is $n+1$.
In this case, the number of terms is $28 + 1 = 29$.
The statement claims that the number of terms is 8.
Since $29 \neq 8$, the statement is false.
The statement is False.
The final answer is $\boxed{False}$.
Question 37. The sum of coefficients of the two middle terms in the expansion of (1 + x)2n – 1 is equal to 2n – 1Cn.
Answer:
Given:
The statement: The sum of coefficients of the two middle terms in the expansion of $(1 + x)^{2n – 1}$ is equal to $\binom{2n – 1}{n}$.
To Determine:
Whether the given statement is True or False.
Solution:
The given expansion is $(1+x)^{2n-1}$. The power of the binomial is $N = 2n-1$.
The total number of terms in the expansion is $N+1 = (2n-1) + 1 = 2n$.
Since the number of terms ($2n$) is even (assuming $n \ge 1$), there are two middle terms.
The positions of the two middle terms are $\frac{2n}{2} = n^{\text{th}}$ term and $\frac{2n}{2} + 1 = (n+1)^{\text{th}}$ term.
The coefficient of the $m^{\text{th}}$ term in the expansion of $(1+x)^N$ is $\binom{N}{m-1}$.
The coefficient of the $n^{\text{th}}$ term is $\binom{2n-1}{n-1}$.
The coefficient of the $(n+1)^{\text{th}}$ term is $\binom{2n-1}{(n+1)-1} = \binom{2n-1}{n}$.
We need to find the sum of these two coefficients:
Sum of coefficients of middle terms = $\binom{2n-1}{n-1} + \binom{2n-1}{n}$.
Using Pascal's Identity: $\binom{N}{K} + \binom{N}{K+1} = \binom{N+1}{K+1}$.
Here, $N=2n-1$ and $K=n-1$. So $K+1 = n$.
$\binom{2n-1}{n-1} + \binom{2n-1}{n} = \binom{(2n-1)+1}{(n-1)+1} = \binom{2n}{n}$.
The sum of the coefficients of the two middle terms is $\binom{2n}{n}$.
The statement claims that the sum of coefficients of the two middle terms is equal to $\binom{2n-1}{n}$.
We found the sum is $\binom{2n}{n}$. We need to compare $\binom{2n}{n}$ with $\binom{2n-1}{n}$.
From Question 21, we found the ratio $\frac{\binom{2n}{n}}{\binom{2n-1}{n}} = 2$.
This means $\binom{2n}{n} = 2 \binom{2n-1}{n}$.
The sum of the coefficients of the middle terms is $2 \binom{2n-1}{n}$.
The statement claims the sum is $\binom{2n-1}{n}$. This is only true if $\binom{2n-1}{n} = 0$, which is not the case for $n \ge 1$.
For example, if $n=1$, the expansion is $(1+x)^{2(1)-1} = (1+x)^1 = 1+x$. The number of terms is 2. The middle terms are the 1st and 2nd terms. Coefficients are $\binom{1}{0}=1$ and $\binom{1}{1}=1$. Sum = $1+1=2$. The statement claims the sum is $\binom{2(1)-1}{1} = \binom{1}{1}=1$. $2 \neq 1$. Statement is false.
If $n=2$, the expansion is $(1+x)^{2(2)-1} = (1+x)^3 = 1+3x+3x^2+x^3$. The number of terms is 4. The middle terms are the 2nd and 3rd terms. Coefficients are $\binom{3}{1}=3$ and $\binom{3}{2}=3$. Sum = $3+3=6$. The statement claims the sum is $\binom{2(2)-1}{2} = \binom{3}{2}=3$. $6 \neq 3$. Statement is false.
The sum of the coefficients of the two middle terms is $\binom{2n}{n}$, which is equal to $2 \binom{2n-1}{n}$. The statement says the sum is $\binom{2n-1}{n}$.
The statement is False.
The statement is False.
The final answer is $\boxed{False}$.
Question 38. The last two digits of the numbers 3400 are 01.
Answer:
Given:
The statement: The last two digits of the number $3^{400}$ are 01.
To Determine:
Whether the given statement is True or False.
Solution:
Finding the last two digits of a number is equivalent to finding the remainder when the number is divided by 100. So we need to calculate $3^{400} \pmod{100}$.
We can use Euler's totient theorem. $\phi(100) = \phi(2^2 \cdot 5^2) = \phi(2^2) \phi(5^2) = (2^2 - 2^1)(5^2 - 5^1) = (4-2)(25-5) = 2 \times 20 = 40$.
Since $\text{gcd}(3, 100) = 1$, by Euler's theorem, $3^{\phi(100)} \equiv 1 \pmod{100}$.
$3^{40} \equiv 1 \pmod{100}$.
We want to find $3^{400} \pmod{100}$.
$3^{400} = 3^{40 \times 10} = (3^{40})^{10}$.
Since $3^{40} \equiv 1 \pmod{100}$, we can raise both sides to the power of 10:
$(3^{40})^{10} \equiv 1^{10} \pmod{100}$
$3^{400} \equiv 1 \pmod{100}$.
This means the remainder when $3^{400}$ is divided by 100 is 1. The remainder 1 corresponds to the last two digits being 01.
The statement is True.
The final answer is $\boxed{True}$.
Question 39. If the expansion of $\left( x - \frac{1}{x^2} \right)^{2n}$ contains a term independent of x, then n is a multiple of 2.
Answer:
Given:
The statement: If the expansion of $\left( x - \frac{1}{x^2} \right)^{2n}$ contains a term independent of $x$, then $n$ is a multiple of 2.
To Determine:
Whether the given statement is True or False.
Solution:
The given expansion is of the form $(a+b)^N$, where $a = x$, $b = -\frac{1}{x^2} = -x^{-2}$, and $N = 2n$.
The general term, $T_{k+1}$, in the expansion of $(a+b)^N$ is given by the formula:
$T_{k+1} = \binom{N}{k} a^{N-k} b^k$
Substitute the values of $N=2n$, $a=x$, and $b=-x^{-2}$ for the given expansion:
$T_{k+1} = \binom{2n}{k} (x)^{2n-k} (-x^{-2})^k$
Simplify the terms involving $x$ and the constant terms separately.
Constant part: $\binom{2n}{k} (-1)^k$
Variable part: $(x)^{2n-k} \cdot (x^{-2})^k = x^{2n-k} \cdot x^{-2k} = x^{2n-k-2k} = x^{2n-3k}$
The general term is $T_{k+1} = \binom{2n}{k} (-1)^k x^{2n-3k}$.
For the term independent of $x$, the power of $x$ must be zero. So, we set the exponent of $x$ equal to 0:
$2n - 3k = 0$
$2n = 3k$
For a term independent of $x$ to exist, there must be a non-negative integer value of $k$ (where $0 \leq k \leq 2n$) that satisfies this equation.
From $2n = 3k$, we see that $3k$ must be an even number (since $2n$ is even). For $3k$ to be even, $k$ must be an even number (since 3 is odd). Let $k = 2m$ for some non-negative integer $m$.
Substitute $k=2m$ into the equation:
$2n = 3(2m)$
$2n = 6m$
$n = 3m$
So, if a term independent of $x$ exists, $n$ must be a multiple of 3 (i.e., $n$ can be written as $3m$ for some integer $m$).
Also, the value of $k = \frac{2n}{3}$ must be an integer such that $0 \leq k \leq 2n$. If $n = 3m$, then $k = \frac{2(3m)}{3} = 2m$. The condition $0 \leq k \leq 2n$ becomes $0 \leq 2m \leq 2(3m)$, which simplifies to $0 \leq 2m \leq 6m$. This is true for any non-negative integer $m$. So, a term independent of $x$ exists if and only if $n$ is a multiple of 3.
The statement says that if a term independent of $x$ exists, then $n$ is a multiple of 2.
We found that $n$ must be a multiple of 3.
Let's check a case where $n$ is a multiple of 3 but not a multiple of 2. Let $n=3$. The expansion is $\left( x - \frac{1}{x^2} \right)^{2 \times 3} = \left( x - \frac{1}{x^2} \right)^6$. The term independent of $x$ exists because $n=3$ is a multiple of 3. The corresponding $k = \frac{2n}{3} = \frac{2(3)}{3} = 2$. $T_3 = \binom{6}{2} (x)^4 (-x^{-2})^2 = 15 x^4 x^{-4} = 15$. A term independent of $x$ exists.
According to the statement, since a term independent of $x$ exists, $n$ must be a multiple of 2. But $n=3$ is not a multiple of 2. This contradicts the statement.
The statement is False.
The statement is False.
The final answer is $\boxed{False}$.
Question 40. Number of terms in the expansion of (a + b)n where n ∈ N is one less than the power n.
Answer:
Given:
The statement: The number of terms in the expansion of $(a + b)^n$ where $n \in \mathbb{N}$ is one less than the power $n$.
To Determine:
Whether the given statement is True or False.
Solution:
The binomial theorem states that the expansion of $(a+b)^n$ is given by:
$(a+b)^n = \binom{n}{0}a^n b^0 + \binom{n}{1}a^{n-1} b^1 + \binom{n}{2}a^{n-2} b^2 + \dots + \binom{n}{n}a^0 b^n$
The terms in the expansion correspond to the values of the index in the binomial coefficient $k = 0, 1, 2, \dots, n$.
The number of possible values for $k$ is the number of integers from 0 to $n$, inclusive. This is $n - 0 + 1 = n+1$.
So, the total number of terms in the expansion of $(a+b)^n$ is $n+1$.
The statement claims that the number of terms is "one less than the power $n$", which means $n-1$.
We found the number of terms is $n+1$.
Since $n+1 \neq n-1$ for any $n \in \mathbb{N}$, the statement is false.
For example, if $n=1$, $(a+b)^1 = a+b$, which has 2 terms. The power is 1. $1+1 = 2$. $1-1=0$. The statement claims 0 terms.
If $n=2$, $(a+b)^2 = a^2 + 2ab + b^2$, which has 3 terms. The power is 2. $2+1 = 3$. $2-1=1$. The statement claims 1 term.
The statement is False.
The final answer is $\boxed{False}$.